The Craming Md

November 13, 2017 | Author: Rosalie Catalan | Category: Aorta, Spinal Cord, Cerebellum, Luteinizing Hormone, Thalamus
Share Embed Donate


Short Description

usmle pearls...

Description

1.What is associated with: Starry sky pattern? Burkitt's lymphoma

18.What is associated with: Kayser-Fleischer rings? Wilson's disease

2.Which organ most commonly recieves mets? Adrenal gland (rich bld

19.What is associated with: Lewy bodies? Parkinson's disease

supply)

20.What is associated with: Orphan Annie cells? Papillary carcinoma

3.What is the most common testicular tumor in children? in Men? Yolk

of the ovary

sac tumor, Seminoma

21.What is associated with: Russell bodies? Multiple myeloma

4.What is associated with: Auer's rods? Acute myelocytic leukemia

22.What is associated with: Reinke's crystals? Leydig cell tumor

(AML)-M3

23.What is associated with: Blue sclera? Osteogenesis imperfecta

5.What is associated with: Aschoff's bodies? Rheumatic fever

24.What is associated with: Soap-bubble appearance on an x-ray?

6.What is associated with: Birbeck granules? Histiocytosis X

Giant cell tumorof the bone

7.What is associated with: Neurofibrillary tangles? Alzheimer's

25.What is associated with: Pseudorosettes? Ewing's sarcoma

disease

26.What is associated with: Lucid interval? Epidural hematoma

8.What is associated with: Bence-Jones proteinuria? Multiple

27.What is associated with: Bloody tap on lumbar puncture?

myeloma

Subarachnoid hemorrhage

9.What is associated with: Cal-Exner bodies? Granulosa/thecal cell

28.What

tumor of the ovary

multiforme

10.What is associated with: Cowdry type A bodies? Herpes virus

29.What is associated with: Charcot-Leyden crystals? Bronchial

11.What is associated with: Codman's triangle on an x-ray?

asthma (eosinophil membranes)

Osteosarcoma

30.What is associated with: Cafe au fait spot on the skin?

12.What is associated with: Councilman bodies? Toxic or viral

Neurofibromatosis

hepatitis

31.What is associated with: Streaky ovaries? Turner's syndrome

13.What is associated with: Calf pseudohypertrophy? Duchenne's

32.What is associated with: Keratin pearls? Squamous cell carcinoma

muscular dystrophy

33.What is associated with: Signet ring cells? Gastric carcinoma

14.What is associated with: Reed-Sternberg cells? Hodgkin's

34.What is associated with: Mallory's bodies? Chronic alcoholism

lymphoma

35.What is associated with: Blue-domed cysts? Fibrocystic change of

15.What is associated with: Heinz bodies? G-6-PD deficiency

the breast

16.What is associated with: Homer-Wright rosettes? Neuroblastoma

36.What is associated with: Schiller-Duval bodies? Yolk sac tumor

17.What is associated with: Curschmann's spirals? Bronchial asthma

37.What is associated with: Senile plaques? Alzheimer's disease

(whorled mucous plugs)

38.What is associated with: WBCs in the urine? Acute cystitis

is

associated

with:

Pseudopalisades?

Glioblastoma

39.What is associated with: RBCs in the urine? Bladder carcinoma

55.What is the most common: Tumor of the liver? Metastatic cancer

40.What is associated with: RBC casts in the urine? Acute

(GI, breast, lungs)

glomerulonephritis

56.What is the most common: Malignant tumor of the esophagus?

41.What is associated with: WBC casts in the urine? Acute

Squamous cell carcinoma

pyelonephritis

57.What is the most common: Tumor arising within the bone? Multiple

42.What is associated with: Renal epithelial casts in the urine? Acute

myeloma

toxic or viral nephrosis

58.What is the most common: Primary malignant tumor of the female

43.What is associated with: Waxy casts? Chronic end-stage renal

genital tractin the world? Cervical neoplasia

disease

59.What is the most common: Primary malignant tumor of the female

44.What is the most common: Cause of chronic metal poisoning?

genital tractin the US? Adenocarcinoma of the cervix

Lead

60.What is the most common: Tumor of the female genitourinary

45.What is the most common: Cause of congenital cyanotic heart

tract? Leiomyoma

disease? Tetralogy of Fallot

61.What is the most common: Benign tumor of the ovary?

46.What is the most common: Congenital cardiac anomaly?

Serocystadenoma

Ventricular septal defect (VSD)

62.What is the most common: Benign tumor of the breast?

47.What is the most common: Cardiac tumor? Left atrial myxoma

Fibroadenoma

48.What is the most common: Vasculitis? Temporal arteritis

63.What is the most common: Benign lesion that affects the breast?

49.What is the most common: Primary tumor of the liver?

Fibrocystic change of the breast

Hemangioma (benign)

64.What is the most common: Malignant tumor of the breast? Invasive

50.What is the most common: Primary malignant tumor of the lungs?

ductal carcinoma

Adenocarcinoma (30% to 35%)

65.What is the most common: Tumor in men between the ages of 15

51.What is the most common: Cause of nephrotic syndrome?

and 35? Testicular tumors

Membrano proliferative glomerulonephritis

66.What is the most common: Germ cell tumor in men? Seminoma

52.What is the most common: cause of nephrotic syndrome in

67.What is the most common: Testicular tumor in infants and

children? Lipoid nephrosis

children? Yolk sactumor

53.What is the most common: Organism that causes pyelonephritis?

68.What is the most common: Malignant germ cell tumor in women?

Escherichia coli

Choriocarcinoma

54.What is the most common: Renal cell cancer type? Clear cell

69.What

is

the

most

common:

Solid

tumor

in

the

body?

83.What is the most common: Chromosomal disorder involving sex

Nephroblastoma

chromosomes? Kleinfelter's syndrome

70.What is the most common: Acquired GI emergency of infancy?

84.What is the most common: Cardiac pathology in patients with

Necrotizing enterocolitis of infancy

SLE? Libman-Sacks endocarditis

71.What is the most common: Primary malignant tumor of the ovary?

85.What is the most common: Cause of urinary tract obstruction?

Serocystadenocarcinoma

BPH

72.What

is

the

most

common:

Cardiac

tumor

of

infancy?

86.What is the most common: Eye tumor in children? Retinoblastoma

Rhabdomyoma

87.What is the most common: Intraspinal tumor? Ependymoma

73.What is the most common: Acute metal poisoning? Arsenic

88.What is the most common: Lymph node affected in non-Hodgkin's

74.What is the most common: Proliferative abnormality of an internal

lymphoma? Periaortic lymph nodes

organ?

89.What is the most common: Renal pathology in patients with SLE?

Benign prostatic hyperplasia (BPH)

Diffuse proliferative GN

75.What is the most common: Malignant tumor in the bone of

90.What is the most common: Cause of cirrhosis in the USA? Alcohol

teenagers? Osteosarcoma

91.What is the most common: Malignant tumor in women? Breast

76.What is the most common: Site of a cerebral infarct? Middle

92.What is the most common: Cancer of the vulva? Squamous cell

cerebral artery

carcinoma

77.What is the most common: Cause of dementia between the ages

93.What is the most common: Testicular tumor in children? Yolk sac

of 60 and 90 years? Alzheimer's disease

tumor

78.What is the most common: Primary CNS tumor in adults?

94.What is the most common: Benign GI tumor? Leiomyoma

Glioblastoma multiforme

95.What is the most common: Thyroid cancer? Papillary carcinoma

79.What is the most common: Primary CNS tumor in children?

96.What is the most common: Malignancy in children? ALL

Medullablastoma

97.What is the most common: Cause of diarrhea in children?

80.What is the most common: Tumor on sun-exposed sites? Basal

Rotavirus

cell carcinoma

98.What is the most common: Cause of hospitalization in children

81.What is the most common: Chromosomal disorder? Down

younger than 1 year of age? Respiratory syncytial virus (RSV)

syndrome (trisomy 21)

99.What is the most common: Helminthic parasite worldwide? Ascaris

82.What is the most common: Heart defect in Down syndrome?

lumbricoides

Endocardial cushion defect

100.What is the most common: Cause of anovulation? Polycystic

114.What is the most common: Hematologic cause of papillary

ovaries

necrosis? Sickle cell disease

101.What is the most common: Cause of death in neonates?

115.What is the most common: Organ involved in amyloidosis?

Neonatal respiratory distress syndrome (NRDS)

Kidney

102.What is the most common: Cardiac anomaly in children? Patent

116.What is the most common: Cause of abnormal bleeding?

ductus arteriosus (PDA)

Thrombocytopenia

103.What is the most common: Congenital heart defect in adults?

117.What is the most common: Cause of a nontraumatic splenic

Atrial septal defect (ASD)

rupture? Malaria

104.What is the most common: Complication of PDA? Subacute

118.What is the most common: Cause of death in SLE? Renal failure

bacterial endocarditis

119.What is the most common: Cause of infection for a patient on a

105.What is the most common: Cardiac anomaly in Turner's

ventilator? Pseudomonas aeruginosa

syndrome? Coarctation of the aorta

120.What is the most common: Esophageal carcinoma? Squamous

106.What is the most common: Cause of restrictive cardiomyopathy?

cell carcinoma

Amyloidosis

121.What is the most common: Cause of chronic pancreatitis? Alcohol

107.What is the most common: Cause of pulmonary hypertension in

abuse

children? VSD

122.What is the most common: Cause of infectious pancreatitis?

108.What is the most common: Cause of reversible hypertension in

Mumps

the USA? Alcohol abuse

123.What is the most common: Complication of nasogastric tube

109.What is the most common: Inflammatory arthritis? Rheumatoid

feeding? Aspiration pneumonia

arthritis

124.What is the most common learning disability? Dyslexia

110.What

is

the

most

common:

Cause

of

spontaneous

pneumothorax? Emphysematous bleb 111.What

is

the

most

common:

125.What is the most common: Cause of insomnia? Depression 126.What is the most common: Form of necrosis? Coagulative

Cause

of

nonorganic

127.What is the most common: Cause of blindness worldwide?

pneumoconiosis? Asbestosis

Chlamydia trachomatis

112.What is the most common: Cause of painless hematuria? Renal

128.What is the most common: Cause of blindness in the USA?

cell carcinoma

Diabetes mellitus{india ---vit a deficieny}

113.What is the most common: Cause of hematuria? Infection

129.What is the most common: Cause of the croup? Parainfluenza virus

130.What is the most common: Cause of a cold in the winter and

145.What

summer? Coronavirus

Chromosome 15,17

131.What is the most common: Cause of a cold in the spring and fall?

146.What chromosome: Follicular lymphoma? Chromosome 14,18

Rhinovirus

147.What

132.What is the most common: Cause of viral pneumonia leading to

Chromosome 5p

death? RSV

148.What chromosome: Patau's syndrome? Chromosome 13

133.What is the most common: Pituitary tumor? Chromophobe

149.What chromosome: Neurofibromatosis I? Chromosome 17

adenoma

150.What chromosome: Huntington's disease? Chromosome 4p

134What is the most common: Cause of panhypopituitarism?

151.What chromosome: Familial hypercholesterolemia?

Sheehan's syndrome

Chromosome 19

135.What is the most common: Cause of Cushing's syndrome?

152.What chromosome: Gaucher's disease? Chromosome 1

Pituitary adenoma

153.What chromosome: Neimann-Pick disease? Chromosome 11p

136.What is the most common: Kidney stone type? Calcium oxalate

154.What chromosome: Tay-Sachs disease? Chromosome 15q

137.What is the most common: Site of ischemia in the GI tract?

155.What chromosome: Cystic fibrosis? Chromosome 7

Splenic flexure

156.What chromosome: Albinism? Chromosome llp

138.What is the most common: Cause of intestinal obstructions in

157.What

adults? Adhesions and hernias

Chromosome 12

139.What is the most common: Cause of neonatal bowel obstruction?

158.What chromosome: Marfan's disease? Chromosome 15

Hirschsprung's disease

159.What chromosome: Neurofibromatosis II? Chromosome 22q

140.What is the most common: Cause of rectal bleeding?

160.What chromosome: Down syndrome? Chromosome 21

Diverticulosis

161.What chromosome: Edward's syndrome? Chromosome 18

141.What chromosomal translocation is associated with: Chronic

162.What mineral is associated with impaired glucose tolerance?

myeloid

Chromium (Cr)

leukemia

(CML?

Chromosome

9,22

(Philadelphia

chromosome:

Acute

chromosome

chromosome:

is

promyelocytic

associated

with:

leukemia

Cru

(M3)?

di

chat?

Chronic lymphocytic leukemia (CLL)?

chromosome)

163.What mineral is associated with hypothyroidism? Iodine (I)

142.What chromosome: Ewing's sarcoma? Chromosome 11,22

164.What mineral is an important component of the enzyme xanthine

143.What chromosome: Adult familial polyposis? Chromosome 5,21

oxidase? Molybdenum (Mb)

144.What chromosome: Burkitt's lymphoma? Chromosome 8,14

165.What vitamin deficiency has the following signs: angular stomatitis, glossitis, and cheilosis? Riboflavin (B2) deficiency

166.What vitamin is a component of the coenzyme thiamine

178.What vitamin requires intrinsic factor (IF) for absorption?

pyrophosphate (TPP)? Thymine (Bl)

Cyanocobalamin (B12)

167.Avidin decreases the absorption of what vitamin? Biotin. Avidin is

179.What mineral is a component of cytochrome a/a3? Copper (Cu)

found in raw egg whites.

180.Leukopenia, neutropenia, and mental deterioration are signs of

168.What are the four Ds of niacin deficiency? 1. Diarrhea 2.

what mineral deficiency? Copper (Cu) deficiency

Dermatitis 3. Dementia 4. Death

181.What vitamin deficiency causes a glove-and-stocking neuropathy

169.What mineral is an important component of glutathione

seen in alcoholics? Pyridoxine (B6) deficiency

peroxidase? Selenium (Se)

182.What mineral deficiency involves blood vessel fragility? Copper

170.What mineral deficiency in children is associated with poor growth

(Cu) deficiency

and impaired sexual development? Zinc (Zn) deficiency

183.Megaloblastic anemia and thrombocytopenia are signs of what

171.What mineral, via excessive depositions in the liver, causes

vitamin deficiency? Folic acid deficiency

hemochromatosis? Iron (Fe)

184.What is the antidote for an overdose with: Carbon monoxide?

172.What

vitamin

is

needed

in

the

production

of

heme?

Oxygen

Pyridoxine(B6)

What is the antidote for an overdose with: Mercury? Dimercaprol

173.What vitamin is a component of the enzymes fatty acid synthase

What is the antidote for an overdose with: Isoniazid? Pyridoxine

and acyl CoA? Pantothenic acid

What is the antidote for an overdose with: Atropine? Physostigmine

174.What vitamin deficiency has the following signs: homocysteinuria

What is the antidote for an overdose with: Arsenic?

and methylmalonic aciduria? Cyanocobalamin (B12) deficiency [Folic

Dimercaprol, D-penicillamine

acid deficiency has only homocysteinuria as a sign.]

What is the antidote for an overdose with: Digoxin?

175.What vitamin deficiency is evidenced by the following signs: poor

Antidigoxin Fab fragments

wound healing, loose teeth, bleeding gums, petechiae,

What is the antidote for an overdose with: Gold? Dimercaprol

and

ecchymosis? Ascorbic acid (vitamin C) deficiency (These are the

What is the antidote for an overdose with: Ethylene glycol?

signs of scurvy.)

Ethyl alcohol

176.What vitamin is given as prophylactic treatment for patients who

What is the antidote for an overdose with: Opiates/narcotics?

suffer from alcoholism? Thiamine (B1)-to prevent Wernicke's

Naloxone, naltrexone

encephalopathy and Korsakoff's encephalopathy

What is the antidote for an overdose with: Organophosphates?

177.What are the three carboxylase enzymes that require biotin? 1.

Atropine, 2-PAM

Pyruvate 2. Acetyl CoA 3.Propionyl CoA carboxylase

What is the antidote for an overdose with: Warfarin? Vitamin K

What is the antidote for an overdose with: Copper? D-Penicillamine

chromosome 2.Miillerian inhibiting factor (MIF) from Sertoli cells

What is the antidote for an overdose with: Heparin? Protamine sulfate

3.Testosterone from Leydig cells

What is the antidote for an overdose with: Iron? Deferoxamine

11.Where does the embryologic foregutend? At the first part of the

What is the antidote for an overdose with: Cyanide? Amyl nitrate,

duodenum

sodium nitrate, or sodium thiosulfate

12.What is the artery of the embryonic hindgut? The inferior

What is the antidote for an overdose with: Methyl alcohol?

mesenteric artery

Ethyl alcohol

13.What

What is the antidote for an overdose with: Acetaminophen?

Cytotrophoblast 2. Symcytiotrophoblast 3. Extraembrvonic mesoderm

N-Acetylcysteine

14.Which neuropore closes last? Caudal-and it is the first to open,

What is the antidote for an overdose with: Nitrates? Methylene blue

too.

What is the antidote for an overdose with: Lead? EDTA (calcium

15.What is the artery of the embryonic midgut? The superior

disodium edetate), dimercaprol, succimer

mesenteric artery

1.What structure is derived from the prochordal plate? The mouth

16.From where are nephrons derived embryonically? Metanephros

2.What is the only organ supplied by the foregut artery that is of

17.What are the five derivatives of the ventral mesentery? 1. Falciform

mesodermal origin? Spleen

ligament 2. Hepatoduodenal ligament 3. Hepatogastric ligament 4 and

3.In

which

direction

and

how

far

does

the

gut

rotate?

three

embryonic

cell

layers

form

the

chorion?

1.

5. Coronary andtriangular ligaments of the liver. All else is derived

Counterclockwise 270 degrees

from the dorsal mesentery.

4.What structure connects the primitive gut to the yolk sac? The yolk

18.When do the septum primum and the septum secundum of the

stalk (vitelline duct)

heart fuse? After birth

5.What is the artery of the embryonic foregut? The celiac artery

19.The cerebral cortex is a derivative of what? The telencephalon

6.When does the primitive gut herniate out of the embryo? 6 weeks

20.What is the adult structure found in the embryo as the: Umbilical

7.When does it return back into the embryo? 10 weeks

vein?Ligamentum teres

8.What two pathologic conditions occur when the gut does not return

What is the adult structure found in the embryo as the: Ductus

to the embryo? Omphalocele and gastroschisis

venosus? Ligamentum venosum

9.Around what structure does the midgut rotate?

What is the adult structure found in the embryo as the: Foramen

Superior mesenteric artery

ovule? Fossa ovule

10.What three things cause the indifferent gonad to become a testis?

What is the adult structure found in the embryo as the: Ductus

1.Testis-determining factor (TDF) from the short arm of the Y

arteriosus? Ligamentum arteriosum

What is the adult structure found in the embryo as the: Umbilical

33.From what are the pulmonary trunk and the ascending aorta

artery? Medial umbilical ligament

derived? Truncus

21.Where does the hindgut end? At the superior portion of the anal

arteriosum

canal

34.What disorder will result when there is a failure of the urachus to

22.Where does the midgut end? At the right two thirds of the

close, ca

transverse colon

using a leakage of urine out of the umbilicus? Urachal fistula

23.From where is the tongue musculature derived? Occipital somites

35.The common carotid and the internal carotid arteries are

24.What two branchial arches contribute to the formation of the

derivatives of what

anterior two thirds of the tongue? First and some of the second

embryonic structure? Third aortic arch

25.What are the two fourth pharyngeal pouch derivatives? Superior

36.The palatine tonsils are derived from what embryonic structure?

parathyroid glands and the ultimobranchial body

Second p

26.What two structures are derived from the fourth aortic arch? Arch

haryngeal pouch

of the aort

37.What are the sixth aortic arch derivatives? Right and left

a and the right subclavian artery

pulmonary arterie

27.What adult structures are derived from preotic somites? Muscles of

s and the ductus arteriosus

the i

38.The stapedial artery is derived from what? Second aortic arch

nternal eye

39.The mesonephric ducts contribute to what renal structures? The

28.What structure is derived from the first pharyngeal pouch? The

collecting d

middle ear

ucts, calyx, renal pelvis, and ureters

29.What two branchial arches contribute to the posterior two thirds of

40.Of what embryonic structureis the coronary sinus a derivative? The

the tongu

left

e? Third and part of the fourth

horn of the sinus venosus

30.What are the two third pharyngeal pouch derivatives? Inferior

41.What cranial nerve (CN) is associated with the: First pharyngeal

parathyroid gla

arch?

nds and the thymus

CN V

31.What structure is derived from the first aortic arch? Maxillary artery

What cranial nerve (CN) is associated with the: Second pharyngeal

32.From what are the urinary bladder and the urethra derived?

arch? CN VII

Urogenital sinus

What cranial nerve (CN) is associated with the: Third pharyngeal

perineum? Urorectal septum

arch? CN IX

49.In the adult, the thoracic veins are derived from what structure?

What cranial nerve (CN) is associated with the: Fourth pharyngeal

The card

arch? CN X

inal veins

What cranial nerve (CN) is associated with the: Fifth pharyngeal arch?

50.The gastrointestinal tract and abdominal veins are derived from

None-it

what structur

degenerates

e? Vitelline veins

What cranial nerve (CN) is associated with the: Sixth pharyngeal

51.From what is the thyroid gland derived? The floor of the endoderm

arch? CN X

(the p

42.From where is the external auditory meatus derived? First

osterior aspect of the tongue)

pharyngeal groove

52.The thalamus and its related structures are derivatives of what?

43.From where is the smooth portion of the right atrium derived? Right

The dien

ho

cephalon

rn of the sinus venosus

1.Transcriptionally active DNA is known as what? Euchromatin

44.Meckel's diverticulum is a remnant of what embryonic structure?

2.Transcriptionally inactive DNA is called? Heterochromatin

Vitellin

3.What is the only histone not found inside the nucleosomes? H1

e duct (yolk stalk)

histone-Its f

45.The pons and cerebellum are derived from what portion of the

unction is to bind nucleosomes together.

embryonic neural

4.What coating protects proteins from intracellular degradation?

tissue? Metencephalon

Clathrin

46.The medulla is a derivative of what portion of the embryonic neural

coating

tissue?

5.A nucleosome is made up of what two components? Histories and

Myelencephalon

DNA

47.What structure "tells" the overlying cells to begin neurulation? The

6.What are the four functions of smooth endoplasmic reticulum

noto

(SER)? 1. Stero

chord

id synthesis 2. Drug detoxification 3. Ca2+ handling 4. TAG

48.What structure splits the cloacal membrane, resulting in the

resynthesis

formation of the

7.What are the long microvilli found in the inner ear and the male

16,What cell surface modification of ependymal cells and respiratory

reproductive

epithelium

tract called? Stereocilia

has a 9 + 2 microtubular configur- ation and movement as its

8.What cell junction type allows for communication between two

function? Cilia

adjacent cells?

17.What protein binds hemidesmosomes to the basal lamina? Integrin

Gap junctions (nexus)

18.What intermediate filament is found in the zona adherens? Actin

9.Where are the enzymes for ATP production and the ETC located?

19.The proteins to be exported or incorporated into the lysosome are

Inner fold of th

produced by

e mitochondria membrane

what organelle? Bound polysome (polyribosome), attached to rough

10.What cell membrane structure increases the surface area of a cell

endopla

and has act

smic reticulum (RER)

in randomly assorted within its structure? Microvillus

20.What is the function of the zonula occludens and the zonula

11.What is the function of desmosomes? To hold adjacent cells

adherens?

together (i.e., a

To provide attachment between contiguous cells and to maintain a

dhesion)

semipermeable b

12.What is the microtubule configuration of a basal body? 9 + 0

arrier

microtubul

The following intermediate filaments are associated with what cell

e arrangement

types: Desmin

13.What are the four components of the basement membrane? 1.

? Muscle cells

Laminin 2. He

The following intermediate filaments are associated with what cell

paran sulfate (heparitin sulfate) 3. Fibronectin 4. Type IV collagen

types: Cytoke

14.The proteins that are to stay within the cell are produced by what

ratins? Epithelial cells

organelle?

The following intermediate filaments are associated with what cell

Free polysome (polyribosome)

types: Viment

15.What is the lysosomal post- translational modification of proteins?

in? Mesenchymal cells

Phosphor

The following intermediate filaments are associated with what cell

ylation of mannose residues

types: Neurof ilaments? Neurons

The following intermediate filaments are associated with what cell

30.Where is tropocollagen aggregated to form a collagen fibril?

types: Glial

Extracellularly

filaments? Astrocytes

31.What cell surface extension allows osteocytes in the lacunaeto

21.The basal lamina + the reticular lamina = what? The basement

"talk" to each

membrane

other? Canaliculi

22.What is the name of the organelle where collagen is made? Rough

32.What cell type produces myelin in the CNS? Oligodendrocytes

endoplasmi

33.In which ventricles is/are choroid plexus found? All four ventricles

c reticulum (RER)

34.What muscle type has calmodulin? Smooth muscle

23.What vitamin is needed for the hydroxylation of proline and lysine

35.What element is needed for the proper alignment of the

in collage

tropocollagen molecule

n synthesis? Vitamin C - {DEFICIENCY--Survy}

s? Copper (Cu+)

24.What are the two amino acids that cross-link elastin molecules?

36.What is added to the procollagen molecules to prevent intracellular

Desmosin

precipita

e and isodesmosine

tion? Registration peptides

25.What is the major inorganic component of bone? Hydroxyapatite

37.In what tissue can you find intercalated disks? Cardiac muscle

26.What cell in bone is a part of the mononuclear phagocytic system?

38.What are intercalated disks? Dense bands containing intercellular

Osteocla

junctions t

sts

hat link adjacent cells mechanically and electrically

27.What are the two types of cells located in the perichondrium of

39.Of what are intercalated disks composed? Fascia adherens

cartilage?

(mainly) Desmoso

Fibroblasts and chondroblasts

mes Gap junctions

28.What cell in the CNS is part of the mononuclear phagocytic

40.What are the proteoglycans of cartilage and bone? Chondroitin

system? Microgli

sulfate and

a{CNS PHAGOCYTE}

keratan sulfate

29.What substance, found in eosinophils, is toxic to parasitic worms?

41.What is the only glycosamino- glycan (GAG) that binds to the linker

Major ba

portion o

sic protein

f the proteoglycan? Hyaluronic acid (all sulfates bind to the core portion)

42.What is the portion of an axon that lacks myelin and is rich in

52.What is the dominant cell type in the red pulp of the spleen? Red

Na+/K+ pumps?

bloo

Node of Ranvier

d cells

43.What type of CNS cells have cilia, line the ventricles, and

53.On what area of the lymph node can you locate plasma cells?

contribute to the

Medulla

blood-brain barrier? Ependymal cells

54.What is the name of the area in the thymus where T cells are

44.What are the largest glial cells in the CNS (Hint: They contribute to

produced?

the blo

Hassall's corpuscle

od- brain barrier.)? Astrocytes

55.What type of muscle is striated and multinuclear? Skeletal muscle-

45.Myelin is produced by what type of PNS cells? Schwann cells

it of T

46.What is the dominant cell type in the lacunae of cartilage?

tubules and SR at the A-I junction

Chondrocytes

56.In what region of the spleen are the germinal centers located?

47.What structure runs perpendicular to the Haversian canals in the

White pu

bone?

lp-where B cell differentiation takes place

Volkmann's canal

57.What layer of the skin is missing in thin skin? Stratum lucidum

48.What are the three reasons for the effectiveness of the blood-brain

58.What are the phagocytic cells of the GI tract called? Paneth cells

barrier?

(Pa

1. Tight junctions 2. Capillaries that lack fenestration 3. Very selective

neth's granular cells)

pinoc

59.Which immunoglobulin is secreted by the plasma cells in the GI

ytosis by the capillaries

tract?

49.What types of muscle have troponin? Skeletal and cardiac

IgA

50.What type of muscle is uninuclear and nonstriated? Smooth

60.In what area of the spleen are the T cells located? Periarterial

muscle - it lacks

lymphatic s

T tubules and has gap junctions

heath (PALS)

51.On what area of the spleen are the APC{antigen presenting cells}s

61.In what area of the lymph node are the T cells and the APCs

located?

located? Paracort

Marginal zone

ical (thymic-dependent) area

62.What type of muscle is striated, branched, and uninuclear?

71.What are the three "tunica" layers of a blood vessel wall? 1. Tunica

Cardiac muscle-d

intima

yadic T tubules with SR at the Z line

2. Tunica media 3. Tunica adventitia

63.What does the tunica intima of arteries have that veins do not? An

72.Within what layer of the heart are the nerves and conducting fibers

inter

located?

nal elastic lamina

Subendocardial space

64.What two layers of skin makeup the malphighian layer? Stratum

73.Which organs have fenestrated capillaries with diaphragms?

basalis

Kidney, Intestin

and spinosus (mitotic area)

es, Endocrine organs

65.What type of skin cells have the mature melanin granules?

74.What secondary lymphoid tissue is encapsulated and has germinal

Keratinocytes. M

centers?

elanocytes inject melanosomes into the keratinocytes and mature

Peyer's patches{duodenum}

there.

75.What organs have sinusoid capillaries (leakiest type)? Liver, Bone

66.What type of skin cells are part of the mononuclear phagocytic

marr

system?

ow, Spleen

Langerhans' cells

76.What type of vessel has a thick tunica media? Arteries {Veins have

67.What cells of the epidermis, derived from the neural crest, act as

a thick tunica adventitia.}

mechano- r

77.Is the spleen a capsulated organ with trabeculae? Yes-although it

eceptors? Merkel's cells (Merkel's tactile cells)

does not

68.What layer of the epidermis acts as a sealant to protect against

have cortical or medullary regions

desiccation?

78.What secondary lymphoid organ is found just below the stratified

Stratum granulosum

squamous epi

69.What layer of the skin is composed of non-nucleated cells full of

thel- ium and is partially capsulated? Tonsils

keratin?

79.What type of capillary lacks fenestrations and has pinocytotic

Stratum corneum

vesicles?

70.What are the antigen- presenting cells in the Peyer's patches of

Continuous capillary

the GI known

80.What region of the body has fenestrated capillaries without

as? M cells

diaphragms?

Kidney glomeruli.

91.Secretin and CCK are produced in what portion of the GI tract?

81.Does the thymus have germinal centers? No. Germinal centers are

Small in

associa

testine

ted with B cells.

92.What cells of the stomach secrete pepsinogen? Chief cells

82.What cell of the liver is part of the mononuclear phagocytic

93.What cell type produces dentin of the teeth? Odontoblasts (neural

system? Kupffer

crest)

cells

94.What cell type produces enamel of the teeth? Ameloblasts

83.What papillae are respons- ible for sweet taste? Circumvalate

(ectoderm)

papillae

95.What gland produces 70% of the total salivary secretions and is a

84.What are the three epi- dermal derivatives? 1. Nails 2. Hair 3.

mixture of

Sweat glands

serous (predominantly) and mucous alveoli and secretory units?

(both apocrine and sebaceous)

Submandibular gl

85.What papillae send their senses via chorda tympani of CN VII?

and

Fungifor

96.What type of cells of the respiratory system secrete surfactant?

m papillae

Type II

86.What is the area of mitotic activity in the GI tract? The crypts of Li

pneumocytes

eberkuhn

97.What zone of the liver is the first to be affected in times of hypoxia?

87.What cells of the GI tract secrete HCl and intrinsic factor? Parietal

Central region (around the central vein)

cells o

98.In what region of the respiratory system do you first see Clara

f the stomach

cells?

88.What type of sweat gland is under cholinergic stimulation? Eccrine

Terminal bronchioles

gland

99.What substance does the juxtaglomerular cells of the kidney

89.What gland produces a serous secretion that is approximately 20%

secrete in respon

of the total

se to low blood pressure? Renin

saliva produced? Parotid gland

100.In what region of the liver is fat stored? Space of Disse -{Ito cells

90.What papillae are touch receptors on the tongue and send their

and V

sensations via

it A also present there}

CN V3 (mandibular division)? Filiform papillae

101.In what region of the GI tract does exfoliation take place? At the

110.What region of the liver is first affected in toxic doses of drugs?

tip of th

Peripher

e villi

al zone (because extraction of substances occurs there first)

102.What are the mucus secreting cells in the respiratory tract above

111.What are the two acidophilic hormones secreted by the

the level

adenohypophysis?

of the terminal bronchioles? Goblet cells

GH and prolactin "

103.What cells of the distal convoluted tubule are sensitive to the low

112.In what region of the kidney does the greatest extraction of

ion cont

nutrients occur

ent of the urine? Macula densa

? Proximal convoluted tubule (-66% of nutrient extraction occurs here)

104.What cells comprise 95% of the alveolar surface and are

113.What cells of the thyroid gland secrete calcitonin? Parafollicular C

responsible for gas

cells

exchange? Type I pneumocytes

114.What cells of the adrenal gland are neural crest derivatives?

105.What are the two hormones secreted by the posterior pituitary

Chromaff

gland via the

in cells (adrenal medulla)

para- ventricular supranuclei? ADH and oxytocin

115.What cells secrete glucagon? Alpha cells of the islet of

106.What area of the nephron is impermeable to water? Ascending

Langerhans

limb of the lo

116.What hormone inhibits glucagon release and pancreatic exocrine

op of Henle

secretions?

107.What region of the kidney is affected by ADH? Collecting duct for

Somatostatin

wate

117.What hormone causes milk letdown? Oxytocin

r reabsorption- but the DCT is also affected by ADH

118.Low levels of what hormone stimulates the uterus to go into its

108.What portion of the kidney actively pumps Cl- out of the tubule?

proliferativ

Thick as

e stage? Estrogen

cending limb of the loop of Henle

119.What cells of the genito- urinary system secrete testosterone?

109.What region of the kidney has a countercurrent multi- plier

Leydig c

producing a grad

ells (stimulated by LH)

ient of hypertonicity in the tubule lumen? Loop of Henle

120.What is the mucus-secreting gland in the male reproductive system? Bulboure

thral glands (Cowper's glands)

d by a sperm. If it is not, the egg is released in metaphase II and

121.Elevated levels of what hormone cause the endometrium to enter

meiosis is i

the secretory

ncomplete.

phase of the female cycle? Progesterone

What is the chromosome number at the end of meiosis I? 23 (2n)-it is

122.What are the cells of the parathyroid gland that produce

the reduct

parathyroid hormone

ive phase of meiosis.

(PTH)? Chief cells

127.What cell is under control of FSH and testosterone; secretes

123.What hormone produced during the night causes a decrease in

inhibin, MIF, a

gonadal function

nd androgen-binding protein; and phagocytizes the excess cytoplasm

? Melatonin

of the sperma

124.Where is melatonin produced? Pineal gland

tid? Sertoli cell

125.What are the four basophilic hormones released from the

128.What is the major androgen released from the zona reticularis?

adenohypophysis?

Dehydroe

1. Adrenocorticotrophic hormone (ACTH) 2. Thyroid-stimulating

piandrosterone (DHEA)

hormone (TSH) 3. L

129.What hormone causes an increase in the accumulation of

uteinizing hormone (LH) 4. Follicle-stimulating hormone (FSH)

adipose and collageno

126.What cells form the blood-testis barrier? Sertoli's cells

us tissue of the breast and an increase in the branching of the ducts

What is the chromosome number of-G1? 46 (2n)

of the bre

What is the chromosome number of-S phase? 46 (4n)

ast? Estrogen

What is the chromosome number of-G2? 46 (4n)

130.What promotes further prolactin and oxytocin release? Suckling

What is the chromosome number of-Mitosis? 46 (4n) to 46 (2n)

131.What part of the placenta is derived from the mother? Decidua

What is the chromosome number of a primary spermatocyte? 46 (4n)

basalis

In females, meiosis is arrested twice - when and at what stages of

132.What is the most common site of fertilization? Ampulla of the

meiosis?

fallopian

1. First, in utero at prophase I, 2. Second, at ovulation in metaphase II

tube

What must occur for an egg to complete ovulation? It needs to be

133.What is the only cranial nerve that comes off the dorsal surface of

fertilize

the brai n stem? CN IV

134.What type of fiber is carried in the dorsal root? Sensory or motor.

144.What area of the eye has the greatest visual acuity? Fovea (it is

Sensor

mad

y only

e up soley of cones)

135.How would a lower motor neuron (LMN) lesion present?

145.What cell type in the eye is for color vision? Cones (Cones and

Hyporeflexia, fa

color)

siculations and flaccid paralysis (always ipsilateral)

146.If there is macula sparing in a visual deficit, where is the lesion?

136.What is the name of the brain stem tract in which the dorsal

In the occipital lobe of the cerebral cortex (optic radiations)

columns run?

...

Medial lemniscus

147.Which way do the eyes drift in a frontal eye field lesion?To the

137.What is the ability to tell what something is without looking at it

side of the

and usin

lesion

g only your hands? Stereognosis

148.What is the thalamic relay nucleus that CN V needs to "speak" to

138.In what tract do pain and temperature fibers run? Spinothalamic

in order to

tract

pass

139.What gyrus in the cerebral cortex receives information from fibers

cortex?Ventroposteromedial(VPM)

of the do

149.Cell bodies of what fibers are found in the mesencephalic nucleus

rsal column tract? Postcentral gyrus

of CN V?Pr

140.What area of the brain is responsible for contralateral gaze?

oprioception of the face (CN V) and motor (jaw jerk reflex)

Frontal

150.If a patient presented with an LMN lesion in CN V, CN VII, or CN

eye field (Brodmann area 8)

XII, what w

141.What is the thalamic relay nucleus for the visual system? Lateral

ould you see? Ipsilateral paresis

genicula

151.What is the motor relay nucleus of the thalamus?Ventrolateral

te body (LGB)

(VL) nucleus o

142.What is the function of the ossicles? They increase the intensity

f thalamus

of s

152.What is the only cell type to leave the cerebellum?Purkinje

ound

(inhibitory) - G

143.What muscle in the eye is responsible for accommodation?

ABA

Ciliary muscle

its

information

on

to

the

cerebral

153.If a patient presented with a right-sided cerebellar lesion, which

162.What fluid of the inner ear has an electrolyte content like that of

way would

the extr

the patient fall if he closed his eyes? To the right

acellular fluid compartment (ECF)?Perilymph

154.What is the function of the superior olivary nucleus? To localize

163.What is the thalamic relay nucleus for the auditory system? MGB

and detemi

164.What region of the cerebellum is responsible for balance and eye

ne the nature of sounds (Sound and superior start with S.)

movement?

155.If a patient presents with a left nystagmus, where is the lesion?

Flocculonodular lobe

On the rig

165.What is the only cell in the cerebellum to have an excitatory

ht, because the nvstagmus is named for the fast component, and the

neurotransmitt

fast componen

er? Granule cell

t is to the unaffected side.

166.What does the nystagmus look like if cold water is placed in the

156.What region of the cerebellum is responsible for the planning of

right ear?

movements?

Slow drift to the right, fast drift to the left COWS = Cold Opposite -

Cerebellar hemisphere

Warm Same

157.What is the thalamic relay nucleus for the limbic system? Anterior

(named in reference to the fast component)

nucleus

167.Information from the cerebellum leaves via what? Superior

158.What fluid is found in the anterior chamber of the eye? Aqueous

cerebellar pedu

humor

ncle

159.What is the dividing line between the anterior and posterior

168.In what portion of the internal capsule are you if you can see the

chambers of the

caudate n

eye? The lens

ucleus? Anterior limb

160.If there is a total anopsia of the left eye, where is the lesion?

169.What type of memory is lost in a hippocampal lesion? Long-term

Optic nerv

memory

e of the left eye

170.In what region of the brain stem does the corticospinal tract cross

161.What is the center for ipsilateral gaze? The paramedian pontine

over?

reticular fo

Medullary decussation

rmation (PPRF)

171.From what gyrus of the brain does the corticospinal tract originate?

Precentral gyrus

178.In which region of the spinal cord does the spinothalamic tract

172.What type of fibers are carried in the ventral rami? Both sensory

cross over?

and

Ventral white commissure (VWC)

motor (from the spinal nerve on both sensory and motor fibers)

179.Sensory information from the spinothalamic tract sends its

173.What are the hallmark signs of an upper motor neuron (UMN)

information to wh

lesion? 1. Hyper

at region of the cerebral cortex? Postcentral gyrus

reflexia 2. Spastic paralysis 3. Positive Babinski sign

180.In which region of the brain stem do the dorsal columns cross

174.What tract carriers fibers for voluntary refined movements of the

over? Lower me

distal ext

dulla (synapse on nucleus gracilis or cuneatus)

remities? Corticospinal tract

181.What tract carries conscious proprioception,fine touch, two-point

175.What is the name of the tract in which the dorsal columns from

discrimina

the lower ext

tion, and vibratory sense? Dorsal column tract (all senses except pain

remities run? Fasciculus gracilis (It is medial of the two tracts on a

and

cross-s

temperature)

ection of the spinal cord; the lateral tract is the fasciculus cuneatus.)

182.What tract of the spinal cord carries dorsal column information

Rememb

from the upp

er: Lower extremities dancing-graceful-gracilis.

er extremities? Fasciculus cuneatus

176.What is the function of the superior colliculi? Cell bodies that are

183.If the right side of the corticobulbar tract to the muscles of facial

to

expres

be relayed to the thalamus for sight are found there. (Sight and

sion were damaged, where would the deficit be seen? In the

superior start

contralateral low

with S.)

er face (left)

177.In order for sensory information from the dorsal columns and the

184.If the corticobulbar tract for CN V and CN XII were cut on the right

spinothalam

side, w

ic tract to get to the cerebral cortex, they must use what thalamic relay

here would the lesion be? There would be no deficit, because the

nucleu

corticobu

s? Ventroposterolateral (VPL)

lbar tract receives bilateral input. 185.What type of fibers are carried in the ventral root? Motor only

186.What peduncle(s) carry information into the cerebellum? Inferior

ion of the optic radiation

and mid

196.What lesion produces a tremor upon movement? A cerebellar

dle cerebellar peduncles

lesion

187.Cell bodies of what fibers are found in the trigeminal ganglion?

197.What part of the inner ear is sensitive to angular acceleration and

Touch, p

decelera

ain, and temperature

tion? Semicircular canals

188.When the head moves, what causes the eyes to move in the

198.What is the normal volume of CSF? Approximately 140 ml{135ml

opposite direction?

most accurate

Vestibular system

ly}

189.Unconscious proprioception, body sense, and motor execution

199.What muscle of the eye is under parasympathetic control?

are functions of

Sphincter pupill

what part of the cerebellum? Vermis and intermediate lobe

ae (part of iris)

190.What three structures contain perilymph? 1. Scala tympani 2.

200.What cranial nerve receives sensory information from the

Scala vestib

cornea? CN VI (o

uli 3. Semicircular canals

phthalmic division)

191.The gravity receptors for changes in the position of the head are

201.What artery supplies blood to the trunk and the lower extremities

located in

on a homun

what part of the inner ear? Saccule and utricle

culus map of the cerebral cortex? Anterior cerebral artery

192.What cells are for black and white vision (night vision)? Rods

202.What structures of the inner ear contain endolymph? 1. Scala

193.What is the fluid of the posterior compartment of the eye?

media 2. Semici

Vitreous humor

rcular ducts 3. Saccule Utricle

194.What type of fluid in the inner ear has the consistency of

203.With what type of lesions do you see tremors at rest? Lesions of

intracellular flu

the b

id (ICF)? Endolymph (high levels of K+)

asal ganglia

195.Name

three

lesions

that

can

cause

left

homonymous

204.What muscle of the eye is under sympathetic control? Dilator

hemianopsia? 1. Lesio

pupillae

n of the right optic tract 2. Lesion of the lateral geniculate body (LGB)

(part of the iris)

3. Les

205.Where is the lesion if the patient presents with a right nasal

215.What tracts are found in the anterior limb of the IC?

hemianopsia?

Thalamocortical

Right internal carotid artery compression on the optic chiasm

tracts

206.What part of the inner ear functions in head movement?

216.If warm water is placed in the right ear, what does the nystagmus

Semicircular duc

look like?

ts

Slow drift to the left and fast drift to the right (COWS = Cold Opposite

207.What part of the internal capsule are you in if you see the

& Warm

thalamus?

Same)

Posterior limb of the internal capsule

217.What lesion causes a bitemporal hemianopsia? Optic chiasm

208.What region of the basal ganglia is affected in Parkinson's

lesion

disease?

218.What area of the brain is known as the motor speech area?

Substantia nigra (degeneration)

Broca's area

209.Hemorrhagic destruction of the contralateral subthalamic nuclei

219.What

results in w

homonymous superio

hat disorder? Herniballismus (wild flailing movements)

r quadrantopia

210.Slow writhing movements (athetosis) are caused by what?

220.Blood supply to the head/neck area and the upper limb on a

Hypermyelinizati

homunculus map in

on of the corpus striatum and the thalamus (seen in cerebral palsy)

the cerebral cortex comes from what artery? Middle cerebral artery

211.Atrophy of the striatum of the basal ganglia results in what?

221.What area of the brain is known for language comprehension?

Chorea (

Wernicke's area

involuntary quick movements)

222.Where is the lesion if the patient presents with a right

212.What tracts are found in the genu of the internal capsule?

homonymous inferior

Corticobulbar tr

quadrantanopia? Left upper loop lesion

acts

223.What region of the cerebellum is affected if a patient has dystaxia

213.What tracts are found in the posterior limb of the IC?

of the l

Corticospinal Sp

egs and trunk during walking? Anterior vermis (It is most commonly

inothalamic Dorsal column Thalamocortical

caused by c

does

Meyer's

hronic alcohol abuse.)

loop

lesion

cause?

Contralateral

224.Where is the lesion in a patient who presents with a broad-based

ord except the dorsal columns? Anterior spinal artery occlusion

gait, hypot

(posterior spin

onia, intention tremors, nystagmus, and ataxia? Cerebellum

al artery supplies the dorsal columns)

225.What are the functions of the external auditory meatus? Sound

232.What spinal cord lesion results in a bilateral loss of pain and

collection

temperature

and protection of the tympanic membrane

at the level of the lesion? Syringomyelia (VWC lesion)

226.What is the function of the inferior colliculi? To receive bilateral

233.What spinal cord lesion causes a bilateral dorsal column loss

aud

below the leve

itory input and arrange the input tonotopically

l of the lesion? Tabes dorsalis (neurosyphilis)

227.If a patient presents with nystagmus, dystaxia, and hypotonia of

234.What disease is associated with demyelination of the dorsal

the ipsilat

column, spinocer

eral limbs, what area of the cerebellum is affected? Hemisphere

ebellar

228.What lesion of the cerebellum is usually caused by an

degeneration

ependymoma or medullob

235.What

lastoma, resulting in dystaxia of the trunk with an inability to maintain

hemiparesis, contra

an upr

lateral spastic lower face, and ipsilateral oculomotor palsy (dilated,

ight posture? Posterior vermis lesion

ptosis, e

229.What spinal cord injury results in flaccid paralysis and muscle

ye down and out)? Posterior cerebral artery occlusion (ventral

atrophy?

midbrain sy

Polio (bilateral ventral horn lesion)

ndrome)

230.What spinal cord lesion results in a lower motor neuron (LMN)

236.What syndrome is associated with an ipsilateral UMN lesion

lesion at the

below the level o

level of the lesion and an upper motor neuron (UMN) lesion below the

f the lesion, ipsilateral dorsal column loss at and below the level of the

level of th

lesio

e lesion? Amyotrophic lateral sclerosis (ALS)-Lou Gehrig's disease

n, an LMN lesion at the level of the lesion, bilateral loss of pain and

231.What arterial occlusion would result in a loss of all tracts in the

temperat

spinal c

ure at the level an Brown-Sequard syndrome (heimisection of the

tract,

spinal cord)

and

arterial

corticospinal occlusion

tract?

results

in

Subacute

combined

contralateral

spastic

What arterial occlusion results in the following syndromes (Name

What arterial occlusion results in the following syndromes (Name

artery and spec

artery and spec

ific region.): Contralateral spastic hemiparesis of the body? Vertebral

ific region.): Nystagmus away from the lesion? Anterior inferior

artery

cerebellar art

-pyramid

ery- vestibular nuclei

What arterial occlusion results in the following syndromes (Name

What arterial occlusion results in the following syndromes (Name

artery and spec

artery and spec

ific region.): Contralateral loss of position and vibration? Vertebral

ific region.): Ipsilateral Horner's syndrome? Anterior inferior cerebellar

artery

art

-medial lemniscus

ery- descending autonomics

What arterial occlusion results in the following syndromes (Name

What arterial occlusion results in the following syndromes (Name

artery and spec

artery and spec

ific region.): Ipsilateral paralysis of the tongue? Vertebral artery-CN XII

ific region.): Ipsilateral facial paralysis? Anterior inferior cerebellar art

What arterial occlusion results in the following syndromes (Name

ery-CN Vll

artery and spec

What arterial occlusion results in the following syndromes (Name

ific region.): Ipsilateral limb ataxia? Anterior inferior cerebellar artery-

artery and spec

infe

ific region.): Deafness? Anterior inferior cerebellar artery-CN VIll

rior cerebellar peduncle

1.What is the name for the most prominent spinous process in the

What arterial occlusion results in the following syndromes (Name

spine? Vertebra

artery and spec

prominens (C7 in 70% of cases, C6 in 20%, T1 in 10%)

ific region.): Ipsilateral pain and temperature loss of the face? Anterior

2.What portion of the intervertebral disk is a remnant of the

inferior cerebellar artery-spinal nucleus of CN V

notochord?

What arterial occlusion results in the following syndromes (Name

Nucleus pulposus

artery and spec

3.What three muscles comprise the erector spinae? 1. Iliocostalis 2.

ific region.): Contralateral pain and temperature of the body? Anterior

Longi

inferio

ssimus 3. Spinalis

r cerebellar artery- spinotbalamic tract

4.What are the names given to the first and second cervical vertebrae? C1-atlas

C2-axis

erve

5.To what vertebral level does the spinal cord extend? LI to L2

16.What forms the anatomic snuff box? Extensor pollicis longus,

6.What is the name of the extension of the dura mater that attaches at

abductor polli

the level

cis longus, extensor pollicis brevis

of S2? External filum terminale

17.What vein, in the antecubital fossa, forms the communica- tion

7.How many pairs of spinal nerves exit from the spinal cord? 31 pairs

between the ba

8.What is the name of the region where the manubrium and the body

silic vein and the cephalic vein? Median cubital vein (most common

of the sternum

site fo

articulate? Sternal angle of Louis

r venipuncture)

9.What muscle originates from the third to the fifth ribs and inserts into

18.What two muscles are inner- vated by the axillary nerve? Deltoid

the c

and tere

oracoid process? Pectoralis minor

s minor

.10.Damage to what nerve will give you "winged scapula'."? Long

19.What nerve is compromised in carpal tunnel syndrome? Median

thoracic ne

nerve

rve To avoid confusing long thoracic nerve and lateral thoracic artery:

20.In what compartment of the thigh is the profundus femoris artery

Long has

found?

an "n" for nerve; lateral 3 & has an "a" for artery.

Anterior compartment (it's the blood supply to the posterior

11.The ventral rami of what regions of the spinal cord make up the

compartment)

brachial plex

21.Foot drop is caused by a compromise in what nerve? Common

us? C5-TI

peroneal nerve

12.What bone houses the ulnar groove? Humerus (between the

22.What nerve is damaged if the patient cannot ADduct the thigh?

medial epicondyle a

Obturato

nd the trochlea)

r nerve (nerve to the medial compartment of the thigh)

13.What muscle initiates Abduction of the arm? Supraspinakis

23.What is the longest muscle of the body? Sartorius

14.What muscle acts in all ranges of motion of the arm? Deltoid

24.What two nerves innervate the pectineus muscle? Femoral and

`

obturator ne

15.What nerve is damaged if a patient presents with "wrist drop"?

rves

Radial n

25.What superficial vein empties into the popliteal fossa? Short

36.How are they separated? By the oblique and the transverse

saphenous

fissures

vein

37.Into what chamber of the heart do the pulmonary veins empty? Left

26.What is the artery of the anterior compartment of the leg? Anterior

atrium (Rem

tibia]

ember-the pulmonary veins carry oxygenated blood.)

artery

38.What is the only valve in the heart that has two cusps? Mitral

27.What nerve supplies the lateral compartment of the leg?

(bicuspid

Superficial pero

) valve

neal nerve

39.What vein travels with the right coronary artery? Small cardiac vein

28.What sensory nerve are you testing when you touch the first web

40.At what vertebral level does the trachea bifurcate? T4-T5 (It is

space of the

known as th

toes? Deep peroneal nerve

e carina.)

29.The peroneal artery is a branch of what artery? Posterior tibial

41.What attaches the cusps of the valves to the papillary muscles in

30.Inflammation of the pre- patellar bursa is often referred to as what?

the heart?

Housemaid's knee

Chordae tendineae

31.What is the prominent "bump" on the lateral aspect of the knee?

42.Around what thoracic structure does the right recurrent laryngeal

Head of

nerve loop

the fibula

before ascending into the larynx? Right subclavian artery

32.How many ribs articulate with the sternum? Seven (Ribs 8, 9, and

43.At what vertebral level does the esophagus originate? C6

10 articul

44.At what level does the abdominal aorta bifurcate into the common

ate with the costal cartilage of rib 7.)

iliac arteri

33.What is the part of the lung that extends above the level of the first

es? L4-L5

rib?

45.The obturator artery is a branch of what major artery? Internal iliac

The cupula

a

34.What type of pleura is adherent to the surface of the organ?

rtery

Visceral pleura

46.What is the first branch off the abdominal artery? Inferior phrenic

35.How many lobes does the right lung have? Three

artery

47.Into what vessel does the right gonadal vein drain? The inferior

59.What structures differentiate the anatomic right and left lobes of

vena cava

the liver?

48.Into what vessel does the left gonadal vein drain? The left renal

Ligamentum teres and ligamentum venosum

vein

60.What structure "runs" along the transverse processes of the

49.At what vertebral level does the common carotid artery bifurcate?

lumbar vertebrae?

C4

Ureters

50.At what vertebral level is the hyoid bone found? C3

61.To enter into the lesser peri-toneal sac, you must traverse through

51.The ophthalmic artery is a branch of what vessel? Internal carotid

what fora

artery

men? Foramen of Winslow

52.What forms the portal vein? The union of the superior mesenteric

62.What is another name for the rectouterine pouch? Pouch of

and the spl

Douglas

enic veins

63.What bones comprise the acetabulum? Pubis, ilium, and ischium

53.Where does the inferior mesenteric vein drain? The splenic vein

64.What two ligaments of the uterus are remnants of the

54.What vein is formed by the union of the right and left

gubernaculum? Bound an

brachiocephalic veins?

d ovarian ligaments

Superior vena cava

65.What muscles comprise the deep perineal space (the urogenital

55.What is the only muscle in the larynx that is not inner- vated by the

diaphragm)?

recurre

Deep transverse perineal and sphincter urethrae

nt laryn- geal nerve? Cricothyroid (It's innervated by the external

66.What three ligaments com- prise the broad ligament of the uterus?

laryngeal

1. Mesos

nerve.)

alpinx 2. Mesovarium 3. Mesometrium

56.The folds of the mucosa of the stomach are known as what?

67.What structure traverses the diaphragm at the level of T8? IVC

Rugae

68.What are the components of the pudendal canal? Pudendal nerve

57.What is the artery of the embryonic foregut? Celiac artery

and inter

58.What comprises the portal triad? 1. Common bile duct 2. Hepatic

nal pudendal artery and vein

artery 3.

69.What

Portal vein

metacarpal/phalangealjoint?

range

of

movements

can

be

Flexion/extension, ABduction, and ADduction

performed

at

the

70.A fracture of the surgical neck of the humerus will most likely

79.What component of the corneal reflex is lost in a CN VII deficit?

damage what n

Motor as

erve? Axillary nerve

pect

71.What compartment of the lower extremity allows flexion of the hip

80.Toward what side would the uvula point if the right CN X were

and extensi

damaged?

on of the knee? Anterior compartment of the thigh

The left (points to the unaffected side)

72.What nerve roots comprise the lumbosacral plexus? L4 to S4

81.What is the name of the urinary bladder where the ureters enter

73.What is the function of gray rami communicans? They are

and the ureth

postganglionic

ra exits? Urinary trigone

sympathetic axons.

82.What is the only organ in the body supplied by preganglionic

74.What compartment of the lower extremities allows ADduction of

sympathetic fibe

the thigh and f

rs? Adrenal rnedulla

lexion of the hip? Medial compartment of the thigh

83.The pudendal canal is formed by splitting the fascia of what

75.What are the only splanchnics in the body that carry preganglionic

muscle? Obturato

parasympat

r internus

hetic

fibers?

Pelvic

splanchnics

(P

begins

preganglionic,

84.What is the name of the duct formed by the union of the vas

parasympathetic, and

deferens and the

pelvic.)

duct of the seminal vesicle? Ejaculatory duct

76.What postganglionic parasympathetic ganglion is associated with

85.What are the fingerlike projections at the end of the fallopian

CN III?

(uterine) tub

Ciliary ganglion

es? Fimbriae

77.What is the name of the ganglion that houses the cell bodies for

86.Where is the seminal vesicle located? On the posterior aspect of

the postgang

the u

lionic sympathetic fibers to the head and neck? Superior cervical

rinary bladder

ganglion

87.What vessel can be found atop the scalene anterior? Subclavian

78.What two muscles do you test to see if CN XI is intact? Trapezius

vein

and st

88.What muscle divides the anterior from the posterior triangles of the

ernocleidoinastoid

neck?

Sternocleidomastoid

Thoracic and sacral

89.Where does the parotid (Stenson's) duct enter the oral cavity?

100.What are the only muscles in the body innervated by dorsal rami?

Opposite

Intrinsi

the second upper molar tooth

c (deep) muscles of the back (All other muscles are innervated by

90.What is the function of the arachnoid granulations? Resorb CSF

ventral rami.)

into the bloo

101.What is the portion of the second cervical vertebra that projects

d

superiorly

91.What muscle is the most superiorly situated muscle in the orbit?

to act as the body for C1? Odontoid (dens) process

Levator

102.What is the actual space that contains CSF? Subarachnoid space

palpebrae superioris

103.What is the protective covering that is adherent to the spinal cord

92.What is the triad of Horner's syndrome? Miosis, ptosis, and

and CNS

anhydrosis

tissue? Pia mater

93.What bone of the middle ear articulates with the tympanic

104.What is the name of the spinal cord that passes within the

membrane? Malleus

subarachnoid spac

94.What chamber of the eye is located between the iris and the lens?

e that forms the spinal nerves that exit the lumbar and sacral

Posterio

foramina?

r chamber

Cauda equina

95.What bone houses the ear? Temporal bone

105.What are the names ligaments that would pierced, in order, by a

96.What is the only muscle of the tongue not innervated by the

lumbar punct

hypoglossal nerve

ure?

? Palatoglossus

Ligamentum flavum

97.Where does the nasolacrimal duct terminate? Inferior meatus of

106.What is the inferiormost segment of the sternum? Xiphoid

the nasal cav

process

ity

107.True or false-the pectoralis major medially rotates the arm? True;

98.What gland is found in the muscular triangle of the neck? Thyroid

it

gland

also ADducts and flexes the arm.

99.What two regions of the vertebral column are con- sidered primary

108.What are the borders of the axillary artery? Lateral border of the fi

curvatures?

rst rib to the inferior border of the teres major

1.

Supraspinous

ligament

2.

Interspinous

ligament

3.

109.What vessels arise from the three segments of the axillary artery?

he hand innervated by the median nerve: Lumbricales, Opponens

1. Super

pollicis, Abductor

ior thoracic artery 2. Lateral thoracic artery and thoracoacromial trunk

pollicis brevis, and Flexor pollicis brevis; All other intrinsic muscles in

3. Subs

the

capular artery, and the anterior and posterior humeral circumflex One

hand are innervated by the ulnar nerve

artery fro

117.What muscles in the hand ADduct the fingers? The palmer

m the first segment, two arteries from the second segment, and three

interosseus a

arteries fr

dducts, whereas the dorsal interosseus abducts (PAD and DAB)

om the

118.In order to pronate the hand, what bones need to cross? Radius

110.What muscle is the main lateral rotator of the arm? Infraspinatus

crosses o

muscle

ver the ulna

111.What

innervates

the

flexor

compartment

of

the

arm?

119.At what point does the femoral artery become the popliteal

Musculocutaneus nerve

artery? When it

112.What nerve is most commonly affected when there is a fracture of

traverses the adductor hiatus

the midshaf

120.Loss of ABduction of the lower limbs results in Trendelenburg

t of the humerus? Radial nerve C deer "(Int4.1 a .

gait; what ner

113.What vein courses along the medial aspect of the forearm?

ve is compromised to cause this? Superior gluteal nerve

Basilic vein

121.What two arteries join together to form the super- ficial and deep

114.What is the blood vessel in the upper extremity most commonly

palmar ar

palpated while

ches of the hand? Uhiar and radial arteries (ulnar is the main supplier)

taking a pulse? Radial artery

122.What muscle "fills" the greater sciatic foramen? Piriformis

115.What is the nerve supply to the forearm? Median nerve (except

123.What nerve is affected when a patient has difficulty rising from a

for the fle

sitting p

xor carpi ulnaris and flexor digiti profundus muscles of the pinkie and

osition? Inferior gluteal nerve (nerve to the gluteus maximus)

ring fin

124.Why are IM injections in the gluteal mass given in the upper outer

ger, which are supplied by the ulnar)

quadrant?

116.What are the "LOAF" muscles of the hand? LOAF stands for the

To avoid damage to the sciatic nerve

muscles of t

125.What two nerves innervate the adductor magnus? Obturator and

Why? Because the neurovascular bundle is located on the inner

tibial ner

surface of the

ves

inferior border of the rib

126.What two nerves innervate the biceps femoris? Common

134.What muscles of the foot are supplied by the medial plantar nerve

peroneal and tibi

(Hint: Thi

al nerves

nk about the median nerve distribution in the hand.)? LAFF-

127.Going from lateral to medial, what structures pass deep to the

Lumbricalis (1st),

inguinal liga

Abductor hallucis, Flexor 3X hallucis brevis, Flexor digitorum brevis.

ment? NAVEL-Nerve, Arterv, Vein, Empty space, and Lacunar

All other

ligament or Lymphati

intrinsic muscles in the foot are supplied by the lateral plantar nerve.

cs

135.What remnant of the middle lobe of the lung is found on the left

128.What artery turns into the dorsalis pedis when it crosses the

side?

extensor retin

The lingula

aculum? Anterior tibial artery

136.The ventral rami of what cervical vertebrae innervate diaphragm?

129.What is the nerve for the anterior compartment of the leg? Deep

C3, C4,

peroneal ne

and C5 keep the diaphragm alive!

rve

137.At the level of rib 6, the internal thoracic artery divides into what

130.What is the artery for the posterior compartment of the leg?

two ar

Posterio

teries? Musculophrenic and superior epigastric arteries

r tibial arterv

138.What portion of the peri- cardium is adherent to the tunica

131.Where is the "magical plane" that divides the superior from the

adventitia of th

inferior med

e great vessels? Fibrous pericardium

ia- stinum? A horizontal line from T4-T5 to the sternal angle of Louis

139.The left anterior descending artery of the heart travels with what

132.What vein drains the lower third of the thoracic wall?

vein?

Hemiazygous vein

Great cardiac vein

133.If you were to do a pleural tap, what region of the intercostal

140.What is the largest muscle in the body? Gluteus maximus

space would

141.The middle cardiac vein of the heart travels with what artery?

your needle enter? The superior border of the rib

Posterio

r intraventricular artery

Four Ms and a U- 1. Median 2. Medial antebrachial 3. Medial pectoral

142.What is the ratio of the myocardial thickness of the atria: right

4. Medial b

ventricle:

rachial cutaneus 3X 5. Ulnar

left ventricle? 1:03:09

150.What are the five branches off the posterior cord of the brachial

143.What chamber of the heart comprises the: Sternal surface? Right

plexus?

ventricle

STARS- 1. Upper Subscapularis 2. Thoracodorsal 3. Axillary ; & 4.

Diaphragmatic surface? Right ventricle and left ventricle

Radial 5. Lowe

Left margin? Left ventricle and left atrium

r Subscapularis

Right margin? Right atria

151.What are the three branches off the lateral cord of the brachial

Base? Left atria

plexus?

144.What structure does the left recurrent laryngeal nerve loop

1. Lateral pectoral 2. Lateral head of the median 3. Musculocutaneus

around before it

152.What are the four branches off the brachial plexus that arise prior

ascends into the larynx? The arch of the aorta

to the f

145.At what point does the axillary artery become the brachial artery?

irst rib? 1. Dorsal scapular 2. Suprascapular 3. Long thoracic 4. Nerve

When it

to

crosses the teres major

subclavius

146.What is the anatomic posi- tioning of the right and left gastric

153.What nerve innervates the extensor compartment of the arm?

nerve plexu

Radial nerve (It

s of the esophagus as they pass through the diaphragm? LARP-

also innervates the extensor compartment of the forearm.)

147.Left goes Anterior and Right goes Posterior (because of the

154.What muscles insert in/on the intertubercular groove of the

rotation of the

humerus?

gut -Remember your embryology!)

Lady between two Majors- latissimus dorsi, pectoralis major, and

148.What muscles comprise the rotator cuff? SITS-Subscapularis,

teres major

Infraspinatu

155.What artery is found in the lateral compartment of the leg? None.

s, Teres minor, & Supraspinatus

The perone

149.What are the five branches off the median cord of the brachial

al artery is in the posterior compartment of the leg.

plexus?

156.What muscle laterally rotates the femur to "unlock" the knee? Popliteu

s

164.The inferior thyroid artery is a branch of what vessel?

157.What bursa is inflamed in "clergyman's knee"? Infrapatellar bursa

Thyrocervical tr

158.Where does the great saphenous vein terminate? In the femoral

unk

vein

165.What is the area of the carotid artery that is an 02 receptor?

159.What comprises the "unhappy triad" of the knee? 1. Medial

Carotid

collateral lig

body (The carotid sinus is a pressure receptor.)

ament 2. Medial meniscus 3. Anterior cruciate ligament (ACL) The

166.What arteries join together forming the basilar artery? Left and

severity of inj

right v

ury to these ligaments is ranked from bad to worst in relation to how

ertebral arteries

many of th

167.What is the major difference between the veins in the face and

em are damaged, and they are usually damaged in the order listed.

the veins in

160.What are the two branches off the external iliac artery before it

the rest of the body? No valves and no smooth muscle in the walls of

becomes th

the veins

e femoral artery? Circumflex iliac and inferior epigastric arteries

in the face

161.From which three sources does the adrenal gland get its blood

168.At what point does the sig- moid sinus become the internal

supply?

jugular vein?

1. Superior suprarenal artery (off inferior phrenic artery) 2. Directly off

When it crosses the jugular foramina

the

169.What connects the lateral ventricles to the third ventricle?

abdominal aorta as the middle suprarenal artery 3. Inferior suprarenal

Foramen

artery of

of Monro

f the renal artery

170.What connects the third and the fourth ventricles together?

162.What are the three branches off the celiac artery? 1. Common

Cerebral aqueduc

hepatic artery

t

2. Splenic artery 3. Left gastric artery

171.How does cerebrospinal fluid leave the fourth ventricle? Through

163.What are the three main branches off the inferior mesenteric

the fora

artery?

mina of Magendie (medial) and Luschka (lateral) M in Magendie =

1. Left colic artery 2. Superior rectal artery 3. Sigmoid artery

medial; L in Lus chka = lateral.

172.What is the lymphatic drainage of the gonads? Lumbar trunk

loic appendages

nodes (Lymp

181.What is the artery of the embryonic midgut? Superior mesenteric

hatic drainage follows blood supply.)

artery

173.What is the lymphatic drainage of the pelvic organs? Internal iliac

182.What two ligaments together comprise the lesser omentum?

n

Gastrohepatic an

odes

d hepatoduodenal

174.What are the five clinical signs of portal hypertension? 1. Caput

183.The quadrate and the caudate lobes are part of what side of the

medusae

anatomic liv

2. Hemorrhoids 3. Retroperitoneal varices 4. Splenomegaly 5.

er? The right

Esophageal varices

184.What is the artery of the embryonic hindgut? Inferior mesenteric

175.What is the region of the body where all tonsillar tissue can be

arte

found?

ry

Waldeyer's ring

185.The hepatic duct and the cystic duct come together to form what?

176.What are the three functions of the nasal cavity? It warms,

Common b

moistens, and

ile duct

filters inspired air.

186.What muscle forms the "bed" for the kidney? Quadratus

177.What region of the pharynx does the eustachian tube enter?

lumborum

Nasopharynx

187.What ligament of the uterus houses the ovarian vessels?

178.In which segment of the duodenum is the ampulla of Vater

Suspensory ligam

located? Second s

ent of the ovary

egment

188.What three muscles comprise the pes anserinus? 1. Sartorius 2.

179.The duodenal/jejunal flexure is suspended from the posterior

Gracilis

abdominal wall

3. Semitendinous

by what? Ligament of Treitz

189.What are the 10 retro- peritoneal organs? 1. Duodenum 2.

180.What are the three anatomic characteristics that differen- tiate the

Ascending Colon 3

large b

. Ureters 4. Pancreas 5. Suprarenals 6. Descending colon 7. Aorta 8.

owel from the small bowel and the rectum? 1. Tinea coli 2. Haustra 3.

Kidneys 9.

Epip

Rectum 10. Inferior vena cava

190.What is the only cranial nerve with the ability to regenerate? CN I

199.What area of the posterior aspect of the eye has no photo

191.What duct transmits secretions from the sub- mandibular gland to

receptors?

the oral ca

The optic disk is the blind spot.

vity? Wharton's duct

200.What muscles are found in the superficial perineal pouch?

192.What are the boundaries of the posterior triangle of the neck?

Superficial tran

The ster

sverse perineal, ischiocavernous, and bulbocavernous muscles

n ocleidomastoid, the trapezius, and the clavicle

201.What gland is found in the deep perineal pouch in men?

193.What are the contents of the adductor canal? Femoral artery and

Bulbourethral gl

vein

and; no gland is found in this pouch in women.

and saphenous nerve

202.What pouch of the peri- neum houses the superficial fascia and

194.What are the contents of the submandibular triangle of the neck?

the inferior

Submandi

fascia? The deep perineal pouch (same as the urogenital diaphragm)

bular gland, facial artery and vein, nerve to the mylohyoid

203.What gland is found in the superficial perineal pouch in men and

195.What carpal bones articulate with the radius? Scaphoid and

women?

lunate

None in men, the greater vestibular gland in women

196.What are the borders of the anterior triangle of the neck?

204.What are the five structures that traverse the spermatic cord? 1.

Sternocleidomast

Pampi

oid, mandible, and midline of the neck

niform plexus of veins 2. Vas deferens 3. Testicular artery 4. Nerves

197.What are the six cranial nerves that innervate structures in the

5. Lymphat

orbit?

ics

1. CN II-vision 2. CN Vl-sensory (ophthalmic division) 3. CN VII-

205.What are the borders of Hesselbach's triangle? Rectus abdominis

lacrimal gland

mediall

LR6 (S04)3 (to remember nerves below): 4. CN VI-lateral rectus 5.

y, Inferior epigastric vessels laterally, Inguinal ligament as the base

CN IV-superior

206.Which type of hernia goes through both the deep inguinal and

oblique 6. CN III-all other muscles of the eye

superficial ing

198.What muscle keeps the stapes taut against the oval window?

uinal ring? Indirect; a direct hernia goes directly through Hesselbach's

Stapedius muscle

tri angle

207.What structure(s) traverse the diaphragm at the level of T10?

216.If the left hypoglossal nerve was damaged, which way would the

Esophagu

tongue point?

s and gastric plexus of nerves

To the left (The uvula points to affected side.)

208.What structure(s) traverse the diaphragm at the level of T12?

217.In women, what is the name of the pouch between the bladder

Aorta, a

and the uterus c

zygous vein, and thoracic duct Remember: One at Ts, two at Tlo,three

alled? Uterovesical pouch

at T,Z.

218.What component of the pelvic diaphragm forms the rectal sling

209.What compartment of the lower extremity allows flexion of the

(muscle of con

toe, inversion

tinence)? Puborectalis

of the foot, and plantar flexion of the foot? Posterior compartment of

219.What is the name of the comma-shaped structure that is attached

the leg

to the poste

210.How are the arm and the forearm positioned in "waiter's tip

rior aspect of the testes? Epididymis

hand"? The arm

220.What is the structure of the uterus that projects above the

is medially rotated, and the forearm is extended and pronated.

opening of the u

211.What is the function of white rami communicans? They are

terine tubes? Fundus

preganglionic s

221.What is the region of the fallopian tube where fertilization most

ympathetic axons. They are white because they are myelinated.

commonly o

212.What compartment of the lower extremity allows extension of the

ccurs? Ampulla

hip and flex

222.How can you access the lowermost point in the peritoneal cavity

ion of the knee? Posterior compartment of the thigh

in women?

213.What compartment of the lower extremity allows dorsiflexion,

Via the posterior fomix of the vagina

extension of th

223.Which type of hemorrhoids are painful? External hemorrhoids;

e toes, and inversion of the foot? Anterior compartment of the leg

internal h

214.What cervical nerves comprise the cervical plexus? C1 to C4

emorrhoids lack pain fibers.

215.What compartment of the lower extremity allows eversion and

224.What are the five terminal branches of the facial nerve? 1.

plantar flexion

Temporal 2. Z

of the foot? Lateral compartment of the leg

ygomatic 3. Buccal 4. Mandibular 5. Cervical (Two Zebras Bit My Clavicle.)

225.What two vessels come together to form the external jugular

ion (V3)

vein? 1. Poste

234.What portion of CN V is affected if the corneal reflex is lost?

rior auricular vein 2. Posterior division of the retromandibular vein

Ophthalm

226.What is the position of the eyeball if CN VI is lost? Adducted

ic division (VI)

227.The thyroid gland receives blood from what two different

235.What cranial nerve is affected if you have a laterally deviated eye

sources? 1. Infer

that is

ior thyroid off the thyrocervical trunk 2. Superior thyroid artery off the

dilated with a ptosed eyelid? CN III

exter

236.Which cranial nerves are found in the midline of the brain stem?

nal carotid artery, and sometimes off the arch of the aorta as the

CN I, 11

thyroid ima a

, 111, VI, and XII ARN Add 1 + 1 = 2, 1 + 2 = VXF 3,1+2+3=6,1+2+

rtery

3+6=12

228.What postganglionic para-sympathetic ganglia is associated with:

237.What are the four muscles of mastication? 1. Masseter 2.

CN VII?

Temporalis 3. Med

COPS Submandibular ganglion

ial pterygoid 4. Lateral pterygoid

229.What postganglionic para-sympathetic ganglia is associated with:

1.What happens to prevalence as: Incidence increases? Increases

CN IX?

2.What happens to prevalence as: Duration increases? Increases

Pterygopalatine and otic ganglion

3.What is the formula for IQ? MA/CA x 100 (MA = mental age, CA =

230.What postganglionic para-sympathetic ganglia is associated with:

chronologic a

CN X?

ge)

Terminal ganglion

4.In statistics, what is the measured rate for: A whole population?

231.What type of fibers are carried in the thoracic and lumbar

Crude ra

splanchnics?

te

Preganglionic sympathetic fibers

5.What stage of sleep is associated with high pulse, blood pressure,

232.How are preganglionic parasympathetic fibers carried to the

and respira

embryonic hindgu

tion rates, and is characterized by increased brain oxygen use, penile

t? Via pelvic splanchnics

erection

233.What is the only portion of CN V that carries motor fibers?

in males, and total paralysis of the skeletal muscles? REM sleep

Mandibular divis

(Remember as:

Awake brain & paralyzed body.)

13.In biostatistics, what type of error is due to chance? Random error

6.What stage of sleep is associated with slow pulse and respiratory

14.Dementia is associated with a decrease in what neurotransmitter in

rates, a dec

the amygda

rease

in

blood

pressure,

and

involuntary

skeletal

muscle

la, hippocampus, and temporal neocortex? Acetylcholine (ACh)

contractions? Non-rapi

15.What symptoms of schizophrenia associated with dopamine

d eye movement (NREM) sleep (Remember as: Idle brain in an

receptors include del

awake body.)

usions, hallucinations, and agitation? Positive symptoms (type I)

7.What type of study is prospective-that is, it "looks to see who gets

16.In biostatistics, what type of error has unanticipated factors that

sick" by

obscure t

defining a population at risk of being exposed to a disease? Cohort

he relationship and cause a bias? Confounding error

study (al

17.What is the most frequently occurring value in a set? Mode

so called prospective, follow-up, longitudinal, or incidence study)

18.What is the difference between the highest and the lowest scores

8.What hormone is inhibited by sleep? TSH

in a set?

9.With

what

stage

of

sleep

are

nightmares

associated?

Range

REM=`remember them" ;

19/What type of study has diffusional effects if you separate the

10.With what stage of sleep are night terrors associated? Stage 4-

groups and tes

they are

t the entire population? Community trial

not remembered

20.What type of study is under the greatest possible degree of control

11.What is the triad of normal pressure hydrocephalus? Dementia,

of the in

gait apraxia,

vestigator? Experimental study

and urinary incontinence

21.In what type of skew is the tail to the right and the mean greater

12.What category of symptoms of schizophrenia associated with

than the m

muscarinic recepto

edian? Positive skew

rs include affective flattening, social withdrawal, apathy, anhedonia,

22.In what type of error is the null hypothesis rejected when it is true?

poverty o

Type I error (alpha error)

f thought and of content of speech, and lack of interest? Negative

23.If the P-value is less than or equal to .05, what do you do to the

symptom

null hypot

s (type II)

hesis? Reject it

What drug is used to prevent alcohol consumption by blocking

e

aldehyde dehydrogen

What are the pharmacologic effects seen sexually with: Trazodone?

ase? Disulfiram

Priapism

Which drug is used to treat opiate withdrawal, attention deficit

What are the pharmacologic effects seen sexually with: Dopamine

hyperactive dis

agonists?

order (ADHD), and sometimes Tourette's syndrome? Clonidine

Increased erection and libido

Which drug is used to treat the respiratory depression associated with

What are the pharmacologic effects seen sexually with: Neuroleptics?

an overdo

Erectile

se of opioids? Naloxone or naltrexone

dysfunction

Which opioid agonist, more addictive than heroin, is used in the

In a classic gaussian curve, what percentage of the curve is between:

treatment of he

± 3 standard

roin dependence? Methadone

deviations (SDs)? 99.70%

What type of reinforcement strengthens each response and involves

In a classic gaussian curve, what percentage of the curve is between:

fast learning

The mean a

and fast extinction? Continuous reinforcement

nd ± 1 SD? 34%

According to operant conditioning theory, what type of reinforcement

In a classic gaussian curve, what percentage of the curve is between:

is occurrin

± 1 SD and ± 2

g in avoidance behaviors such as phobias and compulsive rituals?

SDs? 13.50%

Negative

In a classic gaussian curve, what percentage of the curve is between:

reinforcement

± 2 SDs and ±

What are the pharmacologic effects seen sexually with: alpha-

3 SDs? 2.40%

Blockers? Impaired

In a classic gaussian curve, what percentage of the curve is between:

ejaculation

± 3 SDs?

What are the pharmacologic effects seen sexually with: Serotonin?

0.15%

Inhibite

1.What is the rate-limiting step of: Glycolysis? PFK-1

d orgasm

What is the rate-limiting step of: Gluconeogenesis? Pyruvate

What are the pharmacologic effects seen sexually with: beta-

carboxylase

Blockers? Impotenc

What is the rate-limiting step of: TCA (Krebs) cycle? Isocitrate

What is the rate-limiting step of: Purine synthesis? Phosphoribosyl

dehydrogenase

pyrophosp

What is the rate-limiting step of: Glycogenesis (glycogen synthesis)?

hate (PRPP) glutamyl amido transferase

Glycogen

What is the only fatty acid that is gluconeogenic? Propionic acid

synthase

Aldose reductase converts galactose to what? Galactitol

What is the rate-limiting step of: Glycogenolysis? Glycogen

2.How many ATPs are generated from glycolysis? 8

phosphorylase

3.In the mitochondria, what complex is needed in order for pyruvate

What is the rate-limiting step of: Hexose monophosphate (HMP)

carboxylase

shunt? Glucose-

to catalyze the reaction from pyruvate to OAA? Biotin, ATP, and CO2

6-phosphate dehydrogenase (G-6-PD)

4.What is the enzyme for the oxidative reaction in glycolysis?

What is the rate-limiting step of: Fatty acid synthesis? Acetyl CoA

Glyceraldehyde d

carbo

ehydrogenase

xylase

5.What three substrates control the enzyme phosphoenolpyruvate

What

is

the

rate-limiting

step

of:

ß-Oxidation?

Carnithine

carboxykinase (PE

acyltransferase I

PCK) for the conversion of OAA to pyruvate in the cytoplasm? 1.

What is the rate-limiting step of: Ketogenolysis? HMG CoA synthase

Cortisol (sti

What is the rate-limiting step of: Cholesterol synthesis? HMG CoA

mulates PEPCK) 2. Glucagon 3. Guanine triphosphate (GTP)

reductas

6.What is released from the reaction of phosphoenolpyruvate

e

carboxykinase (PEPCK

What is the rate-limiting step of: Urea cycle? Carbamoyl phosphate

) for the conversion of OAA to pyruvate? C02

synthetase I

7.What enzyme deficiency causes cataracts, galactosemia, and

What

is

the

rate-limiting

step

of:

Heme

synthesis?

Delta-

galactosuria?

Aminolevulinic aci

Galactokinase deficiency

d (ALA) synthase

8.The addition of D-2,3-bisphosphoglycerate (D-2,3-BPG) to HbA

What is the rate-limiting step of: Pyrimidine synthesis? Aspartate

does what to the

transc

02 saturation curve? Shifts it to the right

arbomylase

9.In what benign condition do you see excretion of large amounts of fructose aft

er ingestion? Essential fructosuria (fructokinase deficiency)

21.Thiamine pyrophosphate (TPP) is associated with what three

10.What is the glycolysis enzyme found only in the liver? Glucokinase

enzymes? 1. a-ket

11.How many ATPs are generated per acetyl CoA? 12 (Not 15-that

oglutarate

would be the ans

Transketolase

wer if you included the pyruvate to acetyl CoA step.)

22.What hormone stimulates glycogen synthesis? Insulin

12.What

enzyme

is

associated

with

the

substrate-level

dehydrogenase

2.

Pynivate

dehydrogenase

3.

23.Deficiency in what enzyme leads to insoluble glycogen formation?

phosphorylation in the TCA

A-1,6 tr

cycle? Succinate thiokinase

ansferase

13.The availability of OAA and acetyl CoA regulates what pathway?

24.The

TCA cycl

(NADPH) generated fro

e

m the G-6-PD reaction is used exclusively for what? Fatty acid

14.What complex of the electron transport chain (ETC) is inhibited by

synthesis

malonate?

25.What enzyme requires selenium (Se) to function? Glutathione

Complex II

peroxidase

15.What drug blocks the FO portion of the adenosine triphosphatase

26.What are the two essential fatty acids? 1. Linoleic acid 2. Linolenic

(ATPase) syst

ac

em of the electron transport chain (ETC)? Oligomycin

id

16.In what two places is glycogen made and stored? 1. Liver 2.

27.What intermediate of the HMP pathway is used to generate

Muscle: Live

nucleotides?

r stores are for blood glucose; muscle stores are for energy reserves.

Ribose-5-phosphate

17.What drug blocks the ETC by attaching itself to K+ for passage

28.A deficiency in what enzyme causes a decrease in oxidoreductase

through the me

activity in n

mbrane, negating the charge gradient? Valinomycin

eutrophils? G-6-PD

18.At what step of the TCA cycle is FADH2 generated? Succinate

29.What are the nonoxidative enzymes of the HMP shunt?

dehydrogenase

Transketolase and transa

(inhibited by malonate)

ldolase

19.What inhibits complex III of the ETC? Antimycin A

30.Are the reactions they catalyze reversible or irreversible?

20.What inhibits the ATP/ADP translocase of the ETC? Atractyloside

Reversible

reduced

nicotinamide

adenine

dinucleotide

phosphate

31.A patient who presents with cardiomegaly and hepatomegaly has

r (in the mitochondria)

what glycogen s

42.What two tissues prefer ketone bodies over glucose? 1. Heart

torage disease? Pompe's disease (lysosomal glucosidase deficiency)

muscle 2. Renal

32.Very low density lipoprotein (VLDL) remnants are known as what?

cortex

Intermed

43.What enzyme is absent in the liver so that ketogenolysis cannot

iate-density lipoproteins (IDLs)

occur?

33.What carries triacylglycerols (TAGS) and cholesterol from the diet?

Thiophorase

Chylomic

44.What pathway utilizes HMG-CoA synthetase in the cytoplasm?

rons

Cholesterol bios

34.What protein is required for the uptake of low-density lipoproteins

ynthesis

(LDLs) in

45.What two vitamins are inactivated when they come in contact with

the peripheral tissue? Apoprotein B-100

acetaldehyde

35.What 3 apoproteins are on the surface of chylomicrons?

? 1. Thiamine 2. Folate

Apoprotein B-48,

46.What is the precursor of all sphingolipids? Ceramide

C-II, and E

47.What two sugars can be used to produce cerebrosides? 1.

36.What protein carries free fatty acids to the liver? Albumin

Glucose 2. Galactose

37.What hormone is activated in adipose tissue when blood glucose

48.Where does the energy for the urea cycle come from? Fat

levels decreas

metabolism

e? Hormone-sensitive lipase

49.What are the two major carriers of nitrogen from tissues? 1.

38.In the P-oxidation pathway, what enzyme generates the FADH2?

Glutamine (mo

Acyl CoA dehydro

st tissues) 2. Alanine (muscle)

genase

50.What are the ketogenic amino acids? Leucine and lysine

39.How many ATPs are generated per acetyl CoA in [3-oxidation? 5

51.What is the storage form of folate? N-methyl folate

40.How many ATPs are generated per acetyl CoA from [3-oxidation if

52.What disease is produced by a deficiency in the enzyme

it is run thr

tyrosinase? Albinism

ough the TCA cycle? 12

(Tyrosine is converted to melanin by the enzyme tyrosinase.)

41.What is the only organ in the body that can produce ketone

53.What two enzymes are blocked by lead? 1. ALA dehydratase 2.

bodies? The live

Ferrochela

tase

66.What enzyme is blocked by allopurinol? Xanthine oxidase ("suicide

54.Where in the body is heme converted to bilirubin? Reticular

inhib

endothelial sy

itor")

stem (RES)

67.What enzyme is stimulated by PTH to produce 1,25 vitamin D3? 1-

55.What type of bilirubin is found in neonatal jaundice? Indirect or

a-Hydroxylase

unco

68.What three organs are used to produce vitamin D? 1. Skin 2. Liver

njugated

3. Kidn

56.What is the primary end product of pyrimidine synthesis? Uridine

ey

monophos

69.What vitamin is an important component of rhodopsin? Vitamin A

phate (UMP)

70.What G protein is stimulated by activated rhodopsin? Gt

57.All amino acids have titration plateaus at what pH values? PH of 2

(transducin), which d

and 9

ecreases cyclic guanosine monophosphate (cGMP) and closes the

58.What amino acid is a good buffer at a pH of 7? Histidine

Na+ channels, caus

59.What is the only way to increase maximum velocity (VmaY)?

ing nerve transmission

Increase enzyme

71.What are the vitamin K-dependent clotting factors? 2, 7, 9, and 10

concentrations

72.What vitamin is connected to selenium (Se) metabolism? Vitamin

60.What happens to affinity if you increase K�? Affinity decreases

E

(they are inv

73.What is the activated form of vitamin E? Alpha-tocopherol

ersely proportional)

74.What elements make up a nucleoside? A base and a sugar

61.What two amino acids disrupt an a-helix? 1. Glycine 2. Proline

75.What is the most common methylated base? Cytosine

62.What amino acid is a phenol? Tyrosine

76.DNA is replicated at what phase of the cell cycle? S phase

63.What enzyme requires molybdenum (Mo) as a cofactor? Xanthine

77.At which end of DNA are new bases added? 3' end

oxidase

75.What keeps single-strand DNA (ssDNA) from re-annealing during

64.What determines the rate of a reaction? The energy of activation

DNA replication

(Ea)

? Single-strand (ss) binding protein

65.What substrate concentration is required to produce 1/2 Vmax?

76.What enzyme is responsible for producing a single-strand (ss) cut

Km

in the DNA to relieve the coil tension? Topoisomerase I (relaxase)

77.What two amino acids are found in high concentrations in the

l-tRNA synthetase

nucleosome?

88.What is needed to direct enzymes to a lysosome? Phosphorylation

1. Arginine 2. Lysine

of manno

78.What three bases are pyrimidines? 1. Cytosine 2. Uracil (only in

se residues

RNA) 3. T

89.What cofactor is needed for lysyl oxidase? Cu2+

hymidine

90.What part of the 50S and 60S ribosomal subunit is needed for

79.What enzyme creates a short sequence of RNA to start DNA

elongation?

replication?

Peptidyl transferase

Primase

91.In the lac operon: At which site is the repressor gene encoded? I

80.What type of enzyme is reverse transcriptase? RNA-dependent

gene

DNA polyme

92.To which site does the repressor protein bind in order to inhibit

rise

transcripti

81.What is the direction of transcription? 5' to 3' direction

on? Operator

82.Where is the TATA box in located eukaryotes? 25 bases

93.What amplification technique is used to generate a larger amount

downstream (-25)(promot

of DNA?

er)

Polymerase chain reaction (PCR)

83.What causes transcription to stop in eukaryotes? The poly(A) site

94.What test is used to determine whether a gene is expressed?

on the

Northern blottin

DNA

g

84.What protein binds to the promoter region in eukaryotes to initiate

95.At what organ in the body is urea produced? Liver

transcrip

96.What regulatory proteins work during fetal development to ensure

tion? TF II D (transcription factor)

that cells b

85.What part of the 30S ribosome binds to the Shine-Dalgarno

ecome a specific cell type (If there is a defect here, there can be

sequence? 16S subu

profound str

nit

uctural mutations.)? Homeobox genes

86.What is the start codon for translation? AUG

97.What is the mode of inheritance in which a trait is seen in every

87.What is the enzyme that activates the amino acids for the tRNA?

generation

Aminoacy

and is passed on only by females? Mitochondrial inheritance

98.What is the name for the process of going from mRNA to proteins?

110.What amino acid is broken down into N20, causing an increase in

Translat

eGMP of smoo

ion

th muscle resulting in vasodilatation? Arginine

99.What are the components of a nucleotide? A base, a sugar, and a

111.What hormone phosphorylates enzymes to decrease their

phosphate

activity? Glucagon

100.What enzymes hydrolyze 3'-5' phosphodiesterase bonds from the

112.Lack of what enzyme can lead to Wernicke-Korsakoff syndrome

outside of the

through lack of

strand in? Exonucleases

activity in the HMP shunt? Thiamine pyrophospate (TPP)

101.What type of organisms have monocistronic mRNA? Eukaryotes

113.What enzyme found in the liver catalyzes glycerol to glycerol-3-

102.In collagen, every third amino acid is this amino acid. Glycine

phosphate?

103.What form of continuous DNA, used in cloning, has no introns or

Glycerol kinase

regulatory e

114.Which shuttle is used to bring fatty acyl CoA from the cytoplasm

lements? c-DNA, when it is made from mRNA

for ketogen

104.What proteins stimulate a cell to enter the S phase? Growth

esis? Carnitine acyl CoA transferase II

factors

115.Which

105.At what pH is there no net charge on the structure? pI (isoelectric

Phenylalanine hydroxylas

point)

e

106.What complex of the ETC contains Cue+? Complex 4

116.After approximately how many days of a prolonged fast does

107.What two shuttles are needed to keep NAD+ in the reduced

death occur in hu

state? Malate/a

mans? 60 days

spartate and glycerol-3-phosphate shuttles

What is the cause of death? The breakdown of the essential proteins

108.What vitamin is required for y-carboxylation of many Cat+-binding

of the h

proteins?

eart and brain

Vitamin K

117.All the carbons in a fatty acid are derived from what source?

109.From where is the energy for gluconeogenesis derived? ß-

Cytoplas

Oxidation of fat

mic acetyl CoA that left the mitochondria as citrate

ty acids

118.What enzyme is deficient in alcaptonuria? Homogentisic acid

enzyme

is

deficient

in

phenylketonuria

(PKU)?

119.In a diabetic patient, glucose is converted by aldose reductase to

127.What four substances increase the rate of gluconeogenesis? 1.

what?

Glucagon 2. N

Sorbitol (resulting in cataracts)

ADH 3. Acetyl CoA 4. ATP

120.What glycolytic intermediate can be used to synthesize

128.What enzyme is deficient in a patient who presents with:Q A.Liver

triglycerides and pho

damage and

spholipids? DHAP

severe hypoglycemia? Aldolase B (hereditary fructose intolerance)

121.What glycolytic enzyme has a high Vmax high Km and low affinity

QB..Jaundice,

for glucose?

galactosuria? Galactos

Glucokinase

e 1-phosphate uridyltransferase

122.What is the main inhibitor of pyrnvate dehydrogenase? Acetyl

129.What

CoA (pyru

CaL+:calmodulin ratio

vate to acetyl CoA)

2. Epinephrine 3. Glucagon

123.What are the two substrate-level phosphorylations in glycolysis?

130.What are the two inhibitors of complex I of the ETC? 1. Rotenone

1. Pyruv

2. A

ate kinase 2. Phosphoglycerate kinase

mytai (barbiturates)

124.What are the eight liver-specific enzymes? 1. Fructokinase 2.

131.What

Glucokinase 3

dehydrogenase complex

. Glycerol kinase 4. PEPCK 5. Pyruvate carboxylase 6. Galactokinase

? 1. TPP 2. Lipoic acid 3. CoASH 4. FAD 5. NAD

7. Fructose-

132.What attaches to protons and allows them to enter into the

1,6-biphosphate 8. Glucose-6-phosphate

mitochondria with

125.In what cycle does glucose go to the muscle, where it is

out going through the ATP-generating system? 24-Dinitrophenol

converted to pyruva

133.What are the two decarboxylation steps of the TCA cycle? 1.

te and then into alanine before being taken back to the liver? Alanine

Isocitrate de

cycle

hydrogenase 2. a-Ketoglutarate dehydrogenase

126.In what cycle does glucose go to the muscle, where it is

134.What are the three inhibitors of complex IV of the ETC? 1.

converted to lactat

Cyanide 2. CO

e, and then returned to the liver? Cori cycle

3. Azide

vomiting,

three

are

the

lethargy,

substances

five

factors

cat,

stimulate

that

galactosemia,

glycogenolysis?

constitute

the

and

1.

pyruvate

135.What three steps of the TCA cycle generate NADH? 1. Malate

Hers disease

dehydrogenase

143.What causes the lysis of red blood cells by oxidizing agents in a

2. Isocitrate dehydrogenase 3. a-Ketoglutarate dehydrogenase

glucose-6-

136.What disease presents with weakness and cramps on exercise

phosphate dehydrogenase deficiency? The lack of glutathione

without an increa

peroxidase activ

se in blood lactate levels? McArdle's disease (muscle glycogen

ity results in a decrease in NADPH production, leaving glutathione in

phosphorylase

the reduce

deficiency)

d state.

137.NADPH generated from the HMP shunt is used for what? Fatty

144.What disease presents with hepatomegaly and a normal EKG?

acid synth

Glycogen storage

esis, nucleotide synthesis, and glutathione reductase

disease, type III (Forbes disease, Cori disease)

138.Is linolenic acid an omega-3 or -6 fatty acid? Omega-3; linoleic is

145.What apoprotein is on the surface of LDL? Apoprotein B-100

ome

146.What carries cholesterol from the tissues back to the liver? HDLs

ga-6

147.What apoprotein mediates the uptake of remnants by the liver?

139.Is the oxidative reaction of the HMP shunt reversible or

Apoprote

irreversible?

in E

Irreversible (G-6-PD and 6-phosphogluconate dehydrogenase)

148.What is the complex needed for acetyl CoA carboxylase? Biotin,

140.What disease presents with an enlarged liver and kidneys,

ATP, and

dwarfism, hypoglyc

G02 (acetyl CoA to malonyl CoA)

emia, acidosis, and hyperlipidemia? Von Gierke's disease (glucose 6-

149.are the three tissues where TAGS are produced? 1. Liver 2.

phosphat

Muscle 3. Ad

ase deficiency)

ipose tissue

141.At what three sites can the HMP shunt enter into glycolysis? 1.

150.What delivers cholesterol to the tissues? LDLs

Fruct

151.What apoprotein is produced by the intestinal epithelium?

ose-6-phosphate

2.

Glucose-6-phosphate

3.

Glyceraldehyde-3-

Apoprotein B-48

phosphate 1

152.What carries TAGS to the peripheral tissues? VLDLs

142.Deficiency in the liver glycogen phosphorylase enzyme is known

153.How many NADPHs are used per addition of each of acetyl CoA

as what?

into a fatty aci

d chain? 2 NADPHs/acetyl CoA

164.What hormone hydrolyzes TAGS to free fatty acids and glycerol?

154.What apoprotein activates lipoprotein lipase in the capillary

Hormonesensitive

epithelium to

lipase

hydrolyze TAGS? Apoprotein C-II

165.What enzyme is deficient in a patient 2 years of age or younger

155.What apoproteins are on the surface of IDL? Apoproteins B-100

who presents

and E

with vomiting, lethargy, coma, hypoketosis, and hypoglycemia

156.From which two substances are phospholipids made? 1. DAGs 2.

following a fast o

Phosphatidic

f more than 12 hours? Medium-chain acyl dehydrogenase

acid

166.What form of alcohol did the patient drink if he became blind as a

157.What apoproteins are on the surface of HDL? Apoprotein A-I, C-

result?

II, and E

Methanol (wood alcohol)

158.What is needed to produce a double bond in a fatty acid chain in

167.What regulates the rate of ketone body formation? The rate of ß-

the endopla

oxidation

smic reticulum? NADPH, O2, and cytochrome b5

168.What intermediate enables propionyl CoA to enter into the TCA

159.What apoprotein activates lecithin cholesterol acyl transferase to

cycle?

esterify

Succinyl CoA

cholesterol from tissue? Apoprotein A-I

169.What sphingolipid is formed by the union of serine and palmitoyl

160.What apoproteins are on the surface of VLDL? Apoproteins B-

CoA?

100, C-II,

Sphingosine

and E

170.What intermediate of cholesterol synthesis anchors proteins in

161.At the end of each round of ß-oxidation, what is released? Acetyl

the membranes

CoA, FADH

and forms CoQ? Farnesyl pyrophosphate (FPP)

, and NADH

171.What is the complex needed for propionyl CoA carboxylase?

162.What two enzymes are vitamin B12 dependent? 1. Homocysteine

Biotin, ATP, and

methyl transfera

C02

se 2. Methylmalonyl CoA transferase

172.What are the three ketone bodies? 1. Acetoacetate 2. Acetone 3.

163.What enzyme is blocked by disulfiram? Aldehyde dehydrogenase

ß-hydroxybut yrate

173.What type of damage to the kidneys is caused by drinking

sverase (OCT) deficiency

ethylene glycol (an

181.What amino acid is a precursor of the following substances:

tifreeze)? Nephrotoxic oxylate stones

Serotonin?

174.What is the only sphingolipid that contains choline and p04?

Tryptophan

Sphingom

What amino acid is a precursor of the following substances: GABA?

yelin (lecithin also, but it is not a sphingolipid)

Glutamat

175.What is the order of fuel utilization in a prolonged fast? 1.

e

Glucose from

What amino acid is a precursor of the following substances:

liver glycogen 2. Glucose from gluconeogenesis 3. Body protein 4.

Histamine? Histidin

Body fat

e

176.Sialic acid and amino sugars are needed to produce what

What amino acid is a precursor of the following substances: Creatine?

sphingolipid?

Glycine/

Ganglioside

arginine

177.What vitamin is needed as a cofactor for decarboxylation and

What amino acid is a precursor of the following substances: NAD?

transaminase re

Tryptoph

actions? Vitamin B6

an

178.What are the two ways that nitrogen can enter into the urea

What amino acid is a precursor of the following substances: N20?

cycle? 1. Aspar

Arginine

tate 2. Carbomoyl PO4

182.What enzyme deficiency will result in an increase in blood

179.What is the only enzyme in the body that uses N5 methyl folate?

ammonia, but no i

Homocyst

ncrease in uracil concentrations? Carbamoyl-phosphate synthetase

eine methyl transferase

183.What

180.What enzyme deficiency will result in an increase in blood

Phenylalanine, Tyrosine,

ammonia and an in

Tryptophan, Isoleucine, Threonine

crease in uracil and orotate concentrations in both the blood and

184.What are the three diseases of sphingolipids? 1. Niemann-Pick

urine?

disease

Ornithine transcarbamoylase (OTC) deficiency-also called ornithine

2. Gaucher's disease 3. Tay-Sachs disease

carbamoyltran

are

the

glucogenic

and

ketogenic

amino

acids?

185.What type of jaundice is seen in Rotor's syndrome? Conjugated

ct) hyperbilirubinemia (It is a transport defect.)

(direct) hype

198.What form of bilirubin is carried on albumin? Unconjugated

rbilirubinemia

(indirect)

186.What is the pyrimidine intermediate that joins PRPP? Orotic acid

199.In what disease is there a genetic absence of UDPglucuronate

187What three amino acids are used to synthesize the purine ring? 1.

transferase, re

Glyci

sulting in an increase in free unconjugated bilirubin? Crigler-Najjar

ne 2. Aspartate 3. Glutamine

syndrome

188.What enzyme is blocked by hydroxyurea? Ribonucleotide

200.What enzyme is deficient in acute intermittent porphyria?

reductase

Uroporphyrinogen

189.What is the primary end product of purine synthesis? IMP

I synthetase

190.What

201.What enzyme is deficient in congenital erythropoietic porphyria?

enzyme

is

deficient

in

hereditary

protoporphyria?

Ferrochelatase

Uroporph

191.What are the two precursors of heme? 1. Glycine 2. Succinyl-CoA

yrinogen III cosynthase

192.What enzyme is blocked by 5-FU? Thymidylate synthetase

201.What amino acid has a pKa of 13? Arginine

193.What disease has a genetically low level of UDPglucuronate

203.What two amino acids have a pKa of 10? 1. Lysine 2. Tyrosine

transferase, resu

204.What two amino acids have a pKa of 4? 1. Aspartic acid 2.

lting in elevated free unconjugated bilirubin? Gilbert's syndrome

Glutamic aci

194.What form of bilirubin can cross the blood-brain barrier?

d

Unconjugated fre

205.What disease has a genetic deficiency in adenosine deaminase?

e bilirubin

Severe c

195.What substrates are needed to produce carbamoyl P04 (de novo

ombined immunodeficiency (SCID)

pyrimidine synt

206.What type of charge does the molecule have if the pH is greater

hesis)? Glutamine, CO2, and ATP via carbamoyl PO4 synthetase II

than the pI

196.What

(isoelectric point)? A net negative charge

enzyme

is

blocked

by

methotrexate/trimethoprim?

Dihydrofolate re

207.In what disease is there a deficiency in hypoxanthine guanine

ductase

phosphoribosyl

197.What type of jaundice is seen in Dubin-Johnson syndrome?

transferase (HGPRT)? Lesch-Nyhan syndrome

Conjugated (dire

208.What is the end product of purine catabolism? Uric acid

209.What enzyme is deficient in selective T cell immunodeficiency?

Vmax; Km is same)

Purine n

218.What is the maximum rate possible with a given amount of

ucleoside phosphorylase

enzyme? Vmax

210.What substrate builds up in Tay-Sachs disease? Ganglioside

219.To what are intracellular glucose levels inversely related? cAMP

GM2

levels

211.On a Lineweaver-Burke plot, what type of binding has both plots

220.Does a saturated fatty acid have double bonds? No; unsaturated

crossing the

fatty ac

y-axis in the same spot? Competitive, reversible inhibition (Vmax is

ids have double bonds

the

221.What two factors cause PTH to be secreted? 1. A decrease in

same, increase Km)

blood Ca2+ 2. A

212.What toxin ADP-ribosylates via GS protein to increase CAMP?

decrease in PO4 concentrations

Cholera toxin

222.What in the human genome differs in each individual that can

213.What vitamin derivatives are used for growth and differentiation of

serve as an ide

epitheli

ntification marker? RFLP-restriction fragment length polymorphism

um for reproductive and embryonic development? Vitamin A

223.What test is used to identify HIV-positive patients? ELISA-

214.Light causes isomerization of what in the eyes? 11-cis-retinal to

enzyme-lin

trapsretinal

ked immunosorbent assay

(activated rhodopsin)

224.What toxin ADP-ribosylates via G to increase cAMP? Pertussis

215.What are the two actions of calcitonin? 1. Increases Ca2+

toxin

excretion from

225.What vitamin is essential for normal Ca2+ and P04 metabolism?

the kidney 2. Increases bone mineralization

Vitamin

216.What causes an increase in bone mineralization and Cat+, as

D

well as P04 abso

226.What vitamin is deficient in a person who has impaired taste,

rption from the GI tract and kidney tubules? Vitamin D

night blindnes

217.On the Lineweaver-Burke plot, what type of binding has both

s, and increased risk for having an abortion? Vitamin A

plots crossing t

227.What bond does an endonuclease cleave? 3', 5' internal

he x-axis in the same spot? Noncompetitive, reversible binding

phosphodiesteras

(decrease in

e bond

228.What protein separates base pairs and unwinds the DNA at the

238.What eukaryotic DNA polymerase is used for: DNA repair? ß-

replication for

Polymerase

k? Helicase (It is an ATP-dependent enzyme.)

239.What is the orientation of the gene sequence strand? 5' to 3'

229.What vitamin deficiency would cause liver necrosis and red blood

(same a

cell fragil

s RNA)

ity? Vitamin E deficiency

240.What is the location of the TATA box in procaryotes? 10 bases

230.What protein catalyzes the formation of the last phosphodiester

downstr

bond (PDE) b

eam

etween the Okazaki fragments to produce a continuous strand? DNA

241.What binds to the promoter region in procaryotes? Sigma factor

ligase

242.What enzyme has a 5' to 3' synthesis of the Okazaki fragments, 3'

231.What is the hypochromic effect? Increased absorption as DNA

exonucleas

goes from do

e activity, and 5' exonuclease activity? DNA polymerase I

uble stranded to single stranded

243.What type of mutation has the same amino acid coded for, but

232.What technique uses DNA for analysis? Southern blotting

with a differen

233.In what direction is a new DNA strand made? 5' to 3'

t codonsequence? Silent mutation

234.What enzyme has a 5' to 3' synthesis activity and a 3'

244.At what position of the anticodon does the "wobble" hypothesis

exonuclease activity?

occur?

DNA polymerase III

Position 1 of the anticodon (the 5' end)- which is the same as position

235.What enzyme makes a double-stranded cut through DNA, needs

3 of the

ATP, and introduc

codon (the 3' end)

es negative supercoiling? Topoisomerase II

245.What are the three "stop" codons? 1. UAA 2. UAG 3. UGA

236.What eukaryotic DNA polymerase is used for: DNA replication? a-

246.What stops transcription in procaryotes? Rho factor or a hairpin

and d

loop

elta-polymerase

247.What are the three post-transcriptional modifications? 1. 7-methyl

237.What eukaryotic DNA polymerase is used for: Replication in

guan

mitochondria?

ine cap on the 5' end 2. Addition of the poly(A) tail to the 3' end 3.

gamma-Polymerase

Removal o f introns

248.In what type of mutation is a different codon added, resulting in

258.What translation factor is blocked by erythromycin? Elongation

formation

factore G (EF

of a different amino acid? Missense mutation

-G)

249.What amino acid is attached to the 3' end of the tRNA in

259.What is the charge of the molecule if the pH is less than the pI

eukaryotes?

(isoelectri

Methionine

c point)? A net positive charge

250.What enzyme makes tRNA and the SsRNA? RNA polymerase III

260.What technique uses the separation of proteins on a gel

251.What structure of a protein describes the interaction among

electrophoresis?

subunits?

Western blot

Quaternary structure

261.Lack of what vitamin causes multiple carboxylase deficiency?

252.What two amino acids require vitamin C for hydroxylation? 1.

Biotin

Proline 2. Ly

262.What enzyme makes hnRNA/mRNA? RNA polymerase II

sine

263.What enzyme is deficient in cystathioninuria? Cystathionase

253.What is determined by the secondary structure of an amino acid?

264.What enzyme is deficient in maple syrup urine disease?

The fold

Branched-chain k

ing of an amino acid chain

eto acid dehydrogenase

254.Which mutation has a stop codon put in place of the previous

265.What type of mutation has the addition or deletion of a base?

codon? Nonsense

Frameshi

mutation

ft

255.What amino acid is attached to the 3' end of the tRNA in

266.What is the site of action of puromycin? Aminoacyl tRNA (A site)

prokaryotes?

267.What translational factor is blocked by both diphtheria and

f-Methionine

Pseudomonas toxi

256.What enzyme makes rRNA (barring the 5s subunit)? RNA

ns? Elongation factor 2 (EF-2)

polymerase I

268.What substrate gets built up in Gaucher's disease? Glucosyl

257.What is the site of action of cycloheximide? Peptidyl transferase

cerebroside

(60

269.What enzyme is deficient in homocystinuria? Homocysteine

S)

methyl transferase or cystathionine synthetase

270.What substrate is built up in Niemann-Pick disease? Tyrosine

281.Is the hydroxyl (-OH) end of DNA and RNA at the 3' or the 5'

kinase

end? 3' end.

271.What is the second messenger system used by growth factors?

Phosphate (P04) is at the 5' end.

Sphingomyelin

282.What is the only factor of enzyme kinetics that the enzyme

272.What is the name of the sequence on mRNA that precedes the

affects? Ea (acti

start codon in pr

vation energy)

okaryotes? Shine-Dalgarno sequence

283.What tumor suppressor gene prevents a cell from entering S

273.What amino acid undergoes N-glycosylation? Asparagine

phase when no gro

274.What translational factor is blocked by tetracycline? Elongation

wth factors are present? Rb gene

facto

284.What tumor suppressor gene prevents a cell with damaged DNA

r Tu (EF-Tu)

from entering th

275.What translational factor is blocked by streptomycin? IF-2

e S phase? p53 gene

276.Which organisms have polycistronic mRNA? Prokaryotes:

285.What factors are needed for elongation in prokaryotes? EF-Tu or

Polycistonic and pr

EF-is a

okaryotes both start with P.

nd GTP

277.In what disease are lysosomal enzymes released into the

286.What factors are needed for translocation in eukaryotes? EF-2

extracellular space

and GTP

where an accumulation of inclusion bodies compromises the cell's

287.What type of inheritance involves carriers, affects only males, and

function?

skips ge

I-cell disease

nerations? X-linked recessive

278.What is the site of action of chloramphenicol? Peptidyl

288.What

transferase (50

electrophoresis? Northern

s)

blotting

279.What is needed to initiate translation? IF and GTP (OF for

289.What factors are needed for elongation in eukaryotes? EF-1 and

eukaryotes)

GTP

280.What is the name of the process of going from DNA to mRNA?

290.What factors are needed for translation in prokaryotes? EF-G and

Transcription

GTP

type

of

separation

technique

uses

RNA

on

an

291.What type of inheritance has no male-to-male transmission, and

13.What viruses are associated with Cowdry type A intranuclear

every daughte

inclusions?

r is affected from the father in every generation? X-linked recessive

Herpes virus I and II

1.What virus is associated with intranuclear inclusions known as

14.What virus is associated with the Norwalk agent? Calicivirns

"owl's eye" inc

15.What virus affects the motor neurons in the anterior horn?

lusions? Cytomegalovirus (CMV)

Poliovirus

2.What virus is associated with Negri bodies? Rabies virus

16.What is the most common cause of diarrhea in children? rotavirus

3.What virus is associated with Guarnieri bodies? Variola virus -7-ox

17.What virus lies dormant in the: Trigeminal ganglia? Herpes I

vtrc

18.What virus lies dormant in the: Dorsal root ganglia? Varicella

s

19.What virus lies dormant in the: Sensory ganglia of S2 and S3?

4.What virus causes small, pink, benign wart-like tumors and is

Herpes I

associated with

I

HIV-positive patients? Molluscum contagiosum

20.With what virus are Downey type II cells associated? EBV

5.What viruses are associated with cervical carcinoma? Human

21.What are the four segmented RNA viruses? 1. Bunyavirns 2.

papilloma viruses

Orthomyxovirus

(HPVs) 16 and 18

3. Reovirns 4. Arenavirus (BORA)

6.What virus is associated with erythema infectiosum or fifth disease?

22.Koilocytic cells on a Papanicolaou (Pap) smear are indicative of

Parvovir

what virus?

us B-19

HPV

7.What virus binds to: CD4? Human immunodeficiency virus (HIV)

23.What bacteria constitute the most common cause of nosocomial

8.What virus binds to: ß2-Microglobulin? Cytomegalovirus (CMV)

infections in bu

9.What virus binds to: Complement factor C3? Epstein-Barr virus

rn patients and in patients with cystic fibrosis? Pseudomonas

(EBV) - Qp7_1

24.What organism is associated with pneumonia acquired from air

10.What virus binds to: ACh receptors? Rabies virus

conditioners?

11.What is the only dsRNA virus? Reovirus

Legionella

12.What are the three non-enveloped RNA viruses? 1. Picornavirns 2.

25.What species of bacteria is associated with whooping cough?

Calic

Bordetella pertu

ivirns 3. Reovirns (PCR)

s,sis

26.What two bacteria are associated with drinking unpasteurized

37.What organism is associated with a rigid belly and rose spots on

milk? 1. Bruce

the belly?

lla 2. Listeria (has tumbling motility)

Salnwnella typhi

27.What

organism

is

associated

with

gastritis

and

ulcers?

38.What organism will cause an infection if undercooked or raw

Helicobacter pyl

seafood is eaten?

ori

Vibrio parahaemolyticus

28.What species of bacteria is associated with traveler's diarrhea?

39.What organism is likely to infect you if you get cut by a shell at the

Escheric

beach?

hia coli (enterotoxic)

Vibrio vulnificus

29.With what organism is "currant jelly" sputum associated? Klebsiella

40.What infective bacteria are found in undercooked hamburgers?

pneum

Escherichia coli

oniae

strain 0157:H7ss

30.What is the most common cause of enterocolitis? Salmonella

41.What organism is said to have a "spaghetti and meatball"

enteritidis

arrangement under a

31.What organism is so infective that it takes only 1 to 10 organisms

microscope? Malassezia furfur

to cause a

42.What fungus is associated with rose gardener's disease?

n infection? Shigella

Sporothrix schen

32.What organism stains bipolar and causes buboes? Yersinia pesos

ckii

33.Rice water stools are indicative of what organism? Vibrio cholerae

43.What

34.With what organism are intracellular gram-negative inclusions in

Chromomycosis

neutrophils

44.What fungus is found in soil with bird or bat feces? Histoplasma

associated? Neisseria gonorrhoeae

cap.sulatum

35.What organism is most likely to cause an infection if you are bitten

45.Which organism causes San Joaquin fever? Coccidioides immitis

by a dog

46.What fungus causes endocarditis in IV drug users? Candida

or a cat? Pasteurella

albicans

36.What organism needs factor X and NAD in order to grow on growth

47.What fungus is found in pigeon droppings? Cryptococcus

medium?

neoformans

Haemophilu.s influenzae type B

fungus

is

seen

as

colored

cauliflower

lesions?

48.What fungus is seen as a yeast with broad-based buds and a

59.What fungus is a facultative intracellular parasite of the reticular

double refractile

endothel

cell wall? Blastomyces dermatitidis

ial system? Histoplasma capsulatum

49.What fungus is stained positive with India ink? Cryptococcus

60.What virus is helical and has HN and F glycoprotein spikes?

neoforrnans

Paramyxovirus

50.What virus is the most common causative agent of the common

61.What is the most common cause of pneumonia in persons with

cold in the summe

underlying health

r and the fall? Rhiuovirus

problems? Klebsiella pneumoniae

51.To what family of viruses do dengue, St. Louis, and yellow fever

62.What is the most common cause of pneumonia in young children?

belong?

Mycoplas

Flavivirus

ma

52.What is the name of the bullet-shaped virus? Rhabdovirus

63.What virus causes epidemic keratoconjunetivitis? Adenovirus

53.What virus is responsible for causing the croup and also the

64.What is the most common cold virus? Rhinovirus

common cold in t

65.What two viruses have neuraminidase activity? 1. Influenza 2.

he young and the old? Paraiuflueuza virus

Mumps

54.What is the causative agent of orchitis, parotids, and pancreatitis?

66.What is the most common cause of diarrhea in infants? Rotavirus

Mumps vi

67.What is the reservoir for the togavirus? Birds

rus

68.What two viruses cause pancreatitis? 1. Mumps 2. Coxsackie B40

55.What virus causes hoof-and-mouth disease? Vesicular stomatitis

69.With what two viruses are Reye's syndrome associated? 1.

virus

Varicella vir

56.What is the most common cause of pneumonia in children 1 year

us 2. Influenza virus

old or younger?

70.What is the most common cause of meningitis in: Children younger

Respiratory syncytial virus (RSV)

than 3 month

57.What virus is the most common cause of the common cold in the

s of age? Streptococcus agalactiae or Escherichia Coli

winter and earl

71.What is the most common cause of meningitis in: Non-immunized

y spring? Coronavinzs

children 12 mon

58.What yeast is urease positive? Cryptococcus neoformans

ths to 6 years old? Haemophilus influenzae type b

72.What is the most common cause of meningitis in: Immunized

83.Which two organisms can cause sulfur granules in the pus? 1.

children 12 months

Actinomyces 2

to 6 years old? Streptococcus pneumoniae

. Nocardia

73.What is the most common cause of meningitis in: Military recruits?

84.What species of bacteria is responsible for causing endocarditis in

Neisseri

IV drug u

a meningitidis

sers? Staphylococcus epidermidis

74.What

is

the

most

common

cause

of

meningitis

in:

85.What bacteria are responsible for food poisoning from rice, fried

HIV+/immunocompromised person

rice, and r

s? Cryptococcus neoformans

eheated foods? Bacillus cereus

75.What is the most common cause of meningitis in: Adults?

86.Which bacteria present as a common cause of meningitis in renal

Streptococcus pn

transplant pa

eunwniae

tients? Listeria

76.What is the most common cause of bronchiolitis in children? RSV

87.What bacteria get inoculated into the body by a puncture wound in

77.What is the most common cause of urinary tract infections?

the skin an

Escherichia coli

d also inhibit glycine and GABA? Clostridium tetani

78.Which

three

organisms

cause

heterophilic

negative

88.What bacteria are found in poorly preserved canned food and

mononucleosis? 1. CMV 2

cause flaccid par

. Toxoplasma gondii 3. Listeria

alysis? Clostridium botulinum

79.What two genera are spore formers? 1. Clostridia 2. Bacillus

89.What bacteria cause myonecrosis? Clostridium perfringens

80.What bacteria are responsi- ble for woolsorters' disease? Bacillus

90.What bacteria cause pseudomembranous colitis? Clostridium

anthrac

difficile

is

91.What bacteria are associated with food poisoning from ham, potato

81.What is the most common cause of bacterial pneumonia?

salad, and

Streptococcus pn

custards? Staphylococcus aureus

eumoniae

92.What three bacteria are quellung reactive test positive? 1.

82.What bacteria cause subacute endocarditis and dental caries?

Neisseria men

Streptococcus vi

ingitidis 2. Haemophilus influenzae 3. Streptococcus pneumoniae

ridans

93.Which organism causes a painful chancre? Haemophilus ducreyi

94.What is the most common cause of viral pneumonia? RSV

108.Which organism releases endotoxins PRIOR to cell death?

95.What is the predominant anaerobe in the colon? Bacillus fragilis

Neisseria mening

96.Which organism causes trench mouth? Fusobacterium

itidis

97.Which organism causes Lyme disease? Borrelia burgdorferi

109.What is the only ssDNA virus? Parvovirus

98.Which organism causes Weil's disease? Leptospira

110.What is the only DNA virus to replicate in the cytoplasm?

99.What organism causes Q fever? Coxiella burnetii

Poxvirus

100.agent causes pneumonia in college students and military

111.What are the three naked DNA viruses? 1. Parvovirus 2.

recruits? Mycoplas

Adenovirus 3. P

ma pneumoniae

apovavirus (PAP)

101.What is the tetrad of Jarisch-Herxheimer reaction? Rigors,

112.What is the only DNA virus that has the reverse transcriptase

leukopenia, decr

enzyme?

ease in blood pressure, and increase in temperature

Hepadnavirus

102.Which spirochete causes Rocky Mountain spotted fever?

113.Which hepatitis virus is an RNA viroid-like virus that needs

Rickettsia ricke

hepatitis B to

ttsii (wrist to trunk rash)

be infective? Hepatitis D

103.Which organism causes trench fever? Rochalimaea quintana

114.Which hepatitis virus is an enveloped RNA flavivirus, which is

104.Which organism causes epidemic typhus? Rickettsia prowazekii

known for pos

(trunk to

tinfusional hepatitis? Hepatitis C

periphery rash)

115.What antigen is needed to diagnose an infectious patient with

105.Which organism causes pneumonia in bird owners? Chlamydia

hepatitis B?

psittaci

Hepatitis Be antigen

106.Which organism causes multiple infections by antigen switching?

116.Which type of hepatitis can cause hepatocellular carcinoma?

Borrelia

Hepatitis B 4- C

recurrentis

117.Which type of hepatitis is a calicivirus? Hepatitis E (enteric)

107.Which organism has protein A for an anti-opsonization defense?

118.Which type of hepatitis is a picornavirus? Hepatitis A (infectious)

Staphylo

119.What two antigens need to be positive for a patient to have

coccus aureus

chronic active h epatitis? 1. Hepatitis Bs 2. Hepatitis A antigen

120.In the window phase of a hepatitis B infection, which antibodies

and with HIV-positive patients with CD4 counts of 200 or lower?

do you see?

Pneumocy

Hepatitis Be and c antibodies; You see the antibodies c and e.

sti.s carinii

121.Which virus is found in the urine of rodents? Arenavirus

133.What three organs can be affected by Trypanoaoma cruzi? 1.

122.Which virus is associated with hairy T cell leukemia? HTLV-I and

Heart 2. Esop

HTLVII

hagus 3. Colon Remember-you get "megas:" 1. Cardiomegaly 2.

123.What are the components of the rubella triad? 1. Patent ductus

Megaesophagus 3. Meg

arterio

acolon

sus (PDA) 2. Cataracts 3. Mental retardation

134.What organism causes kala-azar, which is associated with

124.What is the drug of choice to treat: HSV encephalitis? Vidarabine

hyperpigmentation o

125.What is the drug of choice to treat: RSV pneumonia? Ribavirin

f the skin, enlargement of the spleen, and decreased bone marrow

126.What is the drug of choice to treat: CMV retinitis/infection?

activity?

Ganciclo

Leishmania donovani

vir

135.Which type of malaria is associated with dark urine? Plasmodium

127.What is the drug of choice to treat: Influenza A? Amantadine

falci

128.What is the drug of choice to treat: HSV? Acyclovir

parum (malignant)

129.What is the drug of choice to treat: HPV? Interferon alpha

136.What people are "protected" from malaria? People with

130.What does Candida albicana do that distinguishes it from other

heterogenous sickle

fungi?

cell trait

It forms a germinal tube at 37°C.

137.What type of Plasmodium is banana or crescent shaped when

131.Which organism, transmitted by sexual contact, is almost

stained with Giems

diagnostic by the f

a stain? Plasmodium falciparum

oul-smelling, green discharge from the vagina and its associated itch?

138.What is the only Plasmodium that is quartan? Plasmodium

Trichomo

malarize; the

nas vaginalis

others are tertian.

132.Which organism is associated with a diffuse bilateral interstitial

139.What types of Plasmodium produce latent hypnozoites in the

pneumonia

liver, which can cause a relapse? Plasmodium vivax and Plasmodium ovale

140.What organism is associated with liver abscess, t ulcers, and

ossibly subacute sclerosing panencephalitis? Rubeola (measles)

perforated dia

152.What are known as jumping genes? Transposons

phragms? Entamoeba histolytica

153.What is the most common viral cause of myocarditis? Coxsackie

141.What

type of

Plasmodium

affects:

Only mature RBCs?

Plasmodium malariae 142.What

type

of

B 154.What virus is associated with heterophil-positive mononucleosis?

Plasmodium

affects:

Only

redculocytes?

EBV

Plasmodium vivax

155.What is the only virus to be eradicated? Smallpox virus

143.What type of Plasmodium affects: RBCs of all ages? Plasmodium

156.What virus, which creates painful vesicular lesions, is a cause of

falciparum

aseptic m

144.What three carcinomas are associated with EBV? 1. Burkitt's

eningitis? Herpes simplex II

lymphoma 2.

157.What virus attaches to fibroblastic growth factor? Herpes simplex

Nasopharvngeal 3. Thymic

I

145.What is the direction of the strand if a virus has infectious +RNA?

158.What is the most prevalent coral infection in the USA? Varicella-

5' to 3'

zoster

RNA

virus

146.What two viruses do not get their envelope from budding but

159.What is the only herpes virus to cross the placenta? CMV

actually from co

160.Of what virus are Guarnieri bodies diagnostic? Smallpox

ding? HIV and poxvirus

161.What cells are atypical on a peripheral blood smear in a

147.What glycoprotein in the HIV virus is used for fusion? GP41

heterophil-positive

148.What glycoprotein in the HIV virus attaches to CD4? GP120

mononucleosis? T cells not the B cells

149.What protein of the HIV virus is used to detect if a patient is HIV-

162.What type of hepatitis has the highest mortality rate among

positive

pregnant women?

by ELISA? P24

Hepatitis E

150.What two viruses cause progressive multifocal encephalitis? 1.

163.What does hepatitis D virus need from hepatitis B virus to be

Simian (SV40)

infective?

virus 2. JC virus

Hepatitis Bs antigen as its envelope

151.With what virus do you see Koplik's spots and Worthin- Finkeldy

164.What are the two hepatitis viruses that can be chronic and can

cells, and p

lead eventual

ly to hepatocellular carcinoma? 1. Hepatitis B 2. Hepatitis C

ent with hepatitis? Hepatitis Be, e, s

165.What are the only two viruses where naked dsDNA is NOT

175.What is the first antigen seen in an individual with hepatitis?

infectious? 1. Poxvi

Hepatiti

rus 2. Hepatitis B virus

s Bs antigen (incubation period)

166.What is the only diploid virus? Retrovirus

176.What antibody is an indication of low transmissibility for hepatitis?

167.What types) of immune response is the body capable of making

Hepatitis Be antibody

when presented

177.What antibody is an indication of recurrent disease for hepatitis?

with a live vaccine? Humoral and cell mediated

Hepatiti

168.What HPV is the causative agent of anogenital warts? HPV 6 and

s Be antibody

11

178.Antibodies to what hepatitis B antigen provide immunity?

169.What types) of immune response is the body capable of making

Antibodies to he

when presented

patitis Bs antigen

with a killed vaccine? Humoral only

179.What are the three C's of measles? 1. Cough 2. Coryza 3.

170.Who are the "typical" women who present with endometrial

Conjunctivitis

carcinoma? Older, n

180.What vector is associated with malaria? Anopheles mosquito

on-sexually active women (whereas young, sexually active women

181.What is the vector for yellow fever? Aedes mosquito

present with cerv

182.What are the only two picornaviruses that do NOT lead to aseptic

ical carcinoma)

meningitis?

171.What type of vaccine is the MMR vaccine? Live, attenuated

1. Rhinovirus 2. Hepatitis A virus

vaccine

183.Are antibiotics helpful in treating a disease caused by a prion?

172.With what virus is postauricular lymphadenopathy associated?

No. Prio

Measles

ns are infectious proteins; thus, antibiotics are useless.

(rubella) virus

184.What is the only part of the virus that is "detectable" during the

173.In what trimester is the fetus most vulnerable to congenital rubella

eclipse p

syndrom

eriod of the viral growth cycle? The viral nucleic acid .

e? The first trimester

185.What is the only virus to carry its own ribosomes? Arenavirus

174.What is the order of the antibodies, from first to last, in an infected

186.What is the term for the period from the onset of an infection to

pati

the appear

ance of the virus extracellularly? Latent period

or for B cells? IgD

187.What is the leading cause of diarrhea in the USA? Campylobacter

198.What is the only IgG that cannot bind to Staphylococcus protein

jejuni{IND

A? IgG3

IA----ROTAVIRUS}

199.By which process do antibodies make microorganisms more

188.What organism would cause a patient to present with constant

easily ingested via

diarrhea after

phagocytosis? Opsonization

drinking mountain stream water on a camping trip? Giardia lamblia

200.Which immunoglobulin is found as a pentamer and activates

189.What

complement?

parasite

can

cause

vitamin

B12

deficiencies?

Diphyllobothrium latum

IgM

190.What viral infection is associated with black vomit? Yellow Fever

201.What is synthesized by epithelial cells, protects IgA from

(fl

degradation, and

avivirus)

transports IgA across epithelial barriers? Secretory component of IgA

191.What are the two nonspecific chemical defenses of the body?

202.Which IgG cannot activate complement? IgG4

Acidic pH and ly

203.What cell surface marker is used to distinguish different stages in

sozymes

the matu

192.What are the two nonspecific physical defenses of the body? Skin

ration of T cells? CD3

and mucus

204.What form of graft involves tissue or organ transplantation

193.What immunoglobulin is the first antibody in an immune

between genetica

response? IgM

lly identical twins? Isograft (isogenic graft, syngraft)

194.What is the major antibody of external secretions? IgA

205.True or false: antigen. antibody binding is irreversible? False; it is

195.What is the major antibody of internal secretions (blood, CSF,

rev

lymph)?

ersible because the antigens and antibodies are not linked covalently.

IgG

206.What is the term for a single isolated antigenic determinant?

196.What is the valence of an immunoglobulin molecule equal to?

Hapten (

The number of an

not immunogenic)

tigens that the antibody can bind

207.Which chromosome is associated with major histocompatibility

197.What immunoglobulin is a marker for mature B cells and is the

complex (MHC) g

antigen recept

enes? Chromosome 6

208.Which region of the variable domain comprises the antigen-

218.In a binding of helper T cells and an antigen-presenting cell

binding site of th

(APC), which i

e antibody? Hypervariable region (three per light chain; three per

s the first cell to secrete activating signals? Helper T cells (cytokines

heavy cha

to act

in)

ivate the APCs)

209.What form of transplantation crosses the species barrier?

219.Which type of cell is responsible for immunologic memory?

Xenograft

Memory B cell

210.What subdivision of MHC is found on all nucleated cells? MHC

220.What region of the immunoglobulin does not change with class

class I (thr

switching?

ee subtypes: -A,-B,-C)

Hypervariable region

211.What protein is used to differentiate MHC class I from MHC class

221.What type of cell does an antigen-stimulated B cell turn into if

II, and on

there is a

what chromosome is it found? ß2-Microglobulin, on chromosome 15

continuous supply of antigen? Plasma cell

212.What cell type recognizes MHC class I? Cytotoxic T cell (CD8)

222.What three cells are essential for T cell differentiation in the

213.What is the predominant antibody of a secondary immune

thymus?

response in the mucos

1. Dendritic cells 2. Macrophages 3. Thymic epithelial cells

al route? IgA

223.What type of T cell leaves the bone marrow? Pre-T cell (unable to

214.What substance is secreted by activated helper T cells to induce

recognize

T and B cel

antigen)

l division? IL-2

224.What cytokine stimulates stem cell differentiation? IL-3

215.What type of antigen do B cells recognize? Free, unprocessed

225.What form of T cell binds to mature B cells? Activated helper T

antigen

cell

216.What type of antigen do T cells recognize? Processed antigenic

226.What co-stimulatory molecules are necessary for effective T cell

peptides bou

and B cell

nd in the groove of the MHC molecule

signaling? B7 and CD28 (stimulatory signal for the T cell)

217.Which protein prevents internal binding of self proteins within an

227.At which site of the lymph node are B cells found? The germinal

MHC class

centers (fo

II cell? Invariant chain

llicles)

228.What are the two primary lymphoid organs? 1. Bone marrow 2.

240.What aspect of the complement system is deficient if there are

Thymus

repeated gono

229.What cytokine, produced by stromal cells of the bone marrow, is

coccal

important in

meningitis? C5, 6 ,7

myeloid development? IL-3 A " 3" on its side looks like an m for

, or 8

myeloid

241.A deficiency in C1 esterase (C1-INH) results in what disease?

230.What immunoglobulins are found in an infant at birth? Maternal

Heredita

IgG and

ry angioedema

fetal IgM

242.What form of immunity kills the host cell in order to recover from

231.What cell surface marker is found on blood B cells? CD19

intracell

232.What cell surface marker do all T cells have? CD3

ular infections? Cell-mediated immunity

233.What is the major antigen-trapping site in the immune system?

243.With what area of the spleen are the T cells associated?

The lymp

Periarteriolar L

h node

ymphatic Sheath (PALS)

234.What cells are the first to come into contact with soluble antigen

244.What are the three secondary lymphoid tissues? 1. Lymph nodes

in the ly

2. Spleen

mph node? Macrophages or dendritic cells

3. Mucosal-Associated Lymphoid Tissue (MALT)

235.In which region of the lymph node do plasma cells spend their

245.What type of cell can never leave the lymph node? Plasma cell

lives secretin

246.What is the B cell-dependent area of the spleen? The marginal

g antibodies? Medulla

zone

236.What cell surface marker is found on activated helper T cells?

247.Which major cell type is found in the red pulp of the spleen?

CD40

RBCs-tha

237.In which region of the lymph node are T cells found? Paracortex

t is why it is called red pulp.

238.Myeloperoxidase uses H202 and what to generate additional

248.At what stage of the immune response do you see an increase in

oxidants? Halide c

serum specifi

ofactor (Cl-, I-)

c antibody levels? Log phase

239.What is the main cell type of chronic inflammation? Macrophages

249.What cytokines do APCs secrete to activate helper T cells? IL-1,

infections

IL-6, TNFa

and

recurrent

episodes

of

meningococcal

250.What immunoglobulin is responsible for antibody- dependent cell-

259.What types of T cell receptors (TCRs) comprise 95% of all TCRs?

mediated cyt

Alpha an

otoxicity (ADCC) of parasites, has a high-affinity Fc receptor on mast

d beta chains

cells and

260.Which TCRs are found on the skin and mucosal surfaces?

basophils, and is responsible for the allergic response? IgE

Gamma and delta

251.What immunoglobulin is responsible for activation of complement,

TCRs

opsonizatio

261.What type of graft transplants from one individual to another with

n, and ADCC, and is actively transported across the placenta? IgG

a differe

252.What immunoglobulin activates the alternate pathway, neutralizes

nt genetic makeup (within the same species)? Allograft

bacterial e

262.What type of graft transplants from one site to another on the

ndotoxins and viruses, and prevents bacterial adherence? IgA

same person?

253.What are defined by antigen-binding specificity? Idiotypes

Autograft

254.What are the genetic variants of a molecule within members of

263.What is the term for the strength of association between multiple

the same speci

antibody-b

es? Allotypes

inding sites and multiple antigenic determinants? Avidity (greater than

255.What are different classes and subclasses of the same gene

on

products known as

e binding site)

? Isotypes

264.How is IgA found in secretions? As a dimer; it is a monomer in

256.What are the four phases of immune system defense? 1.

the blood

Recognition 2. Amplif

.

ication 3. Regulation 4. Elimination-Rare

265.What is the limited portion of a large antigen that will actually be

257.What is the term for a molecule that will trigger an immune

recogni

response?

zed and bound to an antibody and contains approximately five to six

Immunogen (It must be foreign and have at least two antigenic

amino acids

determinants.)

or four to five hexose units? Antigenic determinant (epitope) (Idiotypes

258.What

MHC

class

communication by direct c ontact? MHC class III

does

not

participate

in

immune

cell

bind to epitopes.)

266.What would be the result if an antibody were cleaved with

274.What response involves elevated levels of antibody and a short

papain? There wo

lag phase, an

uld be two Fab and Fc regions.

d requires low levels of antigen to precipitate? Secondary response

267.What would be the result if an antibody were cleaved with

275.Whose function is it to present exogenous peptides to helper T

pepsin? There wo

cells?

uld be a Fab' region; thus, it would still be able to participate in

MHC class II

precipitati

276.What process is involved when an antigen is in the endocytic

on and agglutination.

vacuole and the

268.What type of binding is involved when there is binding of one Fab

re is fusion with lysosomes, which contain proteases that cleave the

or one idi

protein ant

otype of IgG? Affinity

igens into peptide fragments? Antigen processing

269.What cytokines are secreted by helper T cells to activate the

277.Which four helper T cell cytokines are involved in differentiation?

APC? INF-y an

I. IL-4

d IL-4

2. IL-5 3. IL-6 4. IL-10

270.What acts as a target for elimination of abnormal host cells? MHC

278.Which co-stimulatory molecules are necessary for B cell

clas

differentiation (cla

sI

ss switching)? CD40 and CD40L (gp39)

271.At what stage of an immune response do we see stable levels of

279.Which cytokine is chemotactic for neutrophils? IL-8

antibody in t

280.Which nondividing cells synthesize immunoglobulins in great

he serum? Plateau phase

amounts?

272.At what stage of an immune response do we see catabolism

Plasma cells

without synthesis o

281.Which process is involved in rearranging the DNA that encodes

f antibody, causing a decline in the levels? Antigen processing

for the consta

273.What stage of the immune response is involved from the time

nt region of the heavy chain? Class switching

when we are firs

282.What are the two functions of the thymus in T cell differentiation?

t presented with an antigen to the first time that there are detectable

1. Hormo

levels o

ne secretion for T cell differentiation 2. T cell education to recognize

f antibody in the serum? Lag phase

self fr

om nonself

290.Which complement fragments cause lysis of cells? C5b-9

283.Which process is involved in rearranging one heavy chain gene to

291.Which complement fragment is deficient if a patient presents with

produce a f

repeated i

unctional gene product, while it shuts off the rearrangement and

nfections, fever, rash, and arthralgia? C3

expression of t

292.What three factors cause opsonization? IgG, C3b, and mannose

he other alleles to ensure that one antibody type is made? Allelic

binding pr

exclusio

otein

n

293.Which cell surface marker binds to C3d fragments? CD21

284.What are the three rules of clonal selection? 1. One cell type 2.

294.What disease is associated with an inherited deficiency in

One

NADPH oxidase, in

antibody type 3. Random selection of hypervariable regions, and only

which the individual is likely to develop infections with catalase-

cells with

positive org

bound antigen undergo clonal expansion

anisms? Chronic granulomatous disease

285.When is the last time that maternal IgG is seen in circulation?

295.What cells are antigen-specific and have MHC restricted killing?

Between

Cytotoxi

9 and 15 months

c T cells

286.What cytokine, produced by stromal cells of the bone marrow, is

296.What large granular lymphocytes have CD16 and CD56 as cell

important in

surface markers,

lymphoid development? IL-7 A "7" upside down looks like an "L" (L for

do not secrete immunoglobulins, and are not antigen specific? NK

Lymphoid

cells

).

297.What three complement fragments are also anaphylatoxins? 1.

287.Which region of the thymus contains the mature T cells? The

C3a 2. C4a 3.

medulla (The

C5a

y mature in the cortex.)

298.What pathology is associated with low levels of immunoglobulin

288.What are the cell surface markers of helper T cells? CD28 and

that persist

CD4

in children for up to 2 years? Transient hypogammaglobulinemia of

289.What are the cell surface markers of cytotoxic T cells? CD28 and

childhood

CD8

299.What cell surface marker is found on pre-T cells? Tdt

300.What cell surface marker is required for class switching? CD40

4.What region of the aorta is affected in syphilitic (luetic) aneurysms?

301.What large granular lymphocytes are stimulated by IL-2 and INF-

Ascending arch or the root

y and are act

5.What organism is associated with hyaline membrane formation and

ivated by natural killer (NK) cells? Lymphokine-activated killer (LAK)

cold agglutini

cells

ns? Mycoplasma

302.Which cells recognize the Fc region of IgG and carry out ADCC?

6.What cell type is commonly elevated in asthma? Eosinophils

Killer c

7.What enzyme level is commonly elevated in sarcoidosis?

ells

Angiotensin conv

303.What are the four chemotactic agents? 1. C5a 2. Leukotriene B4

erting enzyme (ACE)

3. IL-8

8.What form of lung cancer is commonly associated with asbestosis?

4. Bacterial peptides

Malignan

304.What are the five main oxidizing reactions that are used to kill

t mesothelioma

ingested or

9.With what protein-losing enteropathy do you see hypertrophied

ganisms?

1.

H202

2.

Superoxide

3.

Hydroxyl

radical

4.

gastric rugal fo

Myeloperoxidase 5.

lds? Menetrier's syndrome

Hypochlorous acid

10.What region of the GI tract does Giardia lamblia most commonly

305.True or false-it is perfectly normal to have low levels of IgG from 3

affect?

to 12

Duodenum

months of age? True; it is called physiologic hypogammaglobulinemia

11.What

of infancy.

Entamoeba histol

1.What valve of the heart is most commonly affected in IV drug

ytica

abusers? Tricuspi

12.What hepatic pathology is associated with ingestion of oral

d valve

contraceptives?

2.What is the first sign of reversible cellular damage? Ballooning or

Liver adenoma

hydropic c

13.True or false alpha-1-antitrypsin deficiency can lead to cirrhosis of

hanges secondary to mitochondrial injury

the liv

3.What is another name for isolated right-sided heartfailure? Cor

er? True; it is most commonly associated with panacinar emphysema

pulmonale

of the low

organism

is

associated

with

"flask-shaped"

ulcers?

er lobes.

23.What

14.What organism is associated with cholangiocarcinoma? Clonorchis

Vacuolation of t

sinensis

he mitochondria

15.True or false-polycythemia is associated with renal cell carcinoma?

24.Antinuclear antibodies (ANA), anti-dSDNA, and smooth muscle

True, be

antigen (Sm ag) a

cause there is an increase in erythropoietin

re all used to diagnose what disease? Systemic lupus erythematosus

16.What cancer of the male genitourinary system is associated with

(SLE)

osteoblastic

25.SS-A(Ro), SS-B(La), and R-ANA are diagnostic markers of what

bony metastasis? Prostatic carcinoma

disease?

17.What organism is commonly associated with IUD infections?

Sjogren's disease

Actinomyces

26.What disease is characterized by decreased bone resorption due

18.What female genitourinary pathology is associated with elevated

to defective o

levels of hum

steoclast function? Osteopetrosis (Albers- Schonberg disease)

an chorionic gonadotropin (hCG) and has a "snowstorm" appearance

27.What metaplastic cellular change (from what cell type to what cell

on an ultrasoun

type) occu

d? Hydatidiform mole

rs

19.What subset of leukemia involves gingival hypertrophy? Acute

(adenocarcinoma)

myelocytic

28.Antinuclear antibodies (ANAs) and anti-SCL-70 antibodies are

leukemia (AML)-M5, also called acute myelogenous leukemia

diagnostic of wh

20.What chemical pathogen is associated with squamous cell

at disease? Scleroderma

carcinoma of the skin

29.What T cell defect is found in Graves' disease? Defect in antigen-

? Arsenic

specif

21.With what form of cancer is diethylstilbestrol (DES) associated?

ic suppressor T cells

Vaginal

30.A gastric mucosal erosion caused by extensive burns to the body

clear cell adenoma

surface is kn

22.What form of cancer is most commonly associated with

own as what? Curling's ulcer (Burns = hot = curling iron) VXP

nitrosamines? Gastric

31.A superficial gastric mucosal erosion causing an excessive

cancer

secretion of pepsi

in

cellular

process

Barren's

defines

esophagus?

irreversible

Squamous

to

cellular

injury?

columnar

cell

n, due to an increase in intracranial pressure from trauma or surgical

s node (due to metastatic gastric carcinoma)

injury to

41.What pathology is associated with Budd-Chiari syndrome? Hepatic

the CNS, is known as what? Cushing's ulcer

venous t

32.What two antibodies are used to diagnose Hashimoto's thyroiditis?

hrombus

Antithyr

42.Cell fragments of hemolysis are known as what? Helmet cells

oglobulin and antimicrosomal antibodies

(schistocyt

33.What is the term for the copper corneal deposits found in Wilson's

es)

disease?

43.What is the name given to the red blood cell that contains a

Kayser-Fleischer rings

peripheral rim o

34.What is the triad of Meig's syndrome? 1. Right-sided hydrothorax

f hemoglobin along with a dark central area containing hemoglobin?

2. As

Target c

cites 3. Ovarian fibroma

ell

35.What is the triad of Plummer-Vinson syndrome? 1. Atrophic

44.What are the three causes of a microvesicular fatty change in the

glossitis 2.

liver?

Esophageal webs 3. Iron-deficiency anemia

1. Tetracycline toxicity 2. Reye's syndrome 3. Fatty liver of pregnancy

36.In multiple myeloma, what are the immunoglobulin protein droplets

45.Abnormal, unstable RNA within a developing RBC is known as

within plas

what? Basophil

ma cells called? Russell bodies

ic stippling

37.Obstetric hemorrhage or shock to the anterior pituitary leads to

46.What is the term for an RBC with inorganic iron-containing

what syndrom

granules? Siderocy

e? Sheehan's syndrome (approximately 90% destruction of the

te

pituitary)

47.A patient presenting with dark urine, pale stools, and itchy skin has

38.What is the term for a twisting of a loop of bowel leading to an

what fo

obstruction?

rm of jaundice? Obstructive (direct) jaundice

Volvulus

48.With what form of hepatitis do you see elevated levels of AST

39.In what condition is a strawberry gallbladder seen? Cholesterolosis

(SGOT) and GGT

40.What is the term for a palpable left supraclavicular lymph node?

liver enzymes?

Virchow'

In both alcoholic 2nd viral hepatitis, AST and ALT are elevated. SGOT

s

> SGPT in

59.What dye is a major cause of transitional cell carcinoma of the

alcoholic hepatitis, and SGPT > SGOT in viral hepatitis.

urinary bladd

49.What region of the liver is affected during shock? Pericentral

er? Naphthalene

(necrotic) reg

60.Interstitial fibrosis of the lower lobe is pathognomonic of what?

ion

Asbestos

50.What

drug

can

cause

amyloidosis

and

focal

segmental

is

glomerulosclerosis in the

61.With what is cherry red intoxication associated? Acute carbon

kidney? Heroin

monoxide po

51.What type of metal poisoning causes basophilic stippling? Lead

isoning

52.What drug causes a six- fold increase in schizophrenia, can impair

62.With what disease do you see Brushfield's spots? Down syndrome

motor acti

63.What is the causative organism associated with: Acute infectious

vity, and can cause lung problems? Marijuana ...

endocarditis

53.With what disease do you see dust-containing macrophages within

? Staphylococcus aureus; Aureus begins with an "a" just like acute.

a reticulin m

64.Subacute infectious endocarditis? Streptococcus viridans

esh? Coal workers' pneumoconiosis

65.What percentage of a vessel is stenosed in order to cause sudden

54.With what two pathologies is a honeycomb lung associated? 1.

cardiac deat

Asbestosis 2.

h? Greater than 75% of the vessel

Silicosis

66.What foci of fibrinoid necrosis are surrounded by lymphocytes and

55.What is Caplan's syndrome? Rheumatoid arthritis with silicosis

macrophages

56.What chemical can be potentially dangerous if you work in the

throughout all the layers of the heart? Aschoff's bodies of rheumatic fe

aerospace indus

ver

try or in nuclear plants? Beryllium

67.What cardiac pathology are children with pharyngeal infections

57.What are the three causes of angiosarcoma of the liver? 1. Vinyl

more likely to

chlorid

develop? Rheumatic heart disease

e 2. Thorium dioxide 3. Arsenic

68.Where is the embolism site for a right-sided heart lesion? The

58.What causes acellular fibrosis in the upper zone of the lung?

lungs

Silicosi

69.What is the term for white retinal spots surrounded by

r leaflet due to the long chordae tendineae

hemorrhage? Roth's s

80.What disease affects medium to small arteries, occurs most

pots

commonly in men, i

70.In what condition are they seen? In bacterial endocarditis

s associated with elevated perinuclear anti-neutrophilic cytoplasmic

71.The painless hemorrhagic areas on the palms and soles in a

antibody (P

patient with bacte

-ANCA) levels to myeloperoxidase, and involves 30% of patients with

rial endocarditis are known as what? Janeway lesions

hepatitis Bs

72.What pathology involves a diastolic blood pressure greater than 90

antigen? Polyarteritis nodosa (PAN) Remember by P in P-ANCA and

mmHg or a

P in PAN.

systolic blood pressure greater than 140 mmHg? Hypertension

81.With what disease do you see IgA deposits in small vessels of the

73.What are the four cardiac abnormalities associated with tetralogy

skin and th

of Fallot?

e kidneys? Henoch-Schonlein purpura

1. Shifting of the aorta 2. Hypertrophy of the right ventricle 3.

82.What type of vasculitis presents with headache, facial pain, and

74.Interventricular septal defect 4. Pulmonary stenosis 3X (SHIP)

sometimes lo

75.What term applies when a single vessel receives blood from both

ss of vision? Temporal arteritis (usually in women older than 80 years

ventricles?

of age)

Truncus arteriosus

83.What

76.How do you diagnose a patient who presents with an

involvement associ

underdeveloped right ventr

ated with skin rash and lymphadenopathy, and involves 70% of

icle, atrial septal defect, and no tricuspid valve? Tricuspid atresia

persons with corona

77.What are the three left-to-right shunts? 1. Ventricular septa] defect

ry artery aneurysms? Kawasaki disease; Think kids, koronary,

(VS

konjunctiva-Kawa

D) 2. Atrial septal defect (ASD) 3. Patent ductus arteriosus (PDA)

saki

78.What type of coarctation of the aorta is associated with Turner's

84.What is the difference between a true aneurysm and a false

syndrome?

aneurysm? A true a

Preductal (infantile) type

neurysm is the bulging of an arterial wall that is intact, whereas a false

79.Which leaflet is most commonly affected in mitral valve prolapse?

aneur

Posterio

type of

vasculitis affects children,

has conjunctival

ysm is a rupture where a sac is formed by the tissue adjacent to the

submaxillary gland, and submandibular gland with posterior uveal

artery.

tract involveme

85.With what lung pathology do you see a collapsed leathery lung?

nt? Mikulicz's syndrome

Acute re

92.In what autosomal recessive (AR) disease do you see a decrease

spiratory distress syndrome CARDS)

in adenine tri

86.In which region of the lung are 75% of the pulmonary infarcts

phosphatase (ATPase) activity of dynein arms in cilia, also known as

seen? Lower lo

immotile ci

bes

lia syndrome? Kartagener's syndrome

87.What is it called when the cervical lymph node is involved in Tb?

93.What disease involves a decrease in al-antitrypsin activity, causing

Scrofula

all the

88.What is the difference between Raynaud's disease and Raynaud's

alveoli to be affected? Panacinar emphysema

phenomenon?

94.What lung pathology involves columnar to squamous cell

Disease occurs when there is vasospasm in a small artery without any

metaplasia and cannot

underlying

be diagnosed unless the patient has 3 months of productive sputum

pathology, whereas a phenomenon has some underlying pathology

for 2 years or

associated with it

more? Chronic bronchitis

.

95.What bronchogenic carcinoma: Is found in the peripheral aspect of

89.What is the difference between a Ghon focus and a Ghon

the lung an

complex? A Ghon f

d is known as the "scar" carcinoma? Adenocarcinoma

ocus is a TB tubercle, whereas a Ghon complex is a focus with hilar

96.Has a poor prognosis because 50% of the cases have

lymph node i

metastasized to the brain

nvolvement.

by diagnosis? Undifferentiated cell carcinoma

90.What is the term for laminated concretions of Ca2+ and protein

97.What hematoma involves lucid intervals and affects the middle

found in granu

meningeal arter

lomas, especially sarcoidosis? Schaumann's bodies

y? Epidural hematoma

91.What condition is manifested by bilateral sarcoidosis of the parotid

98.What renal pathology involves uniform thickening of the glomerular

glands,

capillary

wall, "granular" appearance under the microscope, and effacement of

idney, just to name a few.

foot process

107.What type of peptic ulcers is associated with elevated gastric

es? Membranous glomerulonephritis (MGN)

secretions an

99.What syndrome has massive proteinuria, hypoalbuminemia,

d blood group O and responds well to cimetidine? Duodenal ulcer,

hyperlipidemia, and a

whereas

nasarca as its components? Nephrotic syndrome

gastric ulcers are associated with blood group A and low gastric

100.What renal pathology is associated with an alteration of the

secretions

basement membra

108.What GI pathology can be caused by a patient taking clindamycin

ne and mesangial cell proliferation, along with a tram-track

or lincomyci

appearance under th

n or by Clostridium diffcile, ischemia, Staphylococcus, Shigella, or

e microscope? Membranoproliferative glomerulonephritis (MPGN)

Candida inf

101.What is the triad of renal cell carcinoma? 1. Hematuria 2.

ections? Pseudomembranous colitis

Costovertebral p

109.How is Pseudomembranous colitis treated? With vancomycin or

ain 3. A palpable mass

metronidazole

102.What is the most common type of kidney stone? Calcium oxalate

110.What regions of the GI tract are the most common sites of peptic

103.In what condition do you see "dimpling" on the kidney's surface?

ulcers?

Pyelonep

The lesser curvature of the stomach and the first part of the

hritis

duodenum

104.What are the four most common metastatic sites for renal cell

111.What three criteria allow you to differentiate an ulcer from an

carcinoma?

erosion or c

1. Lung 2. Liver 3. Brain 4. Bone

arcinoma? 1. Less than 3 cm 2. Clean base 3. Level with the

105.What is the causative organism involved in squamous cell

surrounding mu

carcinoma of the bl

cosa

adder? Schistosomiasis haematobia

112.What is the watershed area of the GI tract? It is the most common

106.Is cigarette smoking associated with transitional cell carcinoma of

site of is

the blad

chemic bowel disease (splenic flexure of large bowel).

der? Yes. It is also a cause of cancers of the lung, esophagus, ureter,

113.What is the term for hypoperfusion of an area involving only the

and k

inner layer

s? Mural infarct

120.With what autosomal dominant disease do you see osteomas in

114.Failure of neural crest cells to migrate to the rectum and sigmoid

the mandible and

colon is

maxilla, epidermoid cysts, and edematous polyps in the GI tract?

known as what? Hirschsprung's disease (congenital megacolon)

Gardner'

115.What is the term for the irregular linear lacerations in the long axis

s syndrome

of th

121.In what autosomal recessive disease do you see a decrease in

e esophagus seen in chronic alcoholics? Mallory-Weiss tears

ceruloplasmin,

116.What is the condition in which progressive dysphagia and

micronodular cirrhosis, and Kayser-Fleischer rings? Wilson's disease

regurgitation occur

122.What is the term for the smooth, round fragments of nuclear

owing to failure of the lower esophageal sphincter to relax when

chromatin seen i

swallowing is

n RBCs? Howell-Jolly bodies

initiated? Achalasia (can also be seen in Chagas' disease)

123.What form of hepatitis involves Kupffer's cell hyperplasia,

117.With what autosomal dominant disease do you see polyps in the

ballooning hepat

GI tract that

ocytes, councilman bodies, and an increase in ALT? Viral hepatitis

can be large and pedunculated, along with melanin pigmentation of

124.What X-linked recessive disease involves a decrease in

the oral mucos

hypoxanthine guanine

a, lips, and palms? Peutz-Jeghers syndrome (rarely predisposes to

phosphoribosyl transferase (HGPRT), mental retardation, self-

colon canc

mutilation, choreoa

er)

thetosis, and spasticity and an increase in uricemia? Lesch-Nyhan

118.What is the most common hemolytic anemia that has an elevated

syndrome

reticulocyte c

125.In what X-linked recessive disease is there a decrease in the

ount and a positive result on a Coombs' test? Autoimmune/immune

hexose monopho

hemolytic anem

sphate (HMP) shunt, along with Heinz body formation? G-6-PD

ia

deficiency (gluco

119.What is the first sign of megaloblasdc anemia on a blood smear?

se-6-phosphate dehydrogenase deficiency)

Hyperseg

126.What illegal drug can cause rhabdomyolysis, myocardial

mented polymorphonucleocytes (more than five lobes)

infarction, cerebral infarct, and lethal cardiac arrhythmias? Cocaine

127.What type of metal poisoning causes mental retardation,

l 4 to 8 days after an MI? SGOT

somnolence, convulsi

136.What are the four right-to-left shunts? 1. Tricuspid atresia 2.

ons, and encephalopathy? Lead

Truncus

128.What type of acute metal poisoning involves stomach and colon

arteriosus 3. Transposition of the great vessels 4. Tetralogy of Fallot;

erosion and ac

All of

ute tubular necrosis? Mercury

the Ts

129.What enzyme level increases in hours and falls 24 to 48 hours

137.What form of angina pectoris is classically brought on by exercise

after a myocar

or by an

dial infarction? Creatinine phospholanase (CPK)

elevated heart rate? Stable angina (S-T depressions are seen on

130.What form of infectious endocarditis is most likely to metastasize?

EKG.)

Acute in

138.What form of angina pectoris is the most serious and is referred

fectious endocarditis

to as "prei

131.What is the terminology for the secondary disease of benign

nfarction angina"? Unstable angina

hypertension in

139.What marker increases in 12 hours, peaks in 48 to 72 hours, and

the kidney? Benign nephrosclerosis

falls in 1 t

132.What is the term for the appearance of the kidney in malignant

o 2 weeks after an MI? Lactate dehydrogenase (LDH)-1 > 2

hypertension

140.What form of angina pectoris is caused by coronary artery

(it has petechiae on its surface)? Flea-bitten kidney (can also be seen

vasospasms?

in

Prinzmetal's angina (variant). S-T elevations are seen on EKG.

pyelonephritis)

141.What is the condition that involves focal accumulations of

133.What is the term for fibrinoid necrosis of the arterioles in the

basophilic, mutin

kidney seco

ous extracellular substance in the media of the aorta? Cystic medial

ndary to malignant hypertension? Onion skinning

necrosis

134.What are the three most common sites for left-sided heart

142.When do you see an elevated blood pressure in the upper

embolisms to metas

extremities with hyp

tasize? 1. Brain 2. Spleen 3. Kidney

otension in the lower extremities? Coarctation of the aorta

135.What enzyme levels increase in 6 to 8 hours, peak in 24 to 48

(postductal), a

hours, and fal

dult type

143.What is the characteristic radiographic finding in postductal

with skin flushing, cramps, diarrhea, and nausea; and affects the

coarctation of

valves on the

the aorta? Notching of the ribs due to dilated internal mammary

right side of the heart along with the endocardium? Carcinoid tumor of

arteries

the h

144.What test uses p24 protein when diagnosing HIV? ELISA test

eart

(enzyme-linke

150.What is the consolidation around the small bronchi known as?

d immunosorbent assay test)

Bronchop

145.What type of vasculitis occurs in male heavy smokers, typically

neumonia

younger than

151.What disease usually occurs in men in their 40s who present with

35 years of age, that can cause gangrene in the lower extremities?

persistent

Buerger'

pneumonia, chronic sinusitis, renal disease, and mucosal alterations,

s disease

and show a

146.Low to absent levels of pulmonary surfactant along with hyaline

n increase in cytoplasmic antineutrophilic cytoplasmic antibodies (C-

membrane for

ANCAs)?

mation is the hallmark of what? NRDS (neonatal respiratory distress

Wegener's granulomatosis

syndrome)

152.What disease involves a weakened pulse and hypotension in the

147.What form of emphysema affects the upper lobes, increases in

upper extremit

smokers, and ha

ies, visual problems, and dizzy spells and occurs in Asian females

s carbon deposits distal to the affected areas? Centriacinar

who are 15 to

emphysema

45 years of age? Takayasu's arteritis (pulseless disease)

148.Is right ventricular hypertrophy (RVH) in utero a sign of

153.What bronchogenic carcinoma occurs most commonly in men

coaretation of the

and smokers; is foun

aorta? Yes, if you are referring to preductal or infantile coarctation of

d centrally; and can have eetopic production of adrenocorticotrophic

the a

hormone (AC

orta; congestive heart failure is also a sign of the infantile form.

TH) or antidiuretic hormone (ADH)? Small-cell carcinoma (oat cell)

149.What tumor involves an increase in 5-hydroxyindoleacetic acid; is

154.What bronchogenic carcinoma is commonly associated with

associated

Addison's disease? Squamous cell carcinoma

155.What disease, diagnosed by exclusion, involves bilateral hilar

163.What autosomal dominant syndrome involves 1000 or more

lymphadenopat

edematous polyps, mos

hy and noncaseating granulomas in many organs and is most

t commonly affects the colorectal area, and is associated with

commonly seen in black

chromosome 5q21?

women? Sarcoidosis

Familial poly posis coli

157.What causes a nutmeg liver? Right-sided heart failure

164.What autosomal dominant disease involves hyperkeratosis of the

158.What bronchogenic carcinoma is associated with an elevated

palms and sol

level of Cat+, in

es in association with esophageal carcinoma? Tylosis

volves keratin pearls, occurs in men more than women, is associated

165.What primary carcinoma is associated with signet ring cells?

with smoking

Gastric

, occurs in the major bronchi, and is seen in the central areas of the

carcinoma

lung?

166.What liver pathology involves an increase in AST, GGT, Mallory

Squamous cell carcinoma

bodies, fatty

159.What GI pathology involves a loss of villi and a decrease in the

change, and micronodular cirrhosis? Chronic alcoholism

absorptive

167.What is the term for a reversible change in which one adult cell

area due to gluten sensitivity? Celiac disease

type is rep

160.What tumor spreads via the lymphatic system into the

laced by another adult cell type? Metaplasia

peritoneum, rectal shel

168.What is a decrease in the size of cells because of the loss of

f, and both ovaries, secondary to metastatic gastric carcinoma?

structural co

Krukenberg's tum

mponents known as? Atrophy

or

169.What hemoglobin-derived pigment contains iron? Hemosiderin

161.What is necrosis and distortion of liver architecture with nodular

170.What brown-black pigment is derived from the oxidation of

regenerat

tyrosine? Melanin

ion and fibrosis known as? Cirrhosis

171.What form of necrosis is seen in the lower extremity or the bowel

162.What disease is due to increased iron deposits causing

due to vas

congestive heart fail

cular occlusion? Gangrenous necrosis

ure

(CHF),

bronze

Hemochromatosis

diabetes,

and

micronodular

cirrhosis?

172.What is the term for depositions of Ca2+ in normal tissue due to hypercalcem

ia? Metastatic calcifications

182.In what rare autosomal recessive disorder do you see

173.What is the most common type of necrosis? Coagulative necrosis

neutropenia, defective

174.What is the increase in the number of cells known as?

degranulation, and delayed microbial killing due to a problem in

Hyperplasia

chemotaxis and

175.What form of necrosis is caused by the actions of lipases on

migration? Chediak-Higashi syndrome

adipose tissue,

183.What is the main cell type of chronic inflammation? Macrophages

seen in acute hemorrhagic pancreatitis? Fat necrosis

(from blood

176.What is the term for cells that have undergone proliferation and

monocytes)

atypical cy

184.What are the two main components of granulation tissue? 1.

tologic alterations involving all sizes, shapes, and orientations?

Fibroblasts 2

Dysplasi

. Neovascularization

a

185.What causes platelets to bind to the collagen of the basement

177.What pigment, called the "aging pigment," is due to free radical

membrane?

injury and

von Willebrand's factor (factor VIII)

lipid peroxidation and is seen in brown atrophy? Lipofuscin

186.What is the term for ischemia to the CNS causing dissolution of

178.The increase in the size of cells due to synthesis, causing an

the tissue d

increase in t

ue to the actions of hydrolytic enzymes? Liquefactive necrosis

he size of the organ, is known as what? Hypertrophy

187.What is a sudden loss of oxygen to tissue, causing death of the

179.What is the term for a collection of epithelioid cells surrounded by

cells becaus

a rim o

e of a cessation of blood flowing to a particular area, known as?

f lymphocytes? Granuloma

Coagulat

180.What form of necrosis is caused by immune-mediated vascular

ive necrosis (most common in heart and kidneys)

damage? Fibrinoi

188.What is the term for depositions of Ca2+ in nonviable or dying

d necrosis

tissue, when

181.What is the term for excess amounts of granulation tissue that

the calcium level in the blood is normal? Dystrophic calcifications

can block reepithelialization

189.What type of necrosis is seen as a part of granulomatous

and wound healing? Proud flesh

inflammation? Gaseous necrosis

190.What are the three causes of transudate? 1. CHF 2. Cirrhosis 3.

199.What is a localized area of necrosis caused by circulatory

Nephrosis

insufficiency kno

191.What is the type of healing that occurs in a clean surgical

wn as? Infarction

incision?

200.What is the term for an abnormal amount of collagen type III that

Primary intention

produces a

192.What protein causes clot retraction? Thrombosthenin

large bulging scar, seen primarily in blacks? Keloid

193.What protein acts as a binding factor in wound healing and

201.What component of the basement membrane binds to collagen

embryogenesis?

type IV and hepari

Fibronectin

n sulfate and is a cell surface receptor? Laminin

194.What IgE-mediated cell secretes major basic protein and has

202.What is the term for an excessive production of collagen that

elevated levels

flattens out a

in the blood during asthma and parasitic infections? Eosinophils

nd does not extend beyond the site of the injury? Hypertrophic scar

195.What are the three platelet aggregating factors? 1. Adenosine

203.What are the three Bs of adult polycystic kidneys? 1. Big 2.

diphosphate

Bilateral 3. B

(ADP) 2. Prostaglandin 3. Thromboxane A2 (TXA2)

erry aneurysm

196.What is the triad of fat embolism? 1. Petechiae 2. Hyperactive

204.What pathway in the coagulation cascade is activated after tissue

mental statu

injury?

s 3. Occurs within 24 to 48 hours of the initial insult (e.g., long bone

Extrinsic pathway

fractur

205.What is an intravascular mass that is carried from its point of

e)

origin to a

197.An intense inflammatory reaction, an increase in the amounts of

distant site known as? Embolism

granulation

206.In what disease do you see horseshoe kidneys, rockerbottom

tissue and wound contraction by myofibroblasts are the characteristics

feet, low-set ear

of what?

s, micrognathia, and mental retardation? Edward's syndrome (trisomy

Healing by secondary intention

18)

198.What are the two most common origins of pulmonary embolism?

207.What genetic disease has an increased risk for developing

1. Deep veins of

carcinoma of the b

the legs 2. The prostatic plexus of veins in the pelvis

reast? Klinefelter's syndrome

208.What is the predominant cell type of humoral immunity? B

chemic necrosis that affect the head of the femur, humerus, and tibia?

lymphocytes

Caisson'

209.What protein causes fibrinolysis? Plasmin

s disease

210.What factors in the coagulation cascade need Ca2+ to be

219.What type of hemostasis occurs in an intravascular space and

activated? Factors

consists of fib

II and X

rin, platelets, and red and white blood cells? Thrombus

211.What tumor comprises 40% of all testicular tumors in children?

220.What disease involves mental retardation, flat face, muscle

Teratoma

hypotonia, and a

212.What most common germ cell tumor can be detected by elevated

"double-bubble" sign on an x-ray and poses an increased risk of

hCG levels and

221.Alzheimer's disease and acute lymphocytic leukemia (ALL)?

metastasizes to the bone, lung, and liver? Choriocarcinoma

Down syndrome (t

213.What pathway in the coagulation cascade is activated by making

risomy 21)

contact with

222.What are the three components of amyloid? 1. Fibrillary protein 2.

foreign substances? Intrinsic pathway

Amyloid

214.What protein is a common activator of the coagulation,

protein 3. Glycosaminoglycans

6brinolytic, and infl

223.What type of hypersensitivity is mediated by IgE? Type I

ammatory systems? Hageman factor (XII)

hypersensitivity

215.What factors in the coagulation cascade need factor Ila

224.What type of hypersensitivity involves the antigen-antibody

(activated) to becom

complex and acti

e activated? Factors V and VIII

vates complement, leading to tissue injury? Type III hypersensitivity

216.What lobe of the brain is most commonly affected by herpes

225.What type of hypersensitivity involves serum sickness, SLE,

virus? Temporal

Arthus reaction,

lobe

and acute glomerulonephritis? Type III hypersensitivity

217.What disease involves microcephaly, mental retardation, cleft lip

226.What test uses gp120 protein when diagnosing HIV? Western

or palate,

blot test

and dextrocardia? Patau's syndrome (trisomy 13)

227.What disease involves a patient with dry eyes and dry mouth; is

218.What disease, caused by decompression sickness, leads to

associated w

multiple foci of is

ith other collagen vascular diseases and B cell dysfunction; occurs in

236.With what are the following microscopic changes associated: loss

women mor

of polarity

e than men; and poses an increased risk of the patient developing a

,

high-grade B

nuclear:cytoplasmic

cell lymphoma? Sjogren's disease

ratio, hyperchromasia, and increase in the rate of mitosis? Malignancy

228.What is the most common fungal infection in HIV? Candida

237.What is the most common pathway for sarcoma to spread?

229.What is the most common infectious agent in HIV? Pneumocystis

Hematogenous pat

carinii

hway

230.What are the two most common viral infections in HIV? 1. CMV

238.What is the most common complement deficiency? C2 deficiency

retinitis

239.What type of erythema do you see in: Ulcerative colitis?

2. HSV-2

Erythema nodosum

231.What is the most common opportunistic infection of the CNS in

Rheumatic fever? Erythema marginatum

HIV? Toxoplas

Stevens-Johnson syndrome? Erythema multiforme

mosis

240.What is the triad of meningitis? 1. Fever 2. Headache 3. Stiff neck

232.What is the term for a hospital-acquired infection? Nosocomial

(Naus

infection

ea, photophobia, and irritability are also commonly seen in patients

233.What are the four DNA oncogenic viruses? 1. HPV (human

with mening

papilloma virus) 2

itis.)

. EBV (Epstein-Barr virus) 3. Hepatitis B 4. Kaposi's sarcoma- HSI/?

241.What carcinoma in the thyroid involves stromal amyloidosis and

234.What disease is X-linked recessive, presents with eczema

an excessive

thrombocytopenia an

release of calcitonin? Medullary carcinoma

d an increased chance of developing recurrent infections, involves a

242.What is the name for an adenoma in the adrenal glands that

decrease in

causes elevated l

serum IgM and in the T cell-dependent paracortical areas of the lymph

evels of aldosterone? Conn's syndrome-page230

nodes, an

243.What does prepubertal hypersecretion of growth hormone lead

d means that the pa Wiskott-Aldrich syndrome

to? Gigantis

235.What is the most common pathway for carcinomas to spread? Via

m

lymphatics

anaplasia,

pleomorphism,

discohesiveness,

increase

in

the

244.What is an ACTH-secreting tumor in the pituitary gland known

251.What bone pathology is associated with teenagers, occurrence in

as? Cushing'

men more tha

s disease

n women, hematogenous spread to the lungs, and Codman's triangle

245.What is a beta cell tumor that secretes an excess of gastrin,

on an x-ray?

which causes m

Osteosarcoma

ultiple peptic ulcers in aberrant locations, known as? Zollinger-Ellison

252.What is the collapse of the vertebral body due to Tb known as?

syndro

Pott's d

me

isease

246.What type of pancreatic tumor has the following common signs:

253.What pathology is associated with cartilage formation in the bone

hypoglycemia,

of the jaw

hunger, sweating, tremors, seizures, and coma? Insulinoma

, shoulder, and pelvic girdle and presents in middle age?

247.The

most

common

causes

of

osteomyelitis:

Overall?

Chondrosarcoma

Staphylococcus aureus

254.What disease is found in persons in the 20- to 40-year-old age

In neonates? Streptococcus agalactiae

range; affect

In patients with sickle cell disease? Staphylococcus aureu.s (but they

s women more than men; occurs at the epiphysis of the knee; and is

are mor

seen as a "so

e prone to developing salmonella infections)

ap bubble" appearance on an x-ray? Giant cell tumor of the bone

In drug addicts? P.seudomonas

255.What disease is seen in the first or second generations; is

248.What renal disease in diabetic patients is seen as a halo of

associated with

capillaries aro

chromosome 11,22; affects men more than women; and has

und the mesangial nodules? Kimmelstiel-Wilson disease

pseudorosettes and an oni

249.With what pancreatic tumor do you see watery diarrhea,

on- skin layering formation? Ewing's sarcoma

hypokalemia, and achl

256.What joint is affected causing Heberden's nodes in osteoarthritis?

orhydria? Vasoactive intestinal peptide (VIP) tumor of the pancreas

Distal i

250.What are the four most common causes of femoral head

nterphalangeal (DIP) joint (Bouchard's nodes are in the proximal

necrosis? 1. Stero

interphalangeal

ids 2. Alcohol 3. Scuba diving 4. Sickle cell anemia

[PIP] joint)

257.What vitamin D deficiency leads to a softening of the bone

munodeficiency? Adenosine deaminase

causing demineral

266.What two components of the complement cascade are

ization? Osteomalacia

anaphylatoxins? 1. C3a 2

258.Podagra, tophi in the ear, and polymorphonucleocytes (PMNs)

. C5a (causes mast cells to secrete histamine)

with monosodium

267.What

urate crystals are associated with what pathology? Gout

contraction? Bradykin

259.With what pathology are bamboo spine on an x-ray and the

ins

haplotype HLA-B27 a

268.What is the most potent vasoconstrictor and can cause platelet

ssociated? Ankylosis spondylitis

aggregation?

260.With what pathology is deposition of calcium pyrophosphate in

TXA2

patients older

269.What prostaglandin is associated with vasodilatation (edema)

than 50 years of age associated? Pseudogout

iand nhibits pl

261.What childhood pathology involves anterior bowing of the tibia,

atelet aggregation? Prostaglandin I2, (PGI2)

epiphyseal e

270.What prostaglandin is associated with vasodilatation, fever, and

nlargements, and costochondral widening with the endochondral

pain?

bones being affect

PGE2

ed? Rickets

271.What are the three characteristics in Virchow's triad of

262.What is the triad of Reiter's syndrome? 1. Peripheral arthritis 2.

thrombosis?

Conju

1. Injury to the endothelium 2. Change in laminar flow 3.

nctivitis 3. Nongonococcal urethritis

Hypercoagulation

263.What are the five chemotactic mediators? 1. LB4, 2. IL-8, 3. C5a,

272.What is collagen and fibrin surrounded by macrophages in

4. TNFa,

rheumatic fever kno

5. N-formyl-methionine

wn as? Aschoff bodies

264.What are the three opsonins? 1. C3b 2. Fc region of IgG 3.

273.What leukotriene is a major chemotactic factor that causes WBCs

Mannose-bi

to adhere to

nding protein

the endothelium? LB4

265.What enzyme is lacking in the autosomal recessive type of

274.What two substances, produced by the body, cause fever? 1.

severe combined im

Interleukin 1

causes

vasodilatation,

pain,

and

smooth

muscle

(IL-1) 2. Tumor necrosis factor (TNF)

in the third to the fifth decade? Scleroderma

275.What three leukotrienes are associated with bronchospasms and

282.What type of hypersensitivity involves autoimmune hemolytic

an increase in

anemia, erythrob

vessel permeability and vasoconstriction? LC4, LD4, and LE4,

lastosis fetalis, Goodpasture's syndrome, parasitic killing, Graves'

276.What substance, derived from IgE-sensitized basophils, causes

disease, an

increased vasc

d myasthenia gravis? Type II hypersensitivity

ular permeability, leukocyte adhesion, chemotaxis, and aggregation

283.What type of hypersensitivity involves systemic anaphylaxis and

along with be

skin and foo

ing one of the most potent platelet stimulators? Platelet-activating fact

d allergies? Type I hypersensitivity

or (PAF)

284.What joints in the hand are most commonly affected by

277.What is the most common organ involved in amyloidosis? Kidney

rheumatoid arthritis?

278.What is the term that describes when treatment for one disease

Proximal interphalangeal and metacarpal phalangeal joints

leads to anot

285.What disease is seen in the 20- to 40-year-old age group, is more

her disease presenting itself? Iatrogenic infection

prevalent

279.What disease involves a failure of the third and fourth pharyngeal

in women than men, involves diarrhea with or without bloody stools,

pouches t

starts in th

o develop, with a lack of T cell immunity causing a poor defense

e rectum and ascends without skipping areas, includes pseudopolyps,

response to cer

and has a th

tain fungal and viral infections and tetany? DiGeorge's syndrome

ickness of the bowe Ulcerative colitis

280.What disease involves a lack of both T cell-mediated and Immoral

286.What disease arises from the adrenal medulla, displaces and

immune resp

crosses the midl

onses that can be either X-linked or autosomal recessive? Severe

ine, metastasizes early, is the most common solid tumor, and is seen

combined

in the 2- t

immunodeficiency

o 4-year-old age group? Neuroblastoma

281.What pathology involves excessive fibrosis throughout the body

287.In which disease do you find a decrease in the bone density and

via increased

thickness of

fibro- blast activity, occurs in women more than men, and is most

the cortex, occurring most commonly in postmenopausal women but

commonly seen

can be induced

by steroids, old age, or idiopathic causes? Osteoporosis

people older than 40 years of age? Paget's disease of the bone

288.What is cortical hypersecretion of the adrenal gland known as?

294.What type of hypersensitivity involves a Tb test, viral infections,

Cushing'

graft re

s syndrome

jections, and tumor-associated antigen? Type IV hypersensitivity

289.What does postpubertal hypersecretion of growth hormone

295.What

cause? Acromega

cytotoxicity?

ly

Type IV hypersensitivity

290.What tumor is seen in the 2- to 4-year-old age group; does not

296.What disease has the following characteristics: occurrence in

cross the mid

women more tha

line; has immature glomeruli, tubules, and stroma; and metastasizes

n men; involvement of the second or third generation; positive ANAs;

late to the

joint pain;

lungs? Wilms' tumor

skin rash in a malar distribution; diffuse proliferate GN; Libman- Sacks

291.What GI pathology is associated with the following: a positive

endoca

string sign,

rditis; and neurolo SLE : MD SOAP N HAIR (malar rash; discoid

an increase in the number of bloody stools, RLQ pain, skip lesions,

lesion; serolo

terminal ile

gic; oligoarthritis; ANA positive; pleuritis/pericarditis; neurologic

um most commonly affected, occurrence in women more than men,

complicati

and an increased t

ons;

hickness of the bow Crohn's disease

antibodies; 3 &

292.What is associated with deposition of an extracellular amorphous

immunologic; and renal)

substance i

297.What form of hypersensitivity includes IgG- or IgM-activating

n the blood vessel walls and connective tissue and green

phagocytosis,

birefringence under pol

complement,

arized light? Amvloidosis

(ADCC)? Type II

293.What disease is characterized by a mosaic pattern of bone

hypersensitivity

marrow replacement

298.What X-linked recessive disease involves failure of maturation of

; involves high-output cardiac failure; and occurs in women more than

pre-B cell

men and in

form

hematologic

or

of

hypersensitivity

involves

T

[leukopenia/thrombocytopenia];

antibody-dependent

cell-mediated

anticardiolipin

cell-mediated

cytotoxicity

s with no surface immunoglobulins, leading to recurrent pyogenic

306.What is the most common cause of meningitis in: Neonates?

infections in i

Escherichia coli

nfants? X-linked agammaglobulinemia

or group B streptococci

299.What syndrome comprises small-cell carcinoma of the lung and

What is the most common cause of meningitis in: Immunized

myasthenia grav

children? Streptoc

is? Lambert-Eaton syndrome

occus pneumoniae

300.What pathologic process and pattern of involvement in the brain

What is the most common cause of meningitis in: Non-immunized

consist of n

children? Haemophi

ecrosis, fibrosis, edema, and gliosis going from the center out?

lus influenzae

Abscess

What is the most common cause of meningitis in: Young adults?

of the brain on a CT scan

Neisseria mening

301.What is the most common site of a contrecoup contusion? Orbital

itidis

surface

What is the most common cause of meningitis in: The base of the

of the frontal lobe

brain? Haemophi

302.What artery is the most common site of infarctions in the cerebral

lus influenzae

circulati

307.What AR disease involves a lack of phenylalanine hydroxylase?

on? Middle cerebral artery

Phenylke

303.What is the most common cause of a cerebral infarct?

tonuria (PKU)

Atherosclerosis/

308.What type of head trauma is characteristic of a fractured temporal

arteriosclerosis

or pariet

304.What

event

involves

the

following

precipitating

factors:

al bone, lucid intervals, headache, and confusion? Epidural

hypertension, diabe

hematoma

tes, aneurysms, atherosclerosis, and occurrence in blacks more than

309.What disease is seen in children younger than 5 years of age; X-

in whites?

linked reces

Intracerebral hemorrhage

sive,

305.What artery is affected in a subarachnoid hematoma? Middle

protuberant bellies;

cerebral artery

an increase then a decrease in CPK; and death in the second

cardiac

myopathies;

generation of life?

calf

pseudohypertrophy;

lordosis;

Duchenne's muscular dystrophy

-abl? Breast cancer

310.What breast pathology involves malignant cells with "halos"

317.What adenocarcinoma presents with elevated levels of acid

invading the epi

phosphatase, dihyd

dermis of the skin? Paget's disease of the breast

rotestosterone, prostate-specific antigen (PSA), and bone pain?

311.What tumor comprises 40% of all testicular tumors in children?

Prostatic carcin

Teratoma

oma

312.What

disease

has

the

following

characteristics:

has

318.What myopathy, due to autoantibodies to ACh receptors, can

autoantibodies to IgG; o

present with thym

ccurs in women more than men; and includes exophthalmos, pretibial

ic abnormalities, red cell aplasia, and muscle weakness? Myasthenia

myxedema, ner

gravi

vousness, heart palpitations, and fatigue? Graves' disease

s

313.What germ cell tumor is seen in the 15- to 35-year-old age group,

319.What two factors of the complement cascade are deficient in the

peaks when

person with

the person is 35 years of age, and is a bulky mass that spreads via

SLE? C2 and C4

the lymphat

320.What aspect of the complement cascade is defective if a patient

ic system? Seminoma

constantly p

314.What sex cell tumor causes precocious puberty, masculinization,

resents with recurrent infections with Neisseria gonorrhoeae or N.

gynecomastia

meningitidis?

in adults, and crystalloids of Reinke? Leydig cell tumor

Membrane attack complex-MAC (C5 to C9)

315.What

thyroid

pathology

causes

dwarfism,

retarded

bone

321.What disease involves "cold" skin abscesses due to a defect in

maturation, myxedema,

neutrophil ch

mental retardation, and decreased T4 with increased thyroid-

emotaxis and a serum IgE level higher than 2000? Job's syndrome

stimulating hormone

322.What X-linked recessive disease, deficient in NADPH oxidase,

(TSH) levels? Cretinism

presents with c

316.What are the following risk factors characteristic of: late

atalase- positive infections? Chronic granulomatous disease (30% are

menopause, early

AR)

menarche, obesity, nulliparity, excessive estrogen, genetic factor p53,

323.When does fetal IgM first appear in the fetal circulation? Third

and brc

trimester

(6 to 9 months)

331.What is the only immunoglobulin found in the fetal circulation in

324.Hereditary angioneurotic edema (AD) presents with local edema

the second

in which organ

trimester? Maternal IgG

s? GI, skin, respiratory tract

332.How does maternal immunoglobulin cross the placenta to get into

325.What enzyme deficiency causes increased capillary permeability

fetal circul

due to a rele

ation? Fc receptor on the heavy chain of the immunoglobulin via

ase of vasoactive peptides? C1 esterase inhibitor (C1INH)

active transpor

326.What AR disease contains the most common neutrophil defect?

t

Myeloperoxidase

333.What disease involves a deficiency in IgM; elevated IgA; normal

deficiency

IgG; and rec

327.How can a deficiency in adenosine deaminase be a bone marrow

urrent pyogenic infections, especially from pneumococci; and

suppressor?

thrombocytopenia?

It causes a buildup of dATP, which inhibits ribonucleotide reductase

Wiskott-Aldrich syndrome

and leads t

334.What syndrome in children involves hypofunction of the adrenal

o a decrease in deoxynucleoside triphosphate (a precursor of DNA),

glands due to

resulting in

bilateral

overall bone marrow suppression.

associated with meni

328.What disease involves an adenosine deaminase deficiency, B

ngococcemia? Waterhouse- Friderichsen syndrome

and T cell defici

335.What thyroid carcinoma presents with psammoma bodies?

ency,

and

defective

IL-2

receptors?

Severe

combined

hemorrhagic

infarctions,

which

are

most

commonly

Papillary carcin

immunodeficiency (SCID)

oma of die thyroid

329.What AR disease presents with B and T cell deficiencies,

336.In what disease do you see a large, hard, fibrous proliferation of

lymphopenia, an IgA

the conne

deficiency, cerebellar problems, and spider angiomas? Ataxia-

ctive tissue of the thyroid? Riedel's thyroiditis

telangiectasia

337.What thyroid pathology presents with a "cold," solitary, discrete

330.What immunoglobulins are present on the surface of mature B

nodule?

cells? IgM and

Adenoma of the thyroid

IgD

338.What cell type involves Immoral immunity? B lymphocytes

339.What

disease

involves

a

goiter,

hypothyroidism,

and

hythmias,

bone

resorption,

kidney

stones,

and

metastatic

autoimmunity due to ag-s

calcifications?

pecific suppressor T cells and occurs in women more commonly than

Primary hyperparathyroidism

in men?

348.What

Hashimoto's thyroiditis

hyponatremia, hypoglycemia

340.How is gonadal sex determined? The gonads' histologic

, and hyperkalemia along with skin pigmentation? Addison's disease

characteristics

349.What disease, with familial mental retardation, presents with large

341.How is ductal sex determined? Presence of mullerian or wolffian

everted

ducts

ears and macroorchidism? Fragile X syndrome

342.How is phenotypic sex determined? Appearance of external

350.What

genitalia

Serocystadenocarcinoma

343.What disease presents with severe mental retardation, VSD,

351.What cystic swelling of the chorionic villi is the most common

asymmetric face,

precursor of

microcephaly, and chromosome 5p deletion? Cri du chat

choriocarcinoma? Hydatidiform mole

344.What is primary hyperparathyroidism most commonly due to?

352.What estrogen- or progesterone-producing tumor is associated

Chief cell adeno

with Call-Exner

ma (80%)

bodies? Granulosa cell tumor

345.What are low levels of Ca2+ and P04 along with neuromuscular

353.What ovarian pathology is associated with Turner's syndrome or

irritability si

malformation

gns of? Hypoparathyroidism

of the genitals and comprises 50% of all malignant germ cell tumors?

346.What pathology has the following signs: severe headache,

Dysgermi

palpitations with o

noma

r without tachycardia, diaphoresis, anxiety, nervousness, and

354.What ovarian pathology involves ectoderm, endoderm, and

hypertensive episo

mesoderm in a histol

des? Pheochromocytoma

ogic section and is most commonly seen in the early reproductive

347.What pathology has the following signs: elevated levels of Cat+,

years? Dermoid

cardiac arr

cyst (teratoma)

adrenal

ovarian

pathology

pathology

presents

involves

with

hypotension,

psammoma

bodies?

355.What AR disease involves a deficiency in glucocerebrosidase,a

362.What AR disease involves a deficiency in tyrosinase, poses an

huge spleen, a

increased risk

nd engorged phagocytic cells and is associated with chromosome 1?

of developing basal cell or squamous cell carcinoma, and is

Gaucher'

associated with chr

s disease

omosome llp? Albinism

356.What disease, with an abnormality in collagen type I, presents

363.What is the most common fatal recessive disease in whites?

with "blue sc

Cystic fibrosis

lera"? Osteogenesis imperfecta

364.What AR disease has a deficiency in homogentisic oxidase that

357.What small, subareolar, solitary tumor that affects the lactiferous

causes brittle

ducts pr

, fibrillated articular cartilage, blue-black pigmentation of collagen, and

esents with bloody discharge from the nipple? Intraductal papilloma

urin

358.What cell consists of a binucleate giant cell with eosinophilic

e that turns black upon standing? Alkaptonuria

inclusions?

365.What

Reed-Sternberg cells

sphingomyelinase, massive org

359.What disease involves bilateral, enlarged, pale ovaries and

anomegaly, zebra bodies, and foamy histiocytes microscopically and

presents with in

is associated

fertility, hirsutism, obesity, secondary amenorrhea with elevated levels

with chromosome llp? Niemann-Pick disease

of LH,

366.What disease has multiple schwannomas, cafe au lait spots on

testosterone, and low levels of FSH? Polycystic ovaries

the skin, and L

360.What benign tumor of the breast, seen in the young, is well

isch

demarcated and h

Neurofibromatosis I

as a very low risk of cancer? Fibroadenoma

(chromosome 22q is with neurofibromatosis II and no Lisch nodules

361.What breast pathology commonly occurs bilaterally in the upper

either)

outer quadran

367.Name the AD disease associated with chromosome 15 in which

ts, and includes microcalcifications, hypertrophy of the ducts, apocrine

the patient has l

metapla

ong extremities, lax joints, pigeon chest, and posterior mural leaflet

sia, sclerosing adenoma, and blue-domed cysts? Fibrocystic change

prolapse,

of the breast

AR

nodules

disease

and

is

involves

associated

a

decreased

with

amount

chromosome

of

17q?

and is prone to developing dissecting aortic aneurysm? Marfan's

372.What is the most common cause of death in this disease? Suicide

syndrome

373.In utero death, caused by a complete lack of an alpha chain, is

368.What AR disease involves a defect in amino acid 508 on

known as wha

chromosome 7, causing

t? Hydrops fetalis

a defect in Cl- transportation that leads to recurrent pulmonary

\

infections and

374.What AR disease involves a substitution of valine for glutamic

an increase in viscid mucoid secretions along with pancreatic

acid at posit

insufficiencies?

ion 6 on the beta chain? Sickle cell anemia

Cystic fibrosis (Parents are usually the first to find out because the

375.What AD disease involves a defect in spectrin and leads to

baby tast

jaundice, splenom

es salty.)

egaly, and cholecystitis? Hereditary spherocytosis

369.What AD disease, associated with chromosome 19, involves a

376.What spinal cord pathology is caused by a degeneration of the

defect in the LDL

cortical spina

receptors,

leading

to

skin

and

tendon

xanthomas?

Familial

ls, leading to weakness, fasciculations, hyperreflexia, and spasticity?

hypercholestero

Amyotrop

lemia

hic lateral sclerosis (ALS)-Lou Gehrig's disease

370.In what syndrome does the patient have angiomatosis; renal cell

377.What are the two reasons for megaloblastic anemia with elevated

carcinomas;

mean corpusc

pheochromocytomas; retinal, cerebellar, medulla, or spinal cord

ular volume (MCV)? 1. Vitamin B12 deficiency 2. Folate deficiency

hemangioblastoma

378.What are the five reasons for normochromic normocytic anemia

s; and epidermal cysts? von Hippel-Lindau syndrome

with a normal M

371.What AD disease is associated with chromosome 4p; does not

CV

present until the

hypersplenism 2. Trauma 3.

person is in his or her 30s; and involves atrophy of the caudate

Anemia 4. Spherocytosis 5. Sickle cell anemia

nucleus, dilat

379.What are the four reasons for hypochromic microcytic anemia

ation of the lateral and third ventricles, and signs of extrapyramidal

with a low MCV?

lesions?

1. Sideroblastic anemias (i.e., porphyrin and heme synthesis

Huntington's disease

disorders) 2. Thala

and

an

elevated

reticulocyte

count?

1.

Autoimmune

ssemia 3. Iron deficiency 4. Lead poisoning

386.What disease affects the basal ganglia and the substantia nigra,

380.What are the three causes for normochromic normocytic anemia

produces co

with a normal M

gwheel rigidity, mask-like faces, and resting tremors; and reveals

CV and a low reticulocyte count? 1. Marrow failure 2. Cancer 3.

Lewy bodies m

Leukemia

icroscopically? Parkinson's disease

381.How is iron-deficiency anemia differentiated from the other forms

387.What pathology has bilateral periventricular plaques, perivascular

of hypochr

inflammat

omic microcytic anemia? RDW value greater than 15 (indicative of

ion, and demyelination and is thought to be autoimmune or brought on

iron-deficiency

by a viral

anemia)

infection? Multiple sclerosis

382.What CNS tumor is seen in persons between 40 and 50 years of

388.What type of vascular pathology, involving damage to the

age; occurs in

bridging veins that

men more than women; affects the cerebral brain stem; is GFAP-

drain into the superior sagittal sinus, is due to blunt trauma and is

positive; has a po

seen most

or prognosis; and microscopically displays pseudopalisades,increased

commonly in old people? Subdural hematoma

cellularity

389.What pathology is associated with a bloody lumbar puncture,

, pleomorphism, neo Glioblastoma multiforme

which is caused

383.In what CNS tumor, arising from arachnoid cells, do you see

most commonly by a ruptured berry aneurysm that produces the worst

psammoma bodies?

headache that

Meningioma occurs in (women more often than in men).

the person has ever experienced? Subarachnoid hemorrhage

384.What CNS pathology has a protracted history of seizures, occurs

390.What is the most common intramedullary spinal cord tumor?

in the 30- t

Ependymoma

o 40-year-age range, and microscopically involves "fried egg" cells?

391.What is the most common cause of dementia; occurs in women

Oligoden

more than men in

droglioma

the 60- to 90-year-old age group; and is associated with (32 amyloid,

385.In what disease do you see atrophy of the frontal and temporal

senile pla

lobes ("walnu

ques, and neuroflbrillary tangles? Alzheimer's disease

t brain")? Pick's disease

392.What common cause of hypogonadism in men involves testicular

399.What type of person has testicular tissue with female genitalia

atrophy, azoosp

and both the

ermia, gynecomastia, and Barr bodies? Klinefelter's syndrome

Y chromosome and testis present? Male pseudohermaphrodite 3 &

393.What is the main type of cell involved in cellular immunity? T

"Dude looks

lympho

like a lady!"

cyte

400.What type of person presents with male external genitalia and is

394.What is the syndrome associated with women of short stature

46XX owing

with a web neck,

to excess exposure to androgenic steroids during the early stage of

a low posterior hairline, streaky ovaries, and preductal coarctation of

gestation?

the aor

Female pseudohermaphrodite

ta? Turner's syndrome

401.What syndrome has the following components: hypertension,

395.What is the term for a person with ambiguous external genitalia

proteinuria, hemat

who has both

uria, azotemia, and oliguria? Nephritic syndrome

ovarian and testicular tissue (Note: 66% of such persons are 46 XX.)?

402.What type of glomerular nephritis (GN) occurs most commonly in

True her

children afte

maphrodite (due to an X,Y translocation)

r a pharyngeal or skin infection; is immune complex mediated; and is

396.What type of Ehlers-Danlos syndrome is X-linked recessive and

seen as "lu

is caused by a

mpy-bumpy"

defect in copper metabolism, causing a problem with cross-linking

nephritis

collagen and

403.In what type of GN, seen in the 2- to 6-year-old age group, is

elastin fibers? Type IX

there albumin

397.What type of Ehlers-Danlos syndrome involves decreased activity

uria and effacement of the visceral epithelial foot process with no

of lysyl hyd

deposits?

roxylase and affects collagen types I and III the most? Type VI (AR)

Minimal change disease (lipoid nephrosis)

398.What type of Ehlers-Danlos syndrome involves a deficiency in

404.What type of GN, associated with celiac disease and dermatitis

procollagen- Npeptidase?

herpetiformis

Type VII (AR)

, has mesangial deposits of IgA, C3, properdin, IgG, and IgM?

subepithelial

Berger's disease

deposits?

Postinfectious

glomerular

(IgA nephropathy)

skin, is a precursor to squamous cell carcinoma? Keratoacanthoma

405.What pathology has pulmonary and renal basement membrane

413.True or false-Hodgkin's lymphoma affects Waldeyer's ring? False;

involvement and cre

non-Hodgk

scent formation and involves type II hypersensitivity? Goodpasture's

in's disease affects Waldeyer's ring and the periaortic nodes

syndrome

414.What pathology is due to increased resorption or impaired

406.In what AD pathology is there a derangement of the epiphyseal

synthesis of bone,

cartilage grow

resulting in decreased bone mass,

th, causing a large skull and a normal-sized vertebral column?

postmenopause, inactiv

Achondroplasia

ity, hyperthyroidism, hyperadrenocorticism, and Ca2+ deficiency?

407.What gynecologic pathology occurs in the third and fourth

Osteopor

decades, is the ca

osis

use of 18 to 25% of all gynecologic laparoscopic procedures, and

415.What is a circumscribed, flat, nonpalpable pigmented change up

presents with c

to 1 cm?

hocolate cysts? Endometriosis

Macule (e.g., a freckle)

408.What form of GN is characteristically associated with crescent

416.What is a palpable, elevated solid mass up to 0.5 cm? Papule

formation?

417.What is the most common tumor on sun-exposed sites that rarely

Rapidly progressive glomerulonephritis (RPGN)

metastasizes

409.What type of GN has C3, IgG, C1q, C4 along with subendothelial

but is locally aggressive and has palisade arrangements of the nuclei?

deposits?

Basal ce

MPGN type I (two thirds of the MPGN cases)

ll carcinoma

410.What type of skin carcinoma occurs on sun-exposed sites, has a

418.What is an elevated, fluid-filled cavity between skin layers up to

low level of

0.5 cm?

metastasis, and involves keratin pearls? Squamous cell carcinoma

Vesicle (e.g., poison ivy)

411.A herniation of the brain through a defect in the skull is known as

419.What is an elevated, fluid-filled cavity between the layers greater

what?

than 0.5

Fungus cerebri

cm? Bulla

412.What type of skin pathology, located on the central face and on

420.What melanocytic tumor has a neural filament tumor marker and

sun-damaged

vertical or ra

and is associated with

dial growth? Malignant melanoma

427.What lymphoma arises from germinal follicles and is associated

421.What benign neoplasm has the total lesion above the level of the

with proto- o

skin with a

ncogene bcl-2 due to translocation of chromosome 14,18? Follicular

"pasted on appearance"? Seborrheic keratosis

lymphoma

422.What are the gray-black patches of verrucous hyperkeratosis,

428.What variant of small lymphocytic lymphoma has a slow course

usually found i

and elevated le

n the axillary folds, that can be a sign of an underlying malignancy?

vels of IgM that lead to hyperviscosity syndrome and blindness?

Acanthos

Waldenstrom's ma

is nigricans (It is commonly seen in obese patients.)

croglobulinemia

423.What is it called when the posterior cerebellar mass pushes the

429.What lymphoma is seen in male adolescents, is associated with a

tonsils thro

thymic mass,

ugh the foramen magnum? Tonsillar herniation

and progresses to ALL? Lymphoblastic lymphoma {verify this}

424.What form of lymphoma is diffuse in the lymph node, has an IgM

430.What is the term for the condition in which the brain is pushed

spike, slowly

under the fa

evolves to chronic lymphocytic leukemia (CLL), is seen in the old, and

lx cerebri by a one-sided mass lesion? Subfalcial herniation

also has

431.What

liver, spleen, and bone marrow involvement? Small-cell lymphoma

Craniopharyngioma

425.What lymphoma is a diffusely mixed, diffusely large cell that

432.What variant of Hodgkin's lymphoma occurs least frequently, is

grows rapidly

seen in peopl

and consists mainly of B cells along with "null" cells? Diffuse

e younger than 35 years of age, is localized, has an excellent

aggressive lymph

prognosis, and in

oma

volves mainly lymphocytes and a few Reed-Sternberg cells?

426.What lymphoma forms 30% of childhood lymphomas, is linked to

Lymphocyte predo

EBV, is associa

minant

ted with chromosome 8,14q translocation, and has a "starry sky"

433.What variant of Hodgkin's lymphoma is the most common type;

pattern of invol

involves women m

vement on a histologic section? Burldtt's lymphoma

ore than men; occurs in adolescence more than in old age; affects the lower cerv

CNS

tumor

arises

from

Rathke's

pouch?

ical, supraclavicular, and mediastinal lymph nodes with broad bands

440.What

leukemia

is

associated

with

a

chromosome

9,22

of fibrous t

translocation; constitute

issue and "lacunar" Nodular sclerosis

s 80% of childhood leukemias; and has blasts with PAS-positive

434.What variant of Hodgkin's lymphoma is widespread with

material and term

extensive fibrosis and

inal deoxy-transferase (TdT) marker present? Acute lymphocytic

necrosis, occurs in older patients, involves many Reed-Sternberg

leukemia (ALL)

cells, and has

441.What leukemia is seen in the 15- to 39-year-old age group, has

a poor prognosis? Lymphocyte depletion

blasts with m

435.What variant of Hodgkin's lymphoma can be localized or

yeloperoxidase-positive granules, tends to invade tissues, and is

widespread and has an

associated wit

intermediate prognosis with lymphocytes, eosinophils, plasma cells,

h a poor prognosis? Acute myelocytic leukemia (AML)

histiocytes

442.What leukemia is seen in the 25- to 60-year-old age group and is

, and Reed-Sternberg cells? Mixed cellularity

associated

436.What AR disease involves a deficiency in hexosaminidase A and

with chromosome 9,22 translocation, bcr-abl oncogene, and blast

cherry red spo

crisis? Chronic

ts on the retina; is seen more commonly in Jewish people; and is

myeloid leukemia (CML)

associated with

443.What leukemia is the most indolent of all leukemias; affects

chromosome 15q? Tay-Sachs disease

persons older t

437.Name the condition described by the following: Adenomas of the

han 55 years of age; and is associated with trisomy 12 (Hint: 95% are

thyroid, para

B cell neo

thyroid, and adrenal cortex along with Zollinger-Ellison syndrome?

plasms.)? Chronic lymphocytic leukemia (CLL)

Multiple

444.What type of GN has C3 MPGN type II deposits in irregular

endocrine neoplasia, type I (MEN I)-Wermer's syndrome

granular/linear fo

438.Pheochromocytoma, medullary carcinoma of the thyroid, and

ci and intramembranous deposits of unknown material? MPGN Type

adenoma of the par

II

athyroid? MEN Ila-Sipple's syndrome

445.What is the term for a News benign melanocytic tumor? Nevus

439.What cranial nerve is most commonly affected in a schwannoma?

1.Which muscarinic receptor uses a decrease in adenyl cyclase as its

CN VIII

second mess

enger? M2

13.What two drugs inhibit the release of neurotransmitters from

2.What drug is used to differentiate a cholinergic crisis from

storage granules

myasthenia gravis

? 1. Guanethidine 2. Bretylium

? Edrophonium

14.What drug blocks intragranular uptake of norepinephrine (NE)?

3.What drug causes a gradual loss of choline from the presynaptic

Reserpin

nerve terminal

e

by blocking its reuptake? Hemicholium

15.What two drugs, when mixed, can lead to malignant hyperthermia?

4.What is the only site in the body that uses Ml receptors? The

1. Succi

stomach

nylcholine 2. Halothane (Treatment is with dantrolene.)

5.What drug is an Ml-specific antispasmodic? Pirenzepine

16.In what phase of noncompetitive depolarization does no further

6.What is the most potent neuromuscular junction Mocker (NMJB),

depolarization

and also has no

occur, producing a desensitized block? Phase 2

cardiovascular side effects? Doxacurium

17.What adrenergic receptors use inositol triphosphate (IP3) and

7.What antimuscarinic is used as an inhalant for asthma? Ipratropium

diacylglycerol

8.What is the antidote for organophosphate ingestion? Atropine and

(DAG) for their second messenger system? a1-Receptors

2-PAM (pral

18.What a1-agonist is used to treat paroxysmal atrial tachycardia with

idoxime)

hypotensi

9.What is the drug of choice for atropine or tricyclic antidepressant

on? Metaraminol (a1, ß1)

(TCA) over

19.What a1-agonist, not inactivated by catechol-O- methyl transferase

dose? Physostigmine

(COMT), is

10.What is the rate-limiting step for norepinephrine synthesis?

used as a decongestant and also for treatment of paroxysmal atrial

Tyrosine hydroxy

tachycardia?

lase

Phenylephrine

11.What two enzymes are blocked by disulfiram? Aldehyde

20.What ß2-agonist is used in the prophylactic treatment of asthma?

dehydrogenase and dopam

Salmeter

ine ß-hydroxylase

ol

12.What is the monoamine oxidase B (MAOB) inhibitor? Selegiline

21.What two ß2-agonists cause myometrial relaxation? 1. Ritodrine 2. Terbutal

ine

31.In what three areas of the body are sympathetics the predominant

22.A hypertensive crisis can be caused by the addition of an MAO

tone?

inhibitor and w

1. Sweat glands 2. Arterioles 3. Veins

hat? Tyramine

32.What three ß-blockers are used in the treatment of glaucoma? 1.

23.What group of drugs is known as the "antihypertensive" group? a1-

Propranolol 2

Antag

. Timolol 3. Carteolol

onists

33What two ß-blockers decrease serum lipids? 1. Pindolol 2.

Why? Because they decrease total peripheral resistance (TPR) and

Acebutolol

preload with

34.In what area of the brain can an excess of dopamine lead to

no change in heart rate or cardiac output

psychotic symptom

24.What

mixed

a-antagonists

are

used

for

patients

with

s? Mesocortical area

pheochromocytoma?

35.What area of the brain is linked to emotion and movement?

Phentolamine and phenoxybenzamine

Mesolimbic syste

25.What a2-antagonist is used to treat impotence and postural

m

hypotension?

36.What two drugs block dopa-decarboxylase in the periphery to

Yohimbine

decrease the conv

26.What drug that penetrates the blood-brain barrier is found in

ersion of L-dopa to dopamine? 1. Carbidopa 2. Benserazide

asthma preparat

37.What is the drug of choice for early Parkinson's disease?

ions and used as a nasal decongestant? Ephedrine

Selegiline

27.What two ß2-agonists are used to produce bronchodilatation? 1.

38.What antiviral agent is used in the treatment of drug-induced

Metaprotereno

Parkinson's dis

l 2. Albuterol

ease? Amantadine

28.What ß-blocker is also an a-blocker? Labetalol

39.What drug which causes Ca2+ independent release of dopamine

29.What are the four cardioselective ß-blockers? 1. Bisoprolol 2.

is used to treat

Atenolol 3. Met

attention deficit hyperactive disorder (ADHD) and narcolepsy?

oprolol 4. Acebutolol (BAMA)

Methylphenidate

30.What ß-blocker is also a membrane stabilizer? Propranolol

40.What dopamine-2 (D2) agonist is used to treat neuroleptic malignant syndrome?

Bromocriptine

Clozapine

41.What cofactor of dopa- decarboxylase decreases the efficacy of L-

51.What is the drug of choice for status epilepticus? Diazepam

dopa?

52.What two neuroleptic agents are associated with tardive

Vitamin B6

dyslunesia? 1. Haldo

42.What three drugs can cause gingival hyperplasia? 1. Phenytoin 2.

l 2. Fluphenazine

Cyclospo

53.What neuroleptic agent causes retinal deposits, hypotension, and

rine 3. Nifedipine

torsades de

43.What antiepileptic agent has syndrome of inappropriate antidiuretic

pointes? Thioridazine

hormone (

54.What neuroleptic has amoxapine as a metabolite? Loxapine

SIADH) as a side effect? Carbamazepine

55.What is the only neuroleptic that does not cause an increase in

44.What drug is used for partial seizures and Lennox-Gastaut

weight or app

syndrome in childre

etite? Molindone

n? Felbamate

56.What TCA causes sudden cardiac death in children? Desipramine

45.What

is

the

drug

of

choice

for

trigeminal

neuralgia?

57.What TCA is used to treat enuresis? Imipramine

Carbamazepine

58.What class of antidepressants are associated with insomnia?

46.What are the first signs of overdose from Phenobarbitals?

Serotonin select

Nystagmus and at

ive reuptake inhibitors (SSRIs)

axia

59.What monoamine oxidase inhibitor (MAOI) does not cause a

47.What drug, used for partial seizures, inhibits the release of

hypertensive crisis?

glutamate and c

Selegiline

auses rashes in 45% of patients taking it? Lamotrigine

60.What occurs if you mix an MAOI and a sympathomimetic? Severe

48.What benzodiazepine is used to treat absent mal seizures?

hypertens

Clonazepam

ion, which can lead to subarachnoid hemorrhage

49.What are the two drugs of choice for simple partial seizures? 1.

61.What two TCAs are considered to be heavily sedative? 1.

Carba

Amitriptyline 2. Traz

mazepine 2. Phenytoin

odone

50.What is the only neuroleptic agent that does not cause

62.What neuroleptic agent is also considered to be an antihistamine?

hyperprolactemia?

Risperid

one

75.What is the neurotransmitter (NT) at the mu receptor? ß-Endorphin

63.What TCA is used to treat obsessive-compulsive disorder and is

76.What is the NT at the delta receptor? Enkephalin

said to cause

77.What is the NT at the kappa receptor? Dynorphin

aggressive behavior? Clomipramine

78.Which type of receptor antagonist is the most clinically efficacious?

64.What neuroleptic agent causes agranulocytosis and also has no

Competitive antagonist

tardive dyslcin

79.Which type of antagonist acts on the same receptor as the agonist

esia as a side effect? Clozapine

that it blo

65.What drug decreases mood swings and is used for the manic

cks? Pharmacologic antagonist

phase of a bipolar

80.How are water-soluble drugs eliminated primarily? Via the kidneys

illness? Lithium

81.What class of pharmaceuticals are initially inactive but are then

66.What group of drugs potentiate the activity of gamma-aminobutyric

metabolized

acid (GABA)

to their active products? Prodrugs

? Benzodiazepines

82.How are drugs that are excreted via the biliary system resorbed by

67.What

benzodiazepine

is

used

for:

Anxiety/panic

attacks?

the GI tra

Alprazolam

ct? Enterohepatic cycling

68.Absence seizures and as an anticonvulsant? Clonazepam

83.What body fluid preferentially breaks down esters? Blood

69.Alcohol withdrawal and as an anticonvulsant? Clorazepate

84.Which type of antagonist takes out a drug by binding to it?

70.Status epilepticus, as a preoperative medication? Diazepam

Chemical antagon

71.Preoperative medication? Lorazepam

ist

72.What benzodiazepines are activated outside of the liver?

85.Which type of antagonist directly reverses the action of a drug by

Oxazepam, temaze

working on

pam, and lorazepam (OTL = "outside the liver")

a different receptor? Physiologic antagonist

73.What benzodiazepine antagonist is used for benzodiazepine

86.Quantal dose-response curves indicate what two things about a

overdose? Flumazen

drug in a patie

il

nt population? 1. Margin of error 2. Relative safety

74.What are the three signs of morphine overdose? 1. Pinpoint pupils

87.When an agonist drug binds to its receptor, what type of

2. De

mechanism does it ac

creased respiratory rate 3. Coma

tivate? Effector mechanism

88.What are the four types of signaling mechanisms? 1. Intracellular

102.What monobactam is resistant to ß-lactamases of some bacteria?

recepto

Aztreona

rs 2. Membrane receptors 3. Enzymes 4. Intracellular effectors

m

89.What are the two factors that influence low oral bioavailability? 1.

103.What are the drugs used in the triple treatment of Helicobacter

First

pylori?

-pass metabolism 2. Acid lability

Pepto-Bismol, metronidazole, and erythromycin or amoxicillin

90.What do the following values stand for: EDS ? Effective dose for

104.What is the commonly IV opioid used in surgery? Fentanyl

50% o

105.Which IV agent has the lowest incidence of postoperative emesis

f drug takers (median effective dose)

and has the

91.TD50? Toxic dose for 50% of drug takers (median toxic dose)

fastest rate of recovery? Propofol

92.LD50? Lethal dose for 50% of drug takers (median lethal dose )

106.What is the major pulmonary side effect of mu-activators?

93.What is the equation for a drug's ther-a e~utic index? LD50 divided

Respiratory depr

by

ession

ED50

107.Which drug causes dissociative anesthesia and is used mainly in

94.What does the FDA regulate? Efficacy and safety of drugs

pediatric su

95.Which phases of drug testing require an investigational new drug

rgery? Ketamine

exemption?

108.Which short-acting benzodiazepine is often used for conscious

Phases I, II, and III

sedation?

96.How many years do preclinical animal studies last? 2 to 5 years

Midazolam

97.How many phases of clinical testing are there? Four phases

109.What is the most commonly used IV opioid in cardiovascular

98.Before which phase of clinical testing is a new drug application

surgery? Morphine

applied for?

sulfate

Phase IV

110.What toxicities are caused by the following agents: Occupational

99.How many years does clinical testing last? 4 to 5 years

nitrous oxi

100.What antineoplastic drug is a prodrug? Flucytosine

de exposure? Anemia

101.What carbapenem is resistant to penase and is a partial cell wall

MethoxyHurane? Nephrotoxicity

inhibitor?

Halothane? Hepatitis, with or without necrosis

Imipenem

111.What is the only local anesthetic that does not cause

122.What form of antimicrobial therapy is better to treat an

vasodilatation?

immunocompromised p

Cocaine (It is vasoconstrictive.)

atient? Bactericidal

112.Do opioids increase or decrease uterine smooth muscle tone?

123.What drug used to treat alcoholism has a long half-life and is

Decrease-but the

given orally?

y increase ureter smooth muscle tone

Naltrexone

113.What are the two side effects of opioids to which the user will not

124.What is the site of action for carbonic anhydrase inhibitors?

develop

Proximal

tolerance? 1. Constipation 2. Miosis

tubule

114.What two classes of drugs can cause schizoid behavior? 1.

125.What is the site where local anesthetics bind? Inactive Na+

Clucocorticoi

channels

ds 2. Amphetamines

126.Which bactericidal agents interfere with cell wall synthesis by

115.What is another name for prolactin inhibiting factor? Dopamine

inhibiting t

116.What is the mechanism behind tardive dyskenesia? Dopamine

ranspeptidation? Penicillins

receptor upregu

127.What are the three ß-lactamase inhibitors? 1. Clavulanic acid 2.

lation

Sulbactam

117.What sedative-hypnotic is used for alcohol withdrawal? Diazepam

3. Tazobactam

118.To what is inhalational anesthesia potency proportional? Lipid

128.What sulfonamide is the drug of choice in treating: UTIs?

solubility

Sulfisoxazole

119.What is the only commonly used inhalant that is not a

Toxoplasmosis? Sulfadiazine and pyrimethamine

halogenated hydrocarbo

Malaria prophylactically? Sulfadoxine and pyrimethamine

n? Nitrous oxide

Ophthalmic infections? Sulfacetamide

120.Which sedative-hypnotic is contraindicated in patients on warfarin

Crohn's or ulcerative colitis? Sulfasalazine

therapy?

129.What

Chloral hydrate

(Sulfonamides block

121.What is the triad associated with "serotonin crisis"? Myoclonus,

dihydropteroate synthase.)

hyper

130.What are the two "broad-spectrum" penicillins? 1. Ampicillin 2.

thermia, and rigidity

Amoxici

drug

blocks

dihydrofolate

reductase?

Trimethoprim

llin

142.What two tetracyclines have the highest plasma binding? 1.

131.Which penicillin can cause interstitial nephritis? Methicillin

Doxycycline 2

132.What form of penicillin is stable in acid environments? Penicillin-V

. Minocycline

133.What form of penicillin is used in the treatment of life-threatening

143.Renal tubular acidosis, nephrosis, and amino aciduria constitute

illness

the triad o

es? Penicillin-G (benzylpenicillin)

f what syndrome? Fanconi-like syndrome

134.What is the drug of choice in the treatment of Pneumocystis

144.What is the cephalosporin of choice for Pseudomonas infections?

carinii pneumoni

Ceftazid

a? Sulfonamide/trimethoprim

ime

135.What are the five penicillinase-resistant penicillins? 1. Cloxacillin

145.Which bacteriostatic drug inhibits translocation of protein

2

synthesis by bin

. Oxacillin 3. Nafcillin 4. Dicloxacillin 5. Methicillin (CONDM)

ding to the 50S ribosomal subunit and is a narrow-spectrum antibiotic

136.Which antiviral agent is teratogenic? Amantadine

used for p

137.Which broad-spectrum antibiotic inhibits the attachment of amino

ulmonary infections? Erythromycin

acyl tRNA b

Vestibular toxicity is associated with what tetracycline? Minocycline

y binding to the 30S ribosomal subunit? Tetracycline

Hepatotoxicity is associated with what tetracycline? Chlortetracycline

138.Which tetracycline is used in the treatment of SIADH?

146.Which two cephalosporins cross the blood-brain barrier? 1.

Demeclocycline

Cefuroxime 2.

139.Which tetracycline is used when there is a decrease in renal

Cefaclor

function?

147.What

Doxycycline

mechanisms? 1. Cefam

140.With which two tetracyclines is phototoxicity associated? 1.

andole 2. Cefoperazone 3. Ceftriaxone

Doxycycline 2

148.What three cephalosporins can produce disul6ram-like reactions?

. Minocycline

1. Cefam

141.Which drug inhibits peptidyl transferase enzyme and binds to the

andole 2. Cefoperazone 3. Moxalactam

50S ribosom

149.What three cephalosporins inhibit vitamin K-dependent factors?

al subunit? Chloramphenicol

1. Cefam

three

cephalosporins

are

andole 2. Cefoperazone 3. Moxalactam

eliminated

via

biliary

150.What three cephalosporins have good penetration against

161.What is the drug of choice for methicillin-resistant Staphylococcus

Bacteroides fragilis

aureus?

? 1. Cefotetan 2. Cefoxitin 3. Ceftizoxime

Vancomycin

151.What cephalosporin-like drug has excellent coverage against

162.Which drugs bind to the 30S ribosomal subunit and interfere with

gram-positive an

the initiat

d gram-negative bacilli activity and is used in conjunction with the

ion complex, causing a misreading of mRNA? Aminoglycosides

enzyme inhi

163.What is the drug of choice for asymptomatic meningitis carriers?

bitor cilastatin? Imipenem

Rifampin

152.What

is

the

drug

of

choice

for

penicillin-resistant

164.Which three aminoglycosides have vestibular toxicity? 1.

gonococcalinfections?

Streptomycin

Spectinomycin

2. Gentamicin 3. Tobramycin

153.Which drugs block the enzyme DNA gyrase? Quinolones/nalidixic

165.What are the two most important features in the diagnosis of

acid

malaria?

154.What aminoglycoside is used before surgery to sterilize the

1. Splenomegaly 2. Anemia (With a high index of suspicion)

bowel? Neomycin

166.Which antihelmintic increases membrane permeability to Ca2+

155.What aminoglycoside causes disruption of CN I? Streptomycin

and is the drug

156.Which anti-Tb drug gives orange urine, saliva, and tears?

of choice for schistosomiasis? Praziquantel

Rifampin

167.What is the drug of choice in the treatment of the lepra reaction?

157.What is the drug of choice for amebic dysentery caused by

Clofazim

Bacillus fragil%s?

ine

Metronidazole

168.What is the drug of choice for taeniasis which inhibits oxidative

158.Which topical agent blocks the enzyme isoprenyl phosphate?

phosphoryl

Bacitracin

ation in cestodes? Niclosamide

159.What is the drug of choice for Legionella, Mycoplasma, and

169.What is the drug of choice for threadworm, trichinosis, and larva

Campylobacter inf

migrans?

ections? Erythromycin

Thiabendazole

160.What drug is used to treat Tb, tularemia, and the plague?

170.What inhibitor of microtubule synthesis is the drug of choice for

Streptomycin

whipworm a

nd pinworm? Mebendazole

180.What is the drug of choice for: Human immunodeficiency virus

171.Which bacteriostatic drug inhibits folic acid synthesis and a major

(HIV) infection

side eff

s? azidothymidine (AZT)

ect is the lepra reaction? Dapsone

181.What is the drug of choice for: Influenza and rubella infections?

172.What mosquito is responsible for the transmission of malaria?

Amantadi

Anophele

ne

s mosquito

182.What is the drug of choice for: Respiratory syncytial virus (RSV)

173.What is the drug of choice for filariasis and onchocerciasis?

infections

Diethylc

? Ribavirin

arbamazine

183.What is the drug of choice for: Human papilloma virus (HPV)

174.What drug of choice for ascaris causes neuromuscular blockade

infections?

of the worm?

a-Interferon

Pyrantel pamoate

184.What antihistamine is used in the treatment of serotonergic

175.What drug blocks glucose uptake, leading to decrease formation

crisis? Cyprohep

of adenosine

tadine

5' triphosphate (ATP) and resulting in immobilization of the parasite?

185.What are the three nonsedating antihistamines? 1. Terfenadine 2.

Albendaz

Astemi

ole

zole 3. Loratadine

176.What drug is an irreversible inhibitor of the Na+/K+ pump?

186.Skin necrosis is caused by a deficiency in what? Transient protein

Omeprazole

C defi

177.What is the drug of choice for: Herpes simplex virus (HSV)

ciency

infections?

187.What

Acyclovir or trifluridine

Chlorpromazine

178.What is the drug of choice for: Varicella-zoster? Acyclovir

188.What prostaglandin El (PGE1) analog is used in the treatment of

179.What is the drug of choice for: Cytomegalovirus (CMV)

ulcers cause

infections? Ganciclo

d by the excessive use of NSAIDs? Misoprostol

vir

189.What ß2-agonist is used as a prophylactic agent in the treatment of asthma?

D2

receptor

blocker

is

used

as

an

antiemetic?

Salmeterol

197.What drugs combined together produce neuroleptanalgesia?

190.What is the only form of insulin that can be given IV? Regular

True. They also

insulin

increase insulin release and decrease glucagon release.

191.What drug, if given during pregnancy, would cause the uterus to

198.What drugs combined together produce neurolepanalgesia?

exhibit sign

Droperidol and f

s of progesterone withdrawal and induce an abortion? RU 486

entanyl

192.What two forms of insulin, if mixed together, would result in

199.What ultra-short-acting barbiturate induces hypnosis and is

precipitation

associated with

of zinc? 1. Lente insulin 2. NPH insulin or protamine zinc insulin (PZI)

cardiovascular and respiratory depression? Thiopental

193.What blood disorder is a side effect of metformin? Megaloblastic

200.What is the most common pain killer used during pregnancy?

anemia (de

Meperidine

creased absorption of vitamin B12 and folic acid)

201.What antimicrobial agent's major side effect is: Gray baby

194.Which antineoplastic agents' site of inhibition is in: M phase of the

syndrome?

cell c

Chloramphenicol

ycle? Vinblastine and vincristine

202.What antimicrobial agent's major side effect is: CN VIII damage

Gl phase of the cell cycle? L-asparaginase and mitomycin

(vestibuloto

G2 phase of the cell cycle? Bleomycin

xic)? Aminoglycosides

Between GI and S phases of the cell cycle? Hydroxyurea

203.What antimicrobial agent's major side effect is: Teratogenicity?

Between S and G2 phases of the cell cycle? Etoposide

Metronid

S

phase

of

the

cell

cycle?

Cytarabine,

methotrexate,

6-

azole

mercaptopurine, and

204.What antimicrobial agent's major side effect is: CholestaHc

6-thioguanine

hepatitis?

195.What is the physiologic basis for the actions of birth control pills?

Erythromycin

They block the midcycle surge of luteinizing hormone (LH)

205.What antimicrobial agent's major side effect is: Hemolytic

196.True or false: Oral sulfonylureas increase the number of insulin

anemia? Nitrofur

receptors.

antoin

False-verify this answer.

206.What antimicrobial agent's major side effect is: Dental staining if used in

the pediatric population? Tetracycline

d tubules

207.What antimicrobial agent's major side effect is: Altered folate

218.What is the only diuretic that works on the blood side of the

metabolism?

nephron?

Trimethoprim

Spironolactone (binds to aldosterone receptors)

208.What antimicrobial agent's major side effect is: Auditory toxicity?

219.What is the active metabolite of spironolactone? Canrenone

Vancomyc

220.What drug is given transdermally for chronic pain but can cause

in

chest wall r

209.What antimicrobial agent's major side effect is: Cartilage

igidity if given IV? Fentanyl

abnormalities?

221.What is the lipid solubility and potency of a drug if the induction

Quinolones

and reco

210.What are the five zero-order processes? 1. Sustained release 2.

very from the drug were both rapid? Low lipid solubility and potency

IV drip

(They a

3. Phenytoin 4. Alcohol 5. Aspirin toxicity

re inversely proportional.)

211.What morphine derivative is used in patients with renal failure?

222.What is the drug of choice for hypertensive patients with a

Hydromor

decreased renal

phone

function? a-Methyldopa (Guanabenz or clonidine is also used.)

212.Which group of antihypertensive agents decreases left ventricular

223.What hormone is released by the atria due to an increase in

hypertroph

blood pressure t

y the best? Thiazide diuretics

o cause an increase in glomerular filtration rate (GFR), Na+ retention,

213.What is the site of action of: Osmotic diuretics? The entire tubule

and reni

barrin

n-angiotensin release? Atrial natriuretic factor

g the thick ascending limb

224.What is the drug of choice in treatment for the late phaseof

214.What is the site of action of: Loop diuretics? Ascending limb

asthma?

215.What is the site of action of: Thiazide diuretics? Early distal tubule

Corticosteroids

216.What is the site of action of: K+-sparing diuretics? Early collecting

225.Which antigenic thrombolytic agent causes a decreased level of

duct

circulating f

217.What is the site of action of: Aldosterone antagonists? Distal

ibrinogen? Streptokinase

convolute

226.What is the only class of diuretics to retain Cl- used in the short-

234.What is the best form of treatment if an elevated blood pressure

term tre

is due to:

atment of glaucoma and also in the treatment of acute mountain

Increase in TPR? Centrally acting sympatholytics, a-antagonists, or

sickness?

Ca2+

Acetazolamide

channel bockers

227.What diuretic is used to decrease intraocular and intracranial

235.Which class of diuretics blocks Na/Cl cotransport in the distal

pressures?

tubules?

Mannitol

Thiazide diuretics

228.Which thrombolytic agent, activated in the presence of fibrin, is

236.Why should codeine be carefully administered acetaminophen or

manufactur

aminosalicylic

ed by recombinant DNA process? Alteplase

acid? Because it has an additive effect with with these agents

229.Which diuretic causes irreversible ototoxicity and GI bleeding as

1.What

its main s

Cricopharyngeus

ide effects? Ethacrynic acid

2True or false: chewing is essential for digestion? False; it just

230.What IV agent is used to treat respiratory depression associated

increases

with withdr

the surface area of the food.

awal from alcohol usage? Naloxone

3.In which region of the stomach are parietal and chief cells located?

231.What is the best form of treatment if an elevated blood pressure

Body or

is due to:

corpus

Elevated heart rate? ß-Blockers

4.What hormone, released in response to low pH, inhibits gastric

232.What is the best form of treatment if an elevated blood pressure

emptying by dec

is due to:

reasing antral contractions, increases constriction of the pyloric

Elevated force of contractions? ß-Blockers

sphincter, an

233.What is the best form of treatment if an elevated blood pressure

d also increases bicarbonate secretions from the pancreas? Secretin

is due to:

5.In which region of the stomach are G cells located? Antrum (They

Increase in fluid volume? Diuretics or angiotensin converting enzyme

secrete gas

(ACE)

trin: G for gastrin.)

inhibitors

6.How long is the transit time through the large intestine? 3 to 4 days

muscle

comprises

the

upper

esophageal

sphincter?

7.How long is the transit time through the small intestine? 2 to 4 hours

15.What hormone is stimulated by glucose and fat in the duodenum,

8.What hormone causes contractions of smooth muscle, regulates

inhibits gastr

interdigestive mo

ic secretions and motility, and stimulates insulin secretion? Gastrin

tility, and prepares the intestine for the next meal? Motilin

inhibito

9.What is the main function of HCl in the stomach? Converts

ry peptide (GIP)

pepsinogen into

16.Which portion of the autonomic nervous system regulates salivary

pepsin

flow?

10.What hormone increases the intestinal secretions of electrolytes

Parasympathetic portion

and H2O, rel

17.A pH of less than 4.5 stimulates the release of what hormone?

axes smooth muscle, dilates peripheral blood vessels, and inhibits

Secretin

gastric secre

(It inhibits acid Secretion.)

tions? Vasoactive intestinal peptide (VIP)

18.What is composed of smooth muscle, innervated by pelvic

11.What gland produces 20% of salivary secretions and contributes to

splanchnics and hypog

almost all

astric nerves, and involuntary? Internal anal sphincter

of the amylase secretions? Parotid gland (serous secretions)

19.What

12.What hormone causes contractions of the gallbladder, augments

Submandibular gl

the action of s

and (produces both mucous and serous secretions)

ecretin to produce an alkaline pancreatic juice, inhibits gastric

20.What is the tonicity of pancreatic juice? Isotonic

emptying, and

21.What organism is associated with gastric ulcers? Helicobacter

increases constriction of the pyloric sphincter? Cholecystoldnin (CCK)

pylori

13.What are the four functions of saliva? 1. Provides antibacterial

22.What is the only gastric secretion required to sustain life? Intrinsic

action

factor

2. Lubricates 3. Begins carbohydrate digestion 4. Begins fat digestion

(IF)

14.What is composed of skeletal muscle, innervated by the pudendal

23.What three structures increase the surface area of the GI tract? 1.

nerve, and in

Plica

a voluntary constant state of contraction that relaxes for defecation?

e circularis 2. Villi 3. Microvilli

External

24.What hormone is the primary regulator of HCO3 secretion from the

anal sphincter

pancreas?

gland

produces

70%

of

total

salivary

secretions?

Secretin

(AV) difference positive or negative? Positive AV difference

\

38.In laminar flow, which area has the fastest flow? The center of the

25.What cells of the GI tract secrete mucus? Goblet cells

tube

26.What are the five F's associated with gallstones? 1. Fat 2. Forty 3.

39.What are four ways to get an increased pump function of the

Femal

heart? 1. Exerc

e 4. Familial 5. Fertile

ise 2. Increase heart rate 3. Increase in arterial pressure 4. Increase

27.Lactose intolerance is caused by a lack of what enzyme? Lactase

contract

28.What are the three end products of amylase digestion? 1. Maltose

ility

2. Ma

40.What system has an increased pressure, decreased resistance,

ltotetrose 3. Alpha limit dextrans (a-1,6 binding)

increased flow,

29.What percentage of bile acids are excreted daily? 5% (95%

increased compliance, and blood volume that is proportional to flow?

reabsorbed via e

Pulmonar

nterohepatic circulation)

y circuit

30.Which glands of the upper duodenum secrete a bicarbonaterich

41.What two organs have local metabolites as the main determinant

solution?

of blood flow?

Brunner's glands

1. Brain (cerebral circulation) 2. GI tract (after a meal) All other organs

31.What is the major route for excretion of cholesterol? Bile

are

32.What hormone potentiates the effect of secretin? CCK

under neural control.

33.What is the major phospholipid in bile? Lecithin

42.What are the three sympathetic effects on the pacemaker cells of

34.What is the rate-limiting step in the formation of bile acids? 7-a-

the heart?

Hydr

1. Increase the slope of prepotential 2. Take less time to reach

oxylase

threshold 3. In

35.What two amino acids are conjugated to bile acids to increase

crease the rate of firing

H2O solubility?

43.What are the three parasympathetic effects on the pacemaker cells

1. Glycine 2. Taurine

of the hear

36.What is absorbed in the gallbladder to concentrate bile? Water

t? 1. Hyperpolarize the cells by increasing K+ conductance 2. Take

37.If a substance is removed from circulation by an organ, is its

longer t

arteriovenous

o reach threshold 3. Decrease the rate of firing

44.What are the two major causes of arterial pressure? 1. Contraction

d equilibrate for a substance

of the he

53.Which region of the lungs has a low perfusion pressure and a high

art 2. Hydrostatic pressure

resistance

45.What causes an increase in cardiac performance with no increase

so that there is little blood flow? Apex

in preload?

54.What fluid is monitored directly by central chemoreceptors?

Contractility (inotropic)

Cerebrospinal fl

46.Change in what intracellular ion causes a change in contractility?

uid (H+;CO2)

Calcium

55.On

47.What are the two main circulations with extrinsic regulation that are

regurgitation? Increase

most af

in stroke volume

fected by nervous reflexes? 1. Cutaneous circulation 2. Resting

On a pressure-volume loop, what is seen with:Aortic stenosis?

skeletal mus

Increase in afte

cle

rload, decrease in stroke volume, increase in peak tension

48.What is the third heart sound caused by? Ventricular filling (heard

On a pressure-volume loop, what is seen with:Increased contractility?

durin

Increase

g diastole)

in stroke volume by decreasing the end-systolic volume

49.What is the fourth heart sound caused by? Atria] contraction

On a pressure-volume loop, what is seen with:Heart failure? Increase

(heard during

in endsystolic

diastole)

volume, decrease in afterload, decrease in peak tension, increase in

50.If a substance is put into circulation by an organ, is its

pe

arteriovenous diff

ak tension

erence positive or negative? Negative

56.What two compensatory mechanisms occur to reverse hypoxia at

51.What is the baroreceptor response to an increase in blood

high altitudes?

pressure? Increase

1. Increase in erythropoietin 2. Increase in 2,3-bisphosphoglycerate

afferent activity of CN IX and CN X to decrease heart rate

(2,3-BPG)

(parasympathetic)

57.What would you give to neutralize the excess base in an alkalotic

52.What is a perfusionlimited situation? When alveolar and capillary

patient?

bloo

NH4Cl(strong acid can lyse RBCs)

a

pressure-volume

loop,

what

is

seen

with:

Aortic

58.What would you give to neutralize the excess acid in an acidotic

66.What phase of the female cycle ALWAYS lasts for the same

patient?

number of days (14 d

NaCO2 (CO2 eliminated by lungs)

ays in most women)? Luteal phase

59.What is a diffusion-limited situation? When alveolar gas and

67.What serves as a marker for 24-hour growth hormone secretion?

capillary

Plasma i

blood attempt to equilibrate but do NOT (i.e., CO2)

nsulin-like growth factor type 1 (IGF-1) levels

60.What must occur in order for PaCO2 to remain constant when

68.What three things inhibit the secretion of glucagon? 1. Insulin 2.

there is an increa

Somatostat

se in the body's metabolism? Need to increase alveolar ventilation (if

in 3. Hyperglycemia

not hy

69.Which three organs or structures have gluconeogenic capabilities?

percapnia would result)

1. Liver

61.What enzyme is needed for conversion of testosterone to

2. Kidney 3. GI epithelium

estradiol? Aromatas

70.Which type of diabetes is more likely to lead to ketoacidosis? Type

e

I(

62.What two anions compete with iodine for the iodine pump in the

insulin-dependent diabetes mellitus [IDDM])

thyroid gland?

71.Excess bone demineralization and remodeling can be detected by

1. Perchlorate 2. Thiocyanate

checking urine

63.What enzyme is associated with osteoblastic activity? Alkaline

levels of what substance? Hydroxyproline (breakdown product of

phospha

collagen)

tase

72.What two things cause l-a-hydroxylase activity to increase? 1.

64.What form of plasma calcium is the physiologically active form and

Parathyroid h

is regulat

ormone (PTH) 2. A decrease in PO4 levels

ed within narrow limits? Free calcium (ionized)

73.What type of membrane is permeable to water and small solutes?

65.Which three factors cause the release of epinephrine from the

Selectiv

adrenal medulla

ely permeable membrane

? 1. Exercise 2. Emergencies (stress) 3. Exposure to cold (The three

74.What is the movement of ions in an electrical held known as?

Es)

Conductance

75.What two components of a body of water cannot be measured and

84.What are the four "stress" hormones? 1. Growth hormone (GH) 2.

need to be calc

Glucagon 3. Co

ulated? 1. Intracellular fluid (ICF) (water minus extracellular fluid) 2.

rtisol 4. Epinephrine

Inters

85.Proopiomelanocortin

titial fluid (ISF) (extracellular fluid minus plasma volume)

substances? 1. Adren

76.What phase of an action potential has the greatest rate of Na+

ocorticotrophic hormone (ACTH) 2. (alpha)-Lipotropins (melanotropins

influx?

and endorph

Phase 0

ins)

77.Which phase of an action potential requires energy? Phase 4 (via

86.What are the six substances that promote the secretion of insulin?

the Na+/K+

1. Gluco

pump)

se 2. Amino acid (arginine) 3. Gastrin inhibitory peptide (GIP) 4.

78.What type of muscle is associated with one T tubule and two

Glucagon 5. A

cisternae (triad)

lpha-Agonists 6. ACh

? Skeletal muscle

87.What is the thin filament that has the attachment site for the

79.What is the region of an axon where no myelin is found? Nodes of

crossbridges a

Ranvier

nd also activates adenosine triphosphatase (ATPase)? Actin

80.What types) of muscle contain the thin filament troponin? Skeletal

88.What types of muscle have a sarcomere? Skeletal and cardiac

and car

muscle

diac muscle

89.Where is the action potential generated on a neuron? Axon hillock

81.Where are antidiuretic hormone (ADH) and oxytocin produced?

90.What is the name for the load that the muscle is working against

The supraoptic a

during stimu

nd paraventricular nuclei of the hypothalamus

lation? Afterload

82.What is the only hormone to INCREASE with a DECREASE in

91.What type of contraction has an active tension when the length is

pituitary function?

shortened?

Prolactin

Isotonic contraction

83.Which enzyme converts cholesterol to pregnenolone? Desmolase-

92.What type of muscle has high creatinine phosphokinase (CPK),

rate limiting

high ATPase acti

step (RLS) in steroid hormone synthesis

(POMC)

is

cleaved

into

what

two

vity, and no myoglobin; is anaerobic; and is for short-term use? White

ase activity

mu

106.What type of muscle has end plates? Skeletal muscle

scle (fast)

107.What type of contraction has an active tension, but the overall

93.What type of muscle uses calmodulin? Smooth muscle

length of th

94.What thick filament has crossbridges and ATPase activity? Myosin

e con traction does not change and no work is done? Isometric

95.What causes actin-myosin crossbridge dissociation? Binding of

contraction

ATP

108.What thin filament covers the attachment site in resting muscle so

96.What is used as an index of cortisol secretions? Urine 17-OH

that the

steroids

crossbridges are unavailable for binding? Tropomyosin

97.What would be the two major consequences if the zona fasciculata

109.What is the load on a muscle in the relaxed state known as?

and the zona

Preload

reticularis were removed? 1. Circulatory failure 2. Inability to mobilize

110.Total tension - preload = what? Active tension (contraction)

energy stores

111.What types of muscle are uninuclear? Cardiac and smooth

98.How many carbons do androgens have? Androgens are 19-carbon

muscle

steroids.

112.In a contractile muscle, what is the source of the calcium?

99.How many carbons do estrogens have? Estrogens are 18-carbon

Sarcoplasmic ret

steroids. (Remov

iculum (The source is NOT extracellular.)

al of one carbon from an androgen = an estrogen.)

113.What is the maximum force of a contraction determined by? The

100.The level of what hormone tends to DECREASE with stress?

number of mo

Insulin

tor units activated during the contraction

101.On what two occasions are cortisol releasing hormone (CRH)

114.What types) of muscle have T tubules associated with them?

secretions elevat

Cardiac and skel

ed? 1. Early morning 2. During stress

etal muscle

102.What is an inhibitory interneuron known as? Renshaw neuron

115.What type of muscle has myoglobin, low CPK, and low ATPase

103.What is the summation of mechanical stimuli known as? Tetany

activity; is aero

104.What is the thin filament that binds to calcium? Troponin C

bic; and is for long-term use? Red muscle (slow-twitch muscle)

105.What determines the maximum velocity of shortening muscle?

116.What event signifies the first day of the menstrual cycle? The first

The muscle's ATP

day of

bleeding

126.What hormone thins cervical mucus, stimulates LH receptors on

117.What hormone is essential for induction of ovulation and

granulosa cell

formation of the co

s, elicits the LH surge, and increases proliferation of the uterine

rpus luteum? Luteinizing hormone (LH)

mucosal laye

118.How many days before the first day of bleeding is ovulation? 14

rs? Estradiol

days

127.What hormone is secreted by the placenta late in pregnancy,

in most women (Remember: The luteal phase is always constant.)

stimulates mamma

119.What is required to maintain lactation? Suckling (stimulates

ry growth during pregnancy, mobilizes energy stores from the mother

oxytocin se

so that the

cretion)

fetus can utilize them, and has an amino acid sequence like GH?

120.What hormone, in high levels, blocks milk production? Estrogen

Human chorionic

121.What hormone is necessary for maintenance of the corpus

somatomanunotropin (hCS) or human placental lactogen (hPL)

luteum for the first

128.What hormone causes an increase in the production of milk?

3 months of pregnancy? Human chorionic gonadotropin (hCG)- from

Prolactin

the trophoblast

129.What is the force necessary to collapse the lung known as? Lung

122.Up to how many hours after ejaculation are sperm able to fertilize

recoil

the egg?

130.For what hormone do Leydig cells have receptors? LH

72 hours

131.What vitamin needs thyroid hormone for conversion to its active

123.What hormone induces myometrial contraction and causes milk

form?

letdown?

Vitamin A

Oxytocin

132.What is the tonicity of fluid that leaves the loop of Henle?

124.What hormone is necessary for the maintenance of the uterine

Hypotoni

endometrium fro

c

m the fourth month of pregnancy on? Progesterone (Estrogen is

133.What enzyme converts androgens to estrogens? Aromatase

needed for pro

134.What does excess production of thyroid-stimulating hormone

gesterone to be effective.)

(TSH) cause?

125.How long after ovulation does fertilization occur? 8 to 25 hours

A goiter

135.What type of cell reabsorbs bone? Osteoclast; Blasts make;

te deficiencies? Before the collecting duct (There is no electrolyte dist

clasts take.

urbance in the collecting duct.)

136.What is the major form of androgen secreted from the adrenal

146.What substance is free filtered but partially reabsorbed by

gland? Dehydroe

passive mechanis

piandrosterone (DHEA)

ms? Urea

137.What cells of the genitourinary system produce testosterone in

147.What hormone promotes mobilization of energy stores, enhances

males?

the capacity o

Leydig cells

f glucagon and catecholamines, and increases the capacity to

138.What type of urine does ADH cause to be excreted? Hypertonic

withstand stress?

urine (becaus

Cortisol

e of the water reabsorption in the collecting duct)

148.What is used as an index of androgen secretion? Urine 17-

139.What is the term for the volume of plasma removed from a

ketosteroids

substance per unit

149.What are the pituitary hormones associated with: Thyrotropin

time? Clearance

releasing hormo

140.What is the most potent male sex steroid? Dihydrotestosterone -

ne (TRH)? Thyroid stimulating hormone (TSH)

DHT

150.What are the pituitary hormones associated with: Cortisol

141.What two substances stimulate Sertoli cells? Follicle stimulating

releasing hormone

hor

(CRH)? Adrenocorticotrophic hormone (ACTH)

mone (FSH) and testosterone

151.What are the pituitary hormones associated with: Gonadotropin

142.At which three sites in the body is T4 converted to T3? 1. Liver 2.

releasing horm

Kidne

one (GnRH)? Luteinizing hormone (LH) and follicle stimulating

y 3. Pituitary gland (via 5'-deiodinase enzyme)

hormone (FSH)

143.The fresh air being delivered to the respiratory zone per minute is

152.What are the pituitary hormones associated with: Growth

known as

hormone releasing ho

what? Alveolar ventilation (the first 150 ml is not included)

rmone (GHRH)? Growth hormone (GH)

144.What region of the lungs gets very little ventilation? Apex

153.What are the pituitary hormones associated with: Somatostatin?

145.Where does polyuric originate if the patient is dehydrated and has

Inhibits

electroly

GH secretion

154.What are the pituitary hormones associated with: Prolactin

162.Which

extravascular

chemoreceptor

detects

low

NaCI

inhibiting factor

concentrations? Macul

(PIF) [dopamine]? Inhibits prolactin secretion

a densa

155.Which hormones are released from the: Zona glomerulosa?

163.What is the major stimulus for cell division in chondroblasts?

Aldosterone (sal

Insulin-li

t)

ke growth factor-1 (IGF-1)

156.Which hormones are released from the: Zona fasciculata?

164.The total air in and out of the respiratory system per minute is

Cortisol (sugar)

known as wh

157.Which hormones are released from the: Zona reticularis?

at? The total ventilation (minute volume or minute ventilation)

Androgens (sex)

165.What is the major hormone secreted by the ovarian follicle? 17

158.Which hormones are released from the: Medulla? NE:Epi (1:4) If

alpha-Estradi

the zona

ol

glomerulosa were removed from the adrenal gland, what would be

166.What two conditions cause ADH to be released? 1. Low blood

seen?

volume 2. E

Decrease in Na+ causing a decrease in the ECF volume, leading to a

levated plasma volume (high solute concentration)

decrease in B

167.What cell converts androgens to estrogens? Granulosa cell

P, and eventually to circulatory shock and death

168.What hormone acts on this cell? FSH

159.What does subatmospheric pressure (negative) do to the lungs?

169.What three lung volumes cannot be measured with a spirometer?

It causes t

1. Resid

hem to expand (because of the decrease in intrathoracic pressure)

ual volume 2. Total lung capacity 3. Functional residual capacity

160.Where is the last conducting zone of the lungs? Terminal

170.What two conditions decrease the secretion of aldosterone? 1. An

bronchioles (No

increase i

gas exchange occurs here.)

n blood pressure 2. Weightlessness

161.Where is there summation, hyperpolarization of the postsynaptic

171.What cell in the female genitourinary system is stimulated by LH

membrane, an

and is the

increase in Cl- conductance, and local gradation? Inhibitory

site where androgens are produced? Thecal cell

postsynaptic po

172.What serves as a marker of endogenous insulin secretions? C

tential (IPSP)

peptide

173.What do you have when there is depolarization of the

183.What is the term for the air that can be taken in after normal

postsynaptic membrane o

inspiration?

wing to an influx of Na+, resulting in summation and local gradation?

Inspiratory reserve volume (IRV)

Excitato

184.What is the first zone of the lungs that is capable of O2

ry postsynaptic potential (EPSP)

exchange? Respir

174.What are days 15 to 28 in the female cycle known as? Luteal

atory bronchioles (because they have alveoli)

phase

185.What is the term for ventilation of underperfused alveoli? Alveolar

175.What hormone is secreted by the Sertoli cells to decrease FSH

dead sp

production?

ace

Inhibin

186.What is the temperature of the scrotum? 4 degrees cooler than

176.What hormone regulates osmolarity because it controls water

the body

excretion?

187.How is the lower temperature of the testes maintained? By a

ADH (It causes water reabsorption.)

countercurr

177.What is the term for the air in the system after maximal

ent heat exchanger in the spermatic cord

inspiration?

188.What happens to sex steroids, LH, and FSH: If the gonads are

Total lung capacity (TLC)

removed?

178.What is a sign of a Sertoli cell tumor in a man? Excess estradiol

Sex steroids decrease; LH increases; FSH increases.

in the

189.What happens to sex steroids, LH, and FSH: In postmenopausal

blood

women? Sex ster

179.What hormone is responsible for the negative feedback onto LH

oids decrease; LH increases; FSH increases.

and FSH of the

190.What happens to sex steroids, LH, and FSH: After the

anterior pituitary and positive feedback onto the granulosa cells?

administration of testo

Estrogen

sterone? Sex steroids increase; LH decreases; nothing happens to

180.What is the term for the total dead space of the lungs?

FSH.

Physiologic dead

191.What happens to sex steroids, LH, and FSH: After the

space

administration of inhib

181.The surge of what hormone induces ovulation? LH

in? Nothing happens to sex steroids; nothing happens to LH; FSH

182.What does positive pressure do to the lungs? It collapses them.

decreases.

192.What happens to sex steroids, LH, and FSH: With constant

201.What hormone level peaks 1 day before the surge of LH and FSH

infusion of GnRH?

in the female

Sex steroids decrease; LH decreases; FSH decreases (needs to be

cycle? Estradiol

given pulsatile)

202.What is the day after the LH surge in the female cycle known as?

.

Ovulatio

193.What region of the lungs is incapable of gas exchange? Anatomic

n

dead sp

203.By what mechanism does chronic constriction keep blood flow

ace

through the peni

194.What is the term for the amount of air that can never leave the

s low during non-aroused states? Alpha-Adrenergic mediated

lungs?

constriction

Residual volume

204.Days 1 to 7 of the female cycle are known as what? Menses

195.If you increase the depth of breathing, what ventilatory

205.The amount of air that enters or leaves the respiratory system in

parameters can be i

a single r

ncreased? Total ventilation and alveolar ventilation

espiratory cycle is known as what? Tidal volume

196.If you increase the rate of breathing, what ventilatory parameters

206.What part of the autonomic nervous system is responsible for the

can be in

movement of

creased? Total ventilation

semen through the vas deferens and related structures? Sympathetic

197.What is the term for the air left in the lungs after normal

nervous syst

expiration?

em

Functional residual capacity (FRC)

207.Which pancreatic cells secrete glucagon? Alpha cells

198.The lung volume from maximum inspiration to maximum

208.Which pancreatic cells secrete somatostatin? Delta cells

expiration is known as w

209.What term describes how easily a vessel stretches? Compliance

hat? Vital capacity (VC)

(pulse pressu

199.Where is renin produced? In the juxtaglomerular ( JG) cells of the

re is inversely proportional to compliance)

kidney

210.What is the most compliant artery in the body? Aorta

200.What phase of the female cycle occurs during days 1 to 15?

211.What is the best way to regulate mean arterial pressure? Via total

Follicular phase

periph eral resistance (TPR)

212.What is the term for resistance to ventricular outflow? Afterload

221.What process occurs when hydrostatic pressure exceeds plasma

213.What is the main determinant of resistance? The radius of the

oncotic pressur

vessel (also t

e? Filtration

he viscosity and length)

222.What vessels have the greatest blood volume? Systemic veins of

214.What is the relationship between Na+ reabsorption and O2

the lo

consumption?

wer extremities

An increase in Na+ causes Oz consumption to increase.

223.What vessels have the smallest total cross-sectional area? The

215.What vessels have the greatest cross-sectional area? Capillaries

aorta, then

216.What is the nontitratable acid that buffers secreted H+ in the

the vena cava

kidney buffer

224.What is the only way to increase O2 delivery to the myocardium?

ed as? NH4+ (ammonium). H2PO4 (dihydrogen phosphate) is the

Increase

titratable acid tha

the blood flow

t buffers secreted H+.

225.What happens to airway resistance during inspiration? It

217.What are the five ways to promote turbulent flow? 1. Increase

decreases. Sy

velocity 2.

mpathetics

Branching 3. Narrow orifice 4. Increase tube diameter 5. Decrease

resistance.

viscosity

226.What is the main drive for ventilation? The PCO2 of systemic

218.What part of the cardiovascular system has the lowest drop in

circulation

pressure?

227.Where does depolarization in the heart begin? From the apex to

Right atrium

the bas

219.What vessels are the resistance vessels and have the largest

e and from the endocardium to the epicardium

drop in pressur

228.Where is the greatest venous PO2 in resting tissue? Renal

e? Arterioles

circulation

220.What part of the autonomic nervous system is the main controller

229.The load on the muscle in the relaxed state is known as what?

of blood fl

Preload

ow when a person is at rest? Sympathetic nervous system (alpha

(also the end-diastolic volume [EDV])

constricts, be

230.What cells of the heart have the highest rate of automaticity?

ta-2 dialates.

Sinoatri

decrease

resistance;

parasympathetics

increase

al (SA) nodal cells 231.What

are

the

240.What are the normal values for: PaO2? 100 mmHg slowest

conducting

cells

of

the

heart?

241.What are the normal values for: PaCO2? 40 mmHg

Atrioventricular (AV) no

242.What are the normal values for: PvO2? 40 mmHg

dal cells

243.What are the normal values for: PvCO2? 47 mmHg

232.What is the main control of flow in exercising muscle? Vasodilator

244.What are the two ways to increase stroke volume? 1. Increase

meta

preload (EDV

bolites

) 2. Decrease end-systolic volume (ESV)

233.During what phase of the cardiac cycle do coronary vessels

245.What is the main factor that determines the glomerular filtration

receive their blo

rate (GFR)

od flow? Diastole

? Hydrostatic pressure

234.On the venous pressure curve, what do the following waves

246.What is the normal value for the GFR? 120 ml/min

represent -a wave,

247.What happens to pulmonary blood flow under conditions of low

-c wave, -v wave? Atrial contraction, Ventricular contraction, Atrial fill

alveolar PO2?

ing (venous filling) Atrial contraction, Venous

A decrease in blood flow secondary to vasoconstriction

235.What causes the second heart sound? Aortic closure

248What is the normal compensatory mechanism for a state of

236.Where is the second heart sound on an EKG? At the T wave

metabolic alkalosis?

237.What causes the diastolic interval to decrease? Increase in the

Hypoventilation (respiratory acidosis)

heart ra

249.What anion is excreted in large amounts in the urine in a patient

te

with a com

238.What happens to cerebral circulation during hypoventilation?

pensated alkalosis? Bicarbonate (alkaline urine)

Blood fl

250.Which hormone affects the osmolarity? ADH

ow increases because PCO2 is increased. (The opposite occurs

251.What are the sympathetic effects on the kidney? A decrease in

during hyperventila

GFR and an

tion.)

increase in filtration fraction: FF = GFR/RPF. (There is a larger

239.What is the main factor affecting PaCO2? Alveolar ventilation

decrease in t

(Hyperventi

he RPF than the GFR, resulting in an increase in the filtration

lation decreases PaCO2 and vice versa. Body metabolism also

fraction.)

affects PaCO2')

252.What are the effects of angiotensin II on the kidney? Constriction

262.What are the three characteristics of autoregulation? 1. Flow

of

independ

the efferent arterioles

ent of BP 2. Flow proportional to local metabolism 3. Flow

253.What four changes occur with an increase in contractility? 1.

independent of nervou

Increased slo

s reflexes

pe of action potential 2. Increased peak left ventricular pressure 3. I

263.What is/are the major autoregulators of: Cerebral circulation?

254.Increased rate of relaxation 4. Decreased systolic interval

Increase

255.What fibers of the heart have the lowest intrinsic rate of

in PCO2

automaticity?

264.What is/are the major autoregulators of: Coronary circulation?

Purkinje fibers

Decrease

256.What causes the first heart sound, and when does it occur on an

in PO2; increase in PCO2 and adenosine

EKG?

265.What is/are the major autoregulators of: Exercising skeletal

Mitral valve closure at the QRS complex

muscle?

257.What is the main determining factor of filtration fraction? Renal

Lactate

plasma flo

266.What process occurs if the capillary oncotic pressure is greater

w (decreases flow; increases filtration fraction)

than the hy

258.What is the normal osmolarity of the filtrate in the renal tubule?

drostatic pressure? Reabsorption

300 mOsm

267.What area of the circulatory system houses the greatest blood

259.What prohibits the filtering of protein anions into the renal tubule?

velocity?

The negative charge on the filtration membrane

Aorta

260.What are the fastest conducting fibers in the heart? Purkinje

268.What two things happen to cutaneous circulation when the

fibers

sympathetic nervous

261.From which point to which point does repolarization travel in the

system is stimulated? 1. Constriction of arterioles to decrease blood

heart?

flow 2.

From the base to the apex and from the epicardium to the

Constriction of the venous plexus to decrease the blood volume

endocardium (opposite o

269.Adenosine in the kidney, decreased PO2 in the lungs, and

f depolarization)

thromboxane A2 (TXA 2) have what effect in the circulation? Vasoconstriction

270.What is the period when higher than normal stimulation is

s and where total resistance is the SUM of the individual resistances?

required to induce

Vessels

a second action potential? Relative refractory period

connected in a series

271.During an action potential, what is the stimulus for opening the

280.What effect on a blood vessel does each of the following have:

Na+ channel

histamine, br

s? Depolarization

adykinin, prostaglandins (A2, E2, I2), nitric oxide, adenosine, an

272.What substance "affects" the action potential? Na+ conductance

increase in K

273.What substance "affects" the resting membrane potential? K+

+, H+, PCO2 and a decrease in PO2? Vasodilatory effect

conductance

281.What are three characteristics of a subthreshold potential? 1.

274.In which direction do osmotically active substances cause water

Graded 2. Sum

to move?

mation 3. Not propagated

Toward them

282.What is the depolarization phase of an action potential caused

275.What are the three tracers for total body water? 1. Urea 2.

by? Na+ infl

Thiourea 3. T

ux

itrated water

283.If the ventilation-perfusion ratio is less than 1, what part of the

276.What are the three characteristics of an action potential? 1. All or

lung is

none 2

involved and what physiologic process is occurring? The base,

. Propagated 3. No summation

because flow e

277.In what system is the second greatest blood volume found?

xceeds delivery of O2

Pulmonary system

284.How do you compensate for metabolic acidosis? Hyperventilate

278.What are the four ways to increase total peripheral resistance

(respirat

(TPR)?

ory alkalosis)

1. Decrease the radius 2. Increase the viscosity 3. Increase the length

285.How many liters of water are there in: Total body water? 42 L

4. Decre

How many liters of water are there in: ICF? 28 L

ase the number of parallel channels

How many liters of water are there in: ECF? 14 L

279.What type of system is a high resistance system with flow equal

How many liters of water are there in: ISF? 10.5 L

at all point

How many liters of water are there in: Plasma volume? 3.5 L

286.What hormone affects fluid volume? Aldosterone (Na+ content

is always less than any individual resistance, and the reciprocal of the

determines the

total

volume of the plasma.)

resistance is the sum of the reciprocal resistances? System

287.The repolarization phase of the action potential is caused by

connected in para

what? K+ efflu

llel

x (depolarization opens the gates)

296.What is happening to the renal arteriole in each of the following

288.What is the name of the period in which, no matter how strong the

situations

stimulus,

: Increased GFR, increased glomerular pressure, decreased RPF,

a second action potential cannot be generated? Absolute refractory

increased FF?

period

Constriction of efferent arteriole

289.What hormone is necessary to maintain normal thyroid hormone

297.What is happening to the renal arteriole in each of the following

levels?

situations

GH

: Decreased GFR, increased RPF, decreased glomerular pressure,

290.What region of the lung has the greatest blood flow? The base

decreased FF?

291.During inspiration, which region of the lung receives the greatest

Dilatation of the efferent arteriole

level of

298.What is happening to the renal arteriole in each of the following

ventilation: the apex or the base? The base

situations

292.What is the function of the stretch receptors in the lungs? To

: Decreased GFR, decreased RPF, decreased glomerular pressure?

prevent overd

Constriction of

istention of the lungs (inhibits inspiration)

the afferent arteriole

293.Where does the inherent rhythm for respiration originate? In the

299.What is happening to the renal arteriole in each of the following

medullary

situations

center of the medullary oblongata

: Increased GFR, increased RPF, increased glomerular pressure?

294.Where is the deep breathing center located? Apneustic center in

Dilatation of th

the pons

e afferent arteriole

295.What type of system is a low-resistance system in which the total

300.Which region in the lungs gives the best ventilation - perfusion

resistance

ratio? The hilum

301.What causes peripheral chemoreceptors to be stimulated? A

2. Increased concentration gradient 3. Decreased thickness of the

decrease in di

membrane

e arterial PO2, H+, and PCO2 of the normal drive for ventilation)

310.What type of dehydration is associated with Addison's disease?

302.What is secreted by the parafollicular C cells of the thyroid?

Hypotoni

Calciton

c dehydration

in

311.When is GH released? At night and during puberty

303.What is the titrated acid that the secreted H+ is buffered as?

312.What is protein-mediated transportation down a concentration

H2PO4

gradient known

304.What type of dehydration is associated with hemorrhage, burns,

as? Facilitated transport

vomiting, and

313.What determines the level of alveolar ventilation? Central

diarrhea? Isotonic dehydration

chemoreceptors (

305.What is the potential at which concentrations are equal and

PCO2)

opposite to the

314.Which point in the lungs is involved if the ventilation - perfusion

electrical forces, and also at which there is no net flux of ions across

ratio is

the mem

greater than 1? Apex

brane? Equilibrium potential (Nernst's equation)

315.Why is the V-P ratio in the apex of the lung greater than 1?

306.What hydration state is caused by the ingestion of salt water?

Delivery

Hyperton

exceeds the flow

ic overhydration

316.How is CO2 carried in the blood? As plasma bicarbonate

307.What is the free water clearance if the osmolarity of urine is

317.To what hydrated state can excess ingestion of water or

greater than

syndrome of inapprop

300 mOsm? Negative free water clearance (concentrated urine)

riate antidiuretic hormone (SIADH) lead? Hypotonic overhydration

308.What is the term for the process of water traveling from a low

318.What are the four major anabolic hormones? 1. Insulin 2. Thyroid

solute to a h

hormone 3.

igh solute concentration? Osmosis

GH 4. Sex steroids

309.What three factors increase simple diffusion? 1. Increased

319.What are the eight insulin INdependent tissues? 1. CNS 2. RBCs

solubility

3. Renal tubules 4. Testis 5. Teeth 6. ß cells 7. Liver 8. Intestinal epithelium

320.What are the growth factors released from the liver called?

332.During what part of the cardiac cycle do you hear: Aortic

Somatomedins

regurgitation?

321.What state of hydration would you be in if you had edema and if

Diastole

you ingested

333.What four factors affect the rate of diffusion for any process? 1.

an excessive amount of salt? Isotonic overhydration

Surfa

322.What type of cell lays down bone? Osteoblast Remember: Blasts

ce area 2. Thickness of the membrane 3. Concentration gradient 4.

make; clasts

Solubility (ma

take.

in factor)

323.What is the only condition in which giving enriched O2 will not

334.What does a decrease in GH in adolescence lead to? Dwarfism

significantl

335.What type of transportation requires ATP and is protein mediated

y increase PaO2? Pulmonary shunt

against a c

324.What is the biologically active form of thyroid hormone? T3

oncentration gradient? Primary active transportation

325.For how many months can you store thyroid hormone? 2 to 3

336.What does an increase in GH in adolescence lead to? Gigantism

months

337.What gas has a low driving force but high solubility? CO2

326.What is the ratio of T4 to T3? 20:01

338.What are two causes of diffusion impairment in the lungs? 1.

327.What type of cell is surrounded by mineralized bone? Osteocvte

Decrease in s

328.What type of dehydration is associated with excess sweating,

urface area 2. Increase in membrane thickness (PAO2 > PaO2)

decreased water

339.What is evident in the urinalysis of a compensated acidotic

intake, fever, alcoholism, lithium salts, excess evaporation, and

patient?

diabetes insi

Low HCO3 - excretion (acidotic)

pidus? Hypertonic dehydration

340.What does angiotensin II do to restore blood pressure? It has a

329.During what part of the cardiac cycle do you hear: Aortic

direct

stenosis? Systole

vasoconstrictive effect.

330.During what part of the cardiac cycle do you hear: Mitral

341.When is systemic venous blood delivered to the left side of the

stenosis? Diastole

heart withou

331.During what part of the cardiac cycle do you hear: Mitral

t O2 exchange in the alveoli? In a pulmonary shunt

regurgitation?

342.What gas has a high driving force and low solubility? O2

Systole (pan)

343.In what type of shunt do you see an increase in right atrial,

R)

ventricular, a

351.From which point to which point does it appear on an EKG? From

nd pulmonary arterial PO2, along with an increase in pulmonary blood

the QRS to

flow?

the T wave (S1 to S2)

Left-to-right shunt

352.What are the four features of aortic stenosis? 1. Increase in

344.What causes Ca+ and PO4 to be reabsorbed from the kidney and

afterload

Ca+ and PO4 to

2. Increase in LV pressure 3. Increase in the pressure gradient

be absorbed from the GI tract, and also promotes bone synthesis?

between the LV

Vitamin

and the aorta 4. Crescendo-decrescendo systolic ejection murmur

D3

(early systolic

345.What does excess secretion of GH in an adult lead to?

ejection click)

Acromegaly

353.When is surface tension the greatest in a respiratory cycle? At the

346.The rate at which a substance is filtered into Bowman's capsule is

e

known as

nd of inspiration

what? Filtered load rate (GFR X plasma concentration)

354.What type of transportation requires ATP, can be co- or

347.What part of the nephron has the greatest osmolarity? Tip of the

countertransport, an

loop

d is a protein-mediated transport with a concentration gradient?

of Henle (1200 mOsm)

Secondar

348.At what region of the nephron does H+/HCO3 - exchange occur?

y active transportation

Distal t

355.What lung pathology is associated with a decrease in FEV1/FVC?

ubule

COPD (ob

349.During what type of heart block do the atria and the ventricles

structive)

beat indepen

356.What is the most important factor in describing lung recoil?

dently of each other? Third-degree heart block

Surface

350.What is the length of systole on a pressure curve? From the

tension (also fibers of tissue)

beginning of th

357.What is the free water clearance if the osmolarity of urine is less

e isovolumic contraction (IVC) to the beginning of the isovolumic

than 300

relaxation (IV

mOsm? Positive free water clearance (dilute urine)

358.What four characteristics are common to all protein-mediated

node and PR intervals greater than 0.21 second? First-degree heart

transportation?

block

1. More rapid than diffusion 2. Zero-order kinetics 3. Chemical

365.What two occurrences cause an increase in the force of

specificity 4. C

contraction? 1. Incre

ompetition for carriers

ase in preload 2. Increase in contractility by increased intracellular

359.What causes an increase in Na+ and water loss from the kidney

Ca+

by increasing

366.What are three features of mural regurgitation? 1. Increase in v

GFR, stimulated by stress and high Na+ concentrations? Atrial

wave 2.

natriuretic facto

Increase in preload 3. Increase in atrial pressure and volume

r (ANF) released from the right atrium

367.What is the length of diastole on a pressure curve, and where is it

360.What growth factors are chondrogenic, working on the epiphyseal

on an EK

end plates o

G? From the beginning of the IVR to the beginning of the IVC, and

f bone? Somatomedins (insulin-like growth factor type 1[IGF-1])

from the

361.What causes an increase in Ca+ reabsorption from the distal

T wave to the QRS complex (S2 to Sl)

tubule, a decrea

368.What are the three features of aortic regurgitation? 1. Increase in

se in PO4 reabsorption from the kidney, and an increase in Ca+ and

p

PO4 reabsorpt

reload 2. Increase in systolic pressure 3. Decrease in aortic diastolic

ion from the GI tract? Parathyroid hormone (PTH)

pressure

362.If a patient is irritated, excited, and emotionally unstable and has

369.What are the three features of mitral stenosis? 1. Increase in a

overall

wave 2.

symptoms of ß-adrenergic stimulation, would you assume that this

Decrease in LV filling 3. Increase in atrioventricular pressure

patient is hyper

370.What type of heart block is characterized by: Progressive

thyroidic or hypothyroidic? Hyperthyroidic

lengthening of the

363.What is needed for proper postnatal and perinatal mental growth

P-R interval until there is failure of the impulse to be transmitted?

and also for

Second-d

proper bone ossification and GH secretion? Thyroid hormone

egree heart block, Wenckebach (Mobitz type I)

364.What type of heart block is associated with slowed conduction

371.What type of heart block is characterized by: Constant P-R

through the AV

interval but with

an occasional failure of conduction, resulting in an atrial rate greater

377.What has happened if the amount filtered and the amount

than t

excreted per unit ti

he ventricular rate? Second-degree heart block, non-Wenckebach

me are the same? Nothing; there has been no tubular modification.

(Mobitz type I

378.What happens to the following parameters in an obstructive

I)

versus restrictiv

372.What are the three functions of surfactant? 1. Increased

e lung problem: Lung recoil?

compliance 2. Decre

Decrease (obstructive); increase (restrictive)

ased surface tension 3. Decreased probability of pulmonary edema

379.What happens to the following parameters in an obstructive

formation

versus restrictiv

373.More negative intrathoracic pressure causes what to happen to

e lung problem: FRO?

systemic venou

Increase (obstructive); decrease (restrictive)

s return and what to the pulmonary vessels? Promotes systemic

380.What happens to the following parameters in an obstructive

venous return

versus restrictiv

into the chest and increases the caliber and volume of the pulmonary

e lung problem: TLC?

vessels

Increase (obstructive); decrease (restrictive)

374.What four factors cause the oxygen-hemoglobin dissociation

381.What happens to the following parameters in an obstructive

curve to shift to

versus restrictiv

the right? 1. Increased PCO2 2. Decreased pH 3. Increased 2,3-BPG

e lung problem: FVC?

4. Increa

Decrease (obstructive); decrease (restrictive)

sed temperature

382.What happens to the following parameters in an obstructive

375.What part of respiration, on a pressure volume curve, acts "like

versus restrictiv

the chest w

e lung problem: FEVI?

all"? Inspiration (collapse is due to elastic recoil)

Decrease (obstructive); decrease (restrictive)

376."Secretion + filtration =

383.What happens to the following parameters in an obstructive

Excretion" is the transport maximum (Tm) for what substance?

versus restrictiv

Paraaminohippura

e lung problem: Peak flow?

te (PAH)

Decrease (obstructive); increase (restrictive)

384.What happens to the following parameters in an obstructive

arriers are saturated and the excess is excreted in the urine? The

versus restrictiv

transport ma

e lung problem: RV?

ximum has been reached (Tm glucose = 375 mg/min).

Increase (obstructive); decrease (restrictive)

341.What four factors cause aldosterone to be released? 1.

385.What hormone increases reabsorption of Na+ by the principal

Conversion of angiote

cells and promot

nsin I to angiotensin II 2. Hyperkalemia 3. Hyponatremia 4. A

es excretion of H+ and K+ by the intercalated cells of the kidney?

decrease in blood

Aldoster

volume

one

342.What disease state includes buffalo hump, moon fades,

386.What three situations cause the rennin-angiotensin-aldosterone

hyperglycemia, hyperli

axis to fire?

pidemia, hypertension, hypokalemia, osteoporosis, and thinning of the

1. A decrease in blood pressure in the afferent arteriole 2. Low Na+

hair?

levels at t

Cushing's disease

he macula densa 3. ß1-Sympathetic nervous system input

343.What condition involves high urine flow, low urine osmolarity, low

387.What pathology is associated with low ACTH levels and high

ECF volum

levels of cortiso

e, high ECF osmolarity, low ICF volume, and high ICF osmolarity?

l? Cushing's syndrome (adrenal)

Diabetes

388.When do you see low urine flow, high urine osmolarity, high ECF

insipidus (lose water)

volume, low

344.Which condition involves elevated ACTH and cortisol levels?

ECF osmolarity (low Na+), high ICF volume, and low ICF osmolarity?

Cushing's diseas

SIADH (w

e (pituitary tumor)

ater retention)

345.Which condition involves high ACTH, low cortisol, high ADH,

389.What is the term for the process in which excretion is less than

elevated renin l

the filtere

evels, hypotension, and low body hair? Addison's disease (primary

d load? Net positive reabsorption (glucose, Na+, urea)

adrenal insuf

340.What has happened when everything that is filtered is reabsorbed

ficiency)

until the c

346.What process has taken place in the kidney when excretion is greater than th

e filtered load? Net negative secretion (PAH, creatinine)

View more...

Comments

Copyright ©2017 KUPDF Inc.
SUPPORT KUPDF